Buy BOOKS at Discounted Price

The Straight Lines

Class 11th Mathematics RD Sharma Solution
Exercise 23.1
  1. - pi /4 Find the slopes of the lines which make the following angles with the…
  2. 2 pi /3 Find the slopes of the lines which make the following angles with the…
  3. 3 pi /4 Find the slopes of the lines which make the following angles with the…
  4. pi /3 Find the slopes of the lines which make the following angles with the…
  5. (- 3, 2) and (1, 4) Find the slopes of a line passing through the following…
  6. (at^2 1, 2at1) and (at^2 2, 2at2) Find the slopes of a line passing through…
  7. (3, - 5) and (1, 2) Find the slopes of a line passing through the following…
  8. State whether the two lines in each of the following are parallel,…
  9. Through (9, 5) and (- 1, 1); through (3, - 5) and 98, - 3) State whether the…
  10. Through (6, 3) and (1,1); through (- 2, 5) and (2, - 5) State whether the two…
  11. Through (3, 15) and (16, 6); through (- 5, 3) and (8, 2) State whether the two…
  12. Find the slopes of a line (i) which bisects the first quadrant angle (ii) which…
  13. A(4, 8), b(5, 12), C(9, 28) Using the method of slopes show that the following…
  14. A(16, - 18), B(3, - 6), C(- 10, 6) Using the method of slopes show that the…
  15. What is the value of y so that the line through (3, y) and (2, 7) is parallel…
  16. What can be said regarding a line if its slope is (i) zero (ii) positive (iii)…
  17. Show that the line joining (2, - 3) and (- 5, 1) is parallel to the line…
  18. Show that the line joining (2, - 5) and (- 2, 5) is perpendicular to the line…
  19. Without using Pythagoras theorem, show that the points A(0, 4), B(1, 2), C(3,…
  20. Prove that the points (- 4, - 1), (- 2, - 4), (4, 0) and (2, 3) are the…
  21. If three points A(h, 0), P(a, b) and B(0, k) lie on a line, show that: a/h +…
  22. The slope of a line is double of the slope of another line. If tangents of the…
  23. Consider the following population and year graph: Find the slope of the line…
  24. Without using the distance formula, show that points (- 2, - 1), (4, 0), (3,…
  25. Find the angle between the X - axis and the line joining the points (3, - 1)…
  26. The line through the points (- 2, 6) and 94, 8) is perpendicular to the line…
  27. Find the value of x for which the points (x, - 1), (2, 1) and (4, 5) are…
  28. Find the angle between X - axis and the line joining the points (3, - 1) and…
  29. By using the concept of slope, show that the points (- 2, - 1), (4, 0), (3, 3)…
  30. A quadrilateral has vertices (4, 1), (1, 7), (- 6, 0) and (- 1, - 9). Show…
Exercise 23.10
  1. 2x - y + 3 = 0 and x + y - 5 = 0 Find the point of intersection of the…
  2. bx + ay = ab and ax + by = ab Find the point of intersection of the following…
  3. y = m_1x + a/m_1 y = m_2x + a/m_2 Find the point of intersection of the…
  4. x + y - 4 = 0, 2x - y + 3 0 and x - 3y + 2 = 0 Find the coordinates of the…
  5. y(t1 + t2) = 2x + 2at1t2. y(t2 + t3) = 2x + 2at2t3 and, y(t3 + t1) = 2x +…
  6. y = m1x + c1, y = m2x + c2 and x = 0 Find the area of the triangle formed by…
  7. y = 0, x = 2 and x + 2y = 3 Find the area of the triangle formed by the lines…
  8. x + y - 6 = 0, x - 3y - 2 = 0 and 5x - 3y + 2 = 0 Find the area of the…
  9. Find the equations of the medians of a triangle, the equations of whose sides…
  10. Prove that the lines y = root 3x+1 , y = 4 and y = - root 3x+2 form an…
  11. Classify the following pairs of lines as coincident, parallel or intersecting:…
  12. Find the equation of the line joining the point (3, 5) to the point of…
  13. Find the equation of the line passing through the point of intersection of the…
  14. Show that the area of the triangle formed by the lines y = m1x, y = m2x and y =…
  15. If the straight line x/a + y/b = 1 passes through the point of intersection of…
  16. Find the orthocenter of the triangle the equations of whose sides are x + y =…
  17. Three sides AB, BC and CA of a triangle ABC are 5x - 3y + 2 = 0, x - 3y - 2 =…
  18. Find the coordinates of the orthocenter of the triangle whose vertices are (-…
  19. Find the coordinates of the incentre and centroid of the triangle whose sides…
  20. Prove that the lines root 3x+y = 0 , root 3y+x = 0 , root 3x+y = 1 and root…
  21. Find the equation of the line passing through the intersection of the lines 2x…
  22. Find the equation of the straight line passing through the point of…
Exercise 23.11
  1. 15x - 18y + 1 = 0, 12x + 10y - 3 = 0 and 6x + 66y - 11 = 0 Prove that the following sets…
  2. 3x - 5y - 11 = 0, 5x + 3y - 7 = 0 and x + 2y = 0 Prove that the following sets of three…
  3. and y = x. Prove that the following sets of three lines are concurrent:…
  4. For what value of λ are the three lines 2x - 5y + 3 = 0, 5x - 9y + λ = 0 and x - 2y + 1 =…
  5. Find the conditions that the straight lines y = m1x + c1, y = m2x + c2 and y = m3x + c3…
  6. If the lines p1x + q1y = 1, p2x + q2y = 1 and p3x + q3y = 1 be concurrent, show that the…
  7. Show that the straight lines L1 = (b + c)x + ay + 1 = 0, L2 = (c + a)x + by + 1 = 0 and L3…
  8. If the three lines ax + a^2 y + 1 = 0, bx + b^2 y + 1 = 0 and cx + c^2 y + 1 = 0 are…
  9. If a, b, c are in A. P., prove that the straight lines ax + 2y + 1 = 0, bx + 3y + 1 = 0…
  10. Show that the perpendicular bisectors of the sides of a triangle are concurrent.…
Exercise 23.12
  1. Find the equation of a line passing through the point (2, 3) and parallel to…
  2. Find the equation of a line passing through (3, -2) and perpendicular to the…
  3. Find the equation of the perpendicular bisector of the line joining the points…
  4. Find the equations of the altitudes of a ΔABC whose vertices are A (1, 4),…
  5. Find the equation of a line which is perpendicular to the line root 3x-y+5 = 0…
  6. If the image of the point (2, 1) with respect to a line mirror is (5, 2), find…
  7. Find the equation of the straight line through the point (α, β) and…
  8. Find the equation of the straight line perpendicular to 2x - 3y = 5 and cutting…
  9. Find the equation of the straight line perpendicular to 5x - 2y = 8 and which…
  10. Find the equation of the straight line which has y-intercept equal to 4/3 and…
  11. Find the equation of the right bisector of the line segment joining the points…
  12. Find the image of the point (2, 1) with respect to the line mirror x + y - 5 =…
  13. If the image of the point (2, 1) with respect to the line mirror be (5, 2),…
  14. Find the equation to the straight line parallel to 3x - 4y + 6 = 0 and passing…
  15. Prove that the lines 2x - 3y + 1 = 0, x + y = 3, 2x - 3y = 2 and x + y = 4…
  16. Find the equation of a line drawn perpendicular to the line x/4 + y/6 = 1…
  17. The perpendicular from the origin to the line y = mx + c meets it at the point…
  18. Find the equation of the right bisector of the line segment joining the points…
  19. The line through (h, 3) and (4, 1) intersects the line 7x - 9y - 19 = 0 at…
  20. Find the image of the point (3, 8) with respect to the line x + 3y = 7…
  21. Find the coordinates of the foot of the perpendicular from the point (-1, 3)…
  22. Find the projection of the point (1, 0) on the line joining the points (-1, 2)…
  23. Find the equation of a line perpendicular to the line root 3x-y+5 = 0 and at a…
  24. The line 2x + 3y = 12 meets the x-axis at A and y-axis at B. The line through…
  25. Find the equation of the straight line which cuts off intercepts on x-axis…
  26. The equations of perpendicular bisectors of the sides AB and AC of a triangle…
Exercise 23.13
  1. 3x + y + 12 = 0 and x + 2y - 1 = 0 Find the angles between each of the following pairs of…
  2. 3x - y + 5 = 0 and x - 3y + 1 = 0 Find the angles between each of the following pairs of…
  3. 3x + 4y - 7 = 0 and 4x - 3y + 5 = 0 Find the angles between each of the following pairs of…
  4. x - 4y = 3 and 6x - y = 11 Find the angles between each of the following pairs of straight…
  5. (m^2 - mn) y = (mn + n^2)x + n^3 and (mn + m^2)y = (mn - n^2)x + m^3 . Find the angles…
  6. Find the acute angle between the lines 2x - y + 3 = 0 and x + y + 2 = 0.…
  7. Prove that the points (2, -1), (0, 2), (2, 3) and (4, 0) are the coordinates of the…
  8. Find the angle between the line joining the points (2, 0), (0, 3) and the line x + y = 1.…
  9. If θ is the angle which the straight line joining the points (x1, y1) and (x2, y2)…
  10. Prove that the straight lines (a + b)x + (a - b)y = 2ab, (a - b)x + (a + b)y = 2ab and x +…
  11. Find the angle between the lines x = a and by + c = 0.
  12. Find the tangent of the angle between the lines which have intercepts 3, 4 and 1, 8 on the…
  13. Show that the line a^2 x + ay + 1 = 0 is perpendicular to the line x - ay = 1 for all…
  14. Show that the tangent of an angle between the lines and is .
Exercise 23.14
  1. Find the values of α so that the point P(α 2, α) lies inside or on the triangle…
  2. Find the values of the parameter a so that the point (a, 2) is an interior…
  3. Determine whether the point (-3, 2) lies inside or outside the triangle whose…
Exercise 23.15
  1. Find the distance of the point (4, 5) from the straight line 3x - 5y + 7 = 0.…
  2. Find the perpendicular distance of the line joining the points (cosθ, sinθ) and (cosϕ,…
  3. Find the length of the perpendicular from the origin to the straight line joining the two…
  4. Show that the perpendicular let fall from any point on the straight line 2x + 11y - 5 = 0…
  5. Find the distance of the point of intersection of the lines 2x + 3y = 21 and 3x - 4y + 11…
  6. Find the length of the perpendicular from the point (4, -7) to the line joining the origin…
  7. What are the points on X-axis whose perpendicular distance from the straight line is a ?…
  8. Show that the product of perpendicular on the line from the points is b^2 .…
  9. Find the perpendicular distance from the origin of the perpendicular from the point (1, 2)…
  10. Find the distance of the point (1, 2) from the straight line with slope 5 and passing…
  11. What are the points on y-axis whose distance from the line is 4 units?…
  12. In the triangle ABC with vertices A(2, 3), B(4, -1) and C(1, 2) find the equation and the…
  13. Show that the path of a moving point such that its distances from two lines 3x - 2y = 5…
  14. If sum of perpendicular distances of a variable point P(x, y) from the lines x + y - 5 = 0…
  15. If the length of the perpendicular from the point (1, 1) to the line ax - by + c = 0 be…
Exercise 23.16
  1. 4x - 3y - 9 = 0 and 4x - 3y - 24 = 0 Determine the distance between the…
  2. 8x + 15y - 34 = 0 and 8x + 15y + 31 = 0 Determine the distance between the…
  3. y = mx + c and y = mx + d Determine the distance between the following pair of…
  4. 4x + 3y - 11 = 0 and 8x + 6y = 15 Determine the distance between the following…
  5. The equations of two sides of a square are 5x - 12y - 65 = 0 and 5x - 12y + 26…
  6. Find the equation of two straight lines which are parallel to x + 7y + 2 = 0…
  7. Prove that the lines 2x + 3y = 19 and 2x + 3y + 7 = 0 are equidistant from the…
  8. Find the equation of the line mid-way between the parallel lines 9x + 6y - 7 =…
  9. Find the ratio in which the line 3x + 4y + 2 = 0 divides the distance between…
Exercise 23.17
  1. Prove that the area of the parallelogram formed by the lines a1x + b1y + c1 = 0, a1x + b1y…
  2. Prove that the area of the parallelogram formed by the lines 3x - 4y + a = 0, 3x - 4y + 3a…
  3. Show that the diagonals of the parallelogram whose sides are lx + my + n = 0, lx + my + n’…
Exercise 23.18
  1. Find the equation of the straight lines passing through the origin and making an angle of…
  2. Find the equations to the straight lines which pass through the origin and are inclined at…
  3. Find the equations of straight lines passing through (2, -1) and making an angle of 45^0…
  4. Find the equations to the straight lines which pass through the point (h, k) and are…
  5. Find the equations to the straight lines passing through the point (2, 3) and inclined at…
  6. Find the equations to the sides of an isosceles right angled triangle the equation of…
  7. The equation of one side of an equilateral triangle is x - y = 0 and one vertex is . Prove…
  8. Find the equations of the two straight lines through (1, 2) forming two sides of a square…
  9. Find the equations of two straight lines passing through (1, 2) and making an angle of…
  10. Two sides of an isosceles triangle are given by the equations 7x - y + 3 = 0 and x + y - 3…
  11. Show that the point (3, -5) lies between the parallel lines 2x + 3y - 7 = 0 and 2x + 3y +…
  12. The equation of the base of an equilateral triangle is x + y = 2 and its vertex is (2,…
  13. If two opposites vertices of a square are (1, 2) and (5, 8), find the coordinates of its…
Exercise 23.19
  1. Find the equation of a straight line through the point of intersection of the…
  2. Find the equation of a straight line passing through the point of intersection…
  3. Find the equation of the line passing through the point of intersection of 2x -…
  4. Find the equation of the straight line passing through the point of…
  5. Find the equation of the straight line drawn through the point of intersection…
  6. Prove that the family of lines represented by x(1 + λ) + y(2 - λ) + 5 = 0, λ…
  7. Show that the straight lines given by (2 + k)x + (1 + k)y = 5 + 7k for…
  8. Find the equation of the straight line passing through the point of…
  9. Find the equation of the straight line which passes through the point of…
  10. Find the equations of the lines through the point of intersection of the lines…
  11. Find the equations of the lines through the point of intersection of the lines…
Exercise 23.2
  1. Find the equation of the parallel to x-axis and passing through (3, - 5).…
  2. Find the equation of the line perpendicular to x-axis and having intercept - 2…
  3. Find the equation of the line parallel to x-axis and having intercept - 2 on y…
  4. Draw the lines x = - 3, x = 2, y = - 2, y = 3 and write the coordinates of the…
  5. Find the equations of the straight lines which pass through (4, 3) and are…
  6. Find the equation of the line which is equidistant from the lines x = -2 and x…
  7. Find the equation of a line equidistant from the lines y = 10 and y = - 2.…
Very Short Answer
  1. Write an equation representing a pair of lines through the point (a, b) and parallel to…
  2. Write the coordinates of the orthocenter of the triangle formed by the lines x^2 - y^2…
  3. If the centroid of a triangle formed by the points (0, 0), (cos θ, sin θ) and (sinθ, -…
  4. Write the value of for which area of the triangle formed by points O(0, 0), A(a cos θ, b…
  5. Write the distance the lines 4x + 3y - 11 = 0 and 8x + 6y - 15 = 0.…
  6. Write the coordinates of the orthocenter of the triangle formed by the lines xy = 0 and…
  7. If the lines x + ay + a = 0, bx + y + b = 0 and cx + cy + 1 = 0 are concurrent then write…
  8. Write the area of the triangle formed by the coordinate axes and the line (sec θ - tan…
  9. If the diagonals of the quadrilateral formed by the lines l1x + m1y + n1 = 0, l2x + m2y…
  10. Write the coordinates of the image of the point (3, 8) in the line x + 3y - 7 = 0.…
  11. Write the integral values of m for which the x-coordinate of the point of intersection…
  12. If a ≠ b ≠ c, write the condition for which the equation (b - c)x + (c - a) y + (a -…
  13. If a, b, c are in G.P. write the area of the triangle formed by the line ax + by + c =…
  14. Write the area of the figure formed by the lines a|x| + b|y| + c = 0…
  15. Write the locus of a point the sum of whose distances from the coordinate’s axes is unity.…
  16. If a, b, c are in A.P., then the line ax + by + c = 0 passes through a fixed point.…
  17. Write the equation of the line passing through the point (1, -2) and cutting off equal…
  18. Find the locus of the mid-points of the portion of the line x sin θ + y cos θ = p…
Mcq
  1. L is variable line such that the algebraic sum of the distances of the points (1, 1),…
  2. The acute angle between the medians drawn from the acute of a right angled isosceles…
  3. The distance between the orthocenter and circumcentre of the triangle with vertices (1,…
  4. The equation of the straight line which passes through the point (-4, 3) such that the…
  5. Which point which divides the join of (1, 2) and (3, 4) externally in the ratio of 1 :…
  6. A line passes through the point (2, 2) and is perpendicular to the line 3x + y = 3. Its…
  7. If the lines ax + 12y + 1 = 0, bx + 13y + 1 = 0 and cx + 14y - 1 = 0 are concurrent, then…
  8. The number of real values of λ for which the lines x - 2y + 3 = 0, λx + 3y + 1 = 0 and 4x…
  9. The equations of the sides AB, BC and CA of ΔABC are y - x = 2, x + 2y = 1 and 3x + y + 5…
  10. If p1 and p2 are the lengths of the perpendiculars form the origin upon the lines x…
  11. Area of the triangle formed by the points ((a + 3)(a + 4), a + 3), ((a + 2)(a + 3), (a…
  12. If a + b + c = 0, then the family of lines 3ax + by + 2c = 0 pass through fixed…
  13. The line segment joining the points (-3, -4) and (1, -2) is divided by y-axis in the…
  14. The area of a triangle with vertices at (-4, -1), (1, 2) and (4, -3) isA. 17 B. 16 C.…
  15. The line segment joining the points (1, 2) and (-2, 1) is divided by the line 3x + 4y…
  16. If the point (5, 2) bisects the intercept of a line between the axes, then its…
  17. A(6, 3), B(-3, 5), C(4, -2) and (x, 3x) are four points. If ΔDBC : ΔABC = 1 : 2, then…
  18. If p be the length of the perpendicular from the origin on the line x/a + y/b = 1,…
  19. If equation of the line passing through (1, 5) and perpendicular to the line 3x - 5y +…
  20. The figure formed by the lines ax ± by ± c = 0 isA. a rectangle B. a square C. a…
  21. Two vertices of a triangle are (-2, -1) and (3, 2) and third vertex lies on the line x…
  22. The inclination of the straight line passing through the point (-3, 6) and the mid-point…
  23. Distance between the lines 5x + 3y - 7 = 0 and 15x + 9y + 14 = 0 isA. 35/root 34 B.…
  24. The angle between the lines 2x - y + 3 = 0 and x + 2y + 3 = 0 isA. 90° B. 60° C. 45° D.…
  25. The value of λ for which the lines 3x + 4y = 5, 5x + 4y = 4 and λx + 4y = 6 meet at a…
  26. Three vertices of a parallelogram taken in order are (-1, -6), (2, -5) and (7, 2). The…
  27. The centroid of a triangle is (2, 7) and two of its vertices are (4, 80 and (-2, 6).…
  28. If the lines x + q = 0, y - 2 = 0 and 3x + 2y + 5 + 0 are concurrent, then the value of q…
  29. The medians AD and BE of a triangle with vertices A(0, b), B(0, 0) and C(a, 0) are…
  30. The equation of the line with slope -3/2 and which is concurrent with the lines 4x + 3y -…
  31. The vertices of a triangle are (6, 0), (0, 6) and (6, 6). The distance between its…
  32. A point equidistant from the line 4x + 3y + 10 = 0, 5x - 12y + 26 = 0 and 7x + 24y -…
  33. The ratio in which the line 3x + 4y + 2 = 0 divides the distance between the lines 3x…
  34. The coordinates of the foot of the perpendicular from the point (2, 3) on the line x +…
  35. The reflection of the point (4, -13) about the line 5x + y + 6 = 0 isA. (-1, -14) B.…
Exercise 23.3
  1. Find the equation of a line making an angle of 150° with the x-axis and cutting…
  2. Find the equation of a straight line: (i) with slope 2 and y - intercept 3;…
  3. Find the equations of the bisectors of the angles between the coordinate axes.…
  4. Find the equation of a line which makes an angle of tan - 1 (3) with the x-axis…
  5. Find the equation of a line that has y - intercept - 4 and is parallel to the…
  6. Find the equation of a line which is perpendicular to the line joining (4, 2)…
  7. Find the equation of the perpendicular to the line segment joining (4, 3) and…
  8. Find the equation of the straight line intersecting y - axis at a distance of 2…
Exercise 23.4
  1. Find the equation of the straight line passing through the point (6, 2) and…
  2. Find the equation of the straight line passing through (- 2, 3) and indicated…
  3. Find the equation of the line passing through (0, 0) with slope m…
  4. Find the equation of the line passing through (2, 2 root 3) and inclined with x…
  5. Find the equation of the straight line which passes through the point (1,2) and…
  6. Find the equation of the straight line passing through (3, - 2) and making an…
  7. Find the lines through the point (0, 2) making angles pi /3 and 2 pi /3 with…
  8. Find the equations of the straight lines which cut off an intercept 5 from the…
  9. Find the equation of the line which intercepts a length 2 on the positive…
  10. Find the equation of the straight line which divides the join of the points…
  11. Prove that the perpendicular drawn from the point (4, 1) on the join of (2, -…
  12. Find the equations to the altitudes of the triangle whose angular points are A…
  13. Find the equation of the right bisector of the line segment joining the points…
  14. Find the equation of the line passing through the point (- 3, 5) and…
  15. Find the equation of the right bisector of the line segment joining the points…
Exercise 23.5
  1. (0, 0) and (2, - 2) Find the equation of the straight lines passing through…
  2. (a, b) and (a + c sin α, b + c cos α) Find the equation of the straight lines…
  3. (0, - a) and (b, 0) Find the equation of the straight lines passing through…
  4. (a, b) and (a + b, a - b) Find the equation of the straight lines passing…
  5. (at1, a/t1) and (at2, a/t2) Find the equation of the straight lines passing…
  6. (a cos α, a sin α) and (a cos β, a sin β) Find the equation of the straight…
  7. (1,4), (2, - 3) and (-1, - 2) Find the equations to the sides of the triangles…
  8. (0,1), (2, 0) and (-1, - 2) Find the equations to the sides of the triangles…
  9. Find the equations of the medians of a triangle, the coordinates of whose…
  10. Find the equations to the diagonals of the rectangle the equations of whose…
  11. Find the equation of the side BC of the triangle ABC whose vertices are A (-1,…
  12. By using the concept of the equation of a line, prove that the three points (-…
  13. Prove that the line y - x + 2 = 0 divides the join of points (3,-1) and (8, 9)…
  14. Find the equation to the straight line which bisects the distance between the…
  15. In what ratio is the line joining the points (2, 3) and (4, -5) divided by the…
  16. The vertices of a quadrilateral are A (-2, 6), B (1, 2), C (10, 4) and D (7,…
  17. The length L (in centimeters) of a copper rod is a linear function of its…
  18. The owner of a milk store finds that he can sell 980 liters milk each week at…
  19. Find the equation of the bisector of angle A of the triangle whose vertices…
  20. Find the equations to the straight lines which go through the origin and…
  21. Find the equations of the diagonals of the square formed by the lines x = 0, y…
Exercise 23.6
  1. cutting off intercepts 3 and 2 from the axes. Find the equation to the…
  2. cutting off intercepts -5 and 6 from the axes. Find the equation to the…
  3. Find the equation of the straight line which passes through (1, -2) and cuts…
  4. Equal in magnitude and both positive Find the equation to the straight line…
  5. Equal in magnitude but opposite in sign Find the equation to the straight line…
  6. For what values of a and b the intercepts cut off on the coordinate axes by the…
  7. Find the equation to the straight line which cuts off equal positive intercepts…
  8. Find the equation of the line which passes through the point (-4, 3) and the…
  9. A straight line passes through the point (α, β) and this point bisects the…
  10. Find the equation of the line which passes through the point (3, 4) and is such…
  11. Point R (h, k) divided line segments between the axes in the ratio 1 : 2. Find…
  12. Find the equation of the straight line which passes through the point (-3, 8)…
  13. Find the equation to the straight line which passes through the point (-4, 3)…
  14. Find the equation of a line which passes through the point (22, -6) and is…
  15. Find the equation of the line, which passes through P(1, -7) and meets the…
  16. Find the equation of the line passing through the point (2, 2) and cutting off…
  17. Find the equation of the straight line which passes through the point P(2, 6)…
  18. Find the equations of the straight lines each of which passes through the…
  19. Find the equations of the straight lines which pass through the origin and…
  20. Find the equation of the straight line passing through the point (2, 1) and…
  21. Find the equation of the straight line passing through the origin and…
Exercise 23.7
  1. p = 5, α = 60° Find the equation of a line for which
  2. p = 4, α = 150° Find the equation of a line for which
  3. p = 8, α = 225° Find the equation of a line for which
  4. p = 8, α = 300° Find the equation of a line for which
  5. Find the equation of the line on which the length of the perpendicular segment…
  6. Find the equation of the line whose perpendicular distance from the origin is 4…
  7. Find the equation of the straight line at a distance of 3 units from the origin…
  8. Find the equation of the straight line on which the length of the perpendicular…
  9. Find the equation of the straight line upon which the length of the…
  10. The length of the perpendicular from the origin to a line is 7, and the line…
  11. Find the value of θ and p if the equation x cos θ + y sin θ = p is the normal…
  12. Find the equation of the straight line which makes a triangle of the area 96…
  13. Find the equation of a straight line on which the perpendicular from the…
Exercise 23.8
  1. A line passes through a point A (1, 2) and makes an angle of 60^0 with the…
  2. A straight line drawn through the point A (2, 1) making an angle pi /4 with…
  3. A line a drawn through A (4, - 1) parallel to the line 3x - 4y + 1 = 0. Find…
  4. The straight line through P(x1, y1) inclined at an angle θ with the x-axis…
  5. Find the distance of the point (2, 3) from the line 2x - 3y + 9 = 0 measured…
  6. Find the distance of the point (3, 5) from the line 2x + 3y = 14 measured…
  7. Find the distance of the point (2, 5) from the line 3x + y + 4 = 0 measured…
  8. Find the distance of the point (3, 5) from the line 2x + 3y = 14 measured…
  9. Find the distance of the line 2x + y = 3 from the point (- 1, - 3) in the…
  10. A line is such that its segment between the straight line 5x - y - 4 = 0 and…
  11. Find the equation of straight line passing through (- 2, - 7) and having an…
Exercise 23.9
  1. Reduce the equation root 3x+y+2 = 0 to: (i) slope - intercept form and find…
  2. x + root 3y-4 = 0 Reduce the following equations to the normal form and find p…
  3. x+y + root 2 = 0 Reduce the following equations to the normal form and find p…
  4. x-y+2 root 2 = 0 Reduce the following equations to the normal form and find p…
  5. x - 3 = 0 Reduce the following equations to the normal form and find p and α…
  6. y - 2 = 0 Reduce the following equations to the normal form and find p and α…
  7. Put the equation x/a + y/b = 1 the slope intercept form and find its slope and…
  8. Reduce the lines 3x - 4y + 4 = 0 and 2x + 4y - 5 = 0 to the normal form and…
  9. Show that the origin is equidistant from the lines 4x + 3y + 10 = 0; 5x - 12y +…
  10. Find the values of θ and p, if the equation x cos θ + y sin θ = p is the normal…
  11. Reduce the equation 3x - 2y + 6 = 0 to the intercept form and find the x and y…
  12. The perpendicular distance of a line from the origin is 5 units, and its slope…

Exercise 23.1
Question 1.

Find the slopes of the lines which make the following angles with the positive direction of x - axis :



Answer:

Given

To Find: Slope of the line


Angle made with the positive x - axis is


The Slope of the line is m


Formula Used: m = tanθ


So, The slope of Line is m = tan = – 1


Hence, The slope of the line is – 1.



Question 2.

Find the slopes of the lines which make the following angles with the positive direction of x - axis :



Answer:

Given

To Find: Slope of the line


Angle made with the positive x - axis is


The Slope of the line is m


Formula Used: m = tanθ


So, The slope of Line is m = tan





Hence, The slope of the line is .



Question 3.

Find the slopes of the lines which make the following angles with the positive direction of x - axis :



Answer:

Given

To Find: Slope of the line


Angle made with positive x - axis is


The Slope of the line is m


Formula Used: m = tanθ


So, The slope of Line is m = tan





Hence, The slope of the line is – 1.



Question 4.

Find the slopes of the lines which make the following angles with the positive direction of x - axis :



Answer:

Given

To Find: Slope of the line


Angle made with positive x - axis is


The Slope of the line is m


Formula Used: m = tan θ


So, The slope of Line is m = tan



Hence, The slope of the line is √3.



Question 5.

Find the slopes of a line passing through the following points :

(– 3, 2) and (1, 4)


Answer:

Given (– 3, 2) and (1, 4)

To Find The slope of the line passing through the given points.


Here,


The formula used: Slope of line =


So, The slope of the line, m =


m =


Hence, The slope of the line is



Question 6.

Find the slopes of a line passing through the following points :

(at21, 2at1) and (at22, 2at2)


Answer:

Given (at21, 2at1) and (at22, 2at2)

To Find: The slope of the line passing through the given points.


The formula used: Slope of line =


So, The slope of the line, m =


m =


m =


[Since, (a2 – b2 = (a – b)(a + b)]


m =


Hence, The slope of the line is



Question 7.

Find the slopes of a line passing through the following points :

(3, – 5) and (1, 2)


Answer:

Given (3, – 5) and (1, 2)

To Find: The slope of line passing through the given points.


Here,


The formula used: Slope of line =


So, The slope of the line, m =


m =


Hence, The slope of the line is



Question 8.

State whether the two lines in each of the following are parallel, perpendicular or neither :
Through (5, 6) and (2, 3); through (9, – 2) and (6, – 5)


Answer:

We have given Coordinates off two lines.

Given: (5, 6) and (2, 3); (9, – 2) and 96, – 5)


To Find: Check whether Given lines are perpendicular to each other or parallel to each other.


Concept Used: If the slopes of this line are equal the lines are parallel to each other. Similarly, If the product of the slopes of this two line is – 1, then lines are perpendicular to each other.


The formula used: Slope of a line, m =


Now, The slope of the line whose Coordinates are (5, 6) and (2, 3)




So, m1 = 1


Now, The slope of the line whose Coordinates are (9, – 2) and (6, – 5)




So, m2 = 1


Here, m1 = m2 = 1


Hence, The lines are parallel to each other.



Question 9.

State whether the two lines in each of the following are parallel, perpendicular or neither :

Through (9, 5) and (– 1, 1); through (3, – 5) and 98, – 3)


Answer:

We have given Coordinates off two line.

Given: (9, 5) and (– 1, 1); through (3, – 5) and (8, – 3)


To Find: Check whether Given lines are perpendicular to each other or parallel to each other.


Concept Used: If the slopes of this line are equal the the lines are parallel to each other. Similarly, If the product of the slopes of this two line is – 1, then lines are perpendicular to each other.


The formula used: Slope of a line, m =


Now, The slope of the line whose Coordinates are (9, 5) and (– 1, 1)




So, m1 =


Now, The slope of the line whose Coordinates are (3, – 5) and (8, – 3)




So, m2 =


Here, m1 = m2 =


Hence, The lines are parallel to each other.



Question 10.

State whether the two lines in each of the following are parallel, perpendicular or neither :

Through (6, 3) and (1,1); through (– 2, 5) and (2, – 5)


Answer:

We have given Coordinates off two line.

Given: (6, 3) and (1,1) and (– 2, 5) and (2, – 5)


To Find: Check whether Given lines are perpendicular to each other or parallel to each other.


Concept Used: If the slopes of this line are equal the the lines are parallel to each other. Similarly, If the product of the slopes of this two line is – 1, then lines are perpendicular to each other.


The formula used: Slope of a line, m =


Now, The slope of the line whose Coordinates are (6, 3) and (1, 1)




So, m1 =


Now, The slope of the line whose Coordinates are (– 2, 5) and (2, – 5)




So, m2 =


Here, m1m2 =


m1m2 = – 1


Hence, The line is perpendicular to other.



Question 11.

State whether the two lines in each of the following are parallel, perpendicular or neither :

Through (3, 15) and (16, 6); through (– 5, 3) and (8, 2)


Answer:

We have given Coordinates off two line.

Given: (3, 15) and (16, 6) and (– 5, 3) and (8, 2)


To Find: Check whether Given lines are perpendicular to each other or parallel to each other.


Now,


Concept Used: If the slopes of these line are equal the the lines are parallel to each other. Similarly, If the product of the slopes of these two line is – 1, then lines are perpendicular to each other.


The formula used: Slope of a line, m =


Now, The slope of the line whose Coordinates are (3, 15) and (16, 6)




So, m1 =


Now, The slope of the line whose Coordinates are (– 5, 3) and (8, 2)




So, m2 =


Here, m1≠m2 nor m1m2 = – 1


Hence, The lines are neither perpendicular and nor parallel to each other.



Question 12.

Find the slopes of a line

(i) which bisects the first quadrant angle

(ii) which makes an angle of 300 with the positive direction of y - axis measured anticlockwise.


Answer:

(i) Given, Line bisects the first quadrant

To Find: Find the slope of the line.


Here, If the line bisects in the first quadrant, then the angle must be between line and the positive direction of x - axis .


Since, Angle =


The formula used: The slope of the line, m = tan θ


Similarly, The slope of the line for a given angle is m = tan 45


m = 1


Hence, The slope of the line is 1.


(ii) To Find: Find the slope of the line.


Here, The line makes an angle of 30o with the positive direction of y - axis (Given)


Since Angle between line and positive side of axis = 90o + 30o = 120o


The formula used: The slope of the line, m = tan θ


Similarly, The slope of the line for a given angle is m = tan 120o


m = – √3


Hence, The slope of the line is – √3 .



Question 13.

Using the method of slopes show that the following points are collinear:

A(4, 8), b(5, 12), C(9, 28)


Answer:

We have three points given A(4, 8), b(5, 12), C(9, 28)

To Prove: Given Points are collinear


Proof: A(4, 8), B(5, 12), C(9, 28)


The formula used: The slope of the line =


The slope of line AB =


AB =


The slope of line BC =


BC = = 4


The slope of line CA =



Here, AB = BC = CA


Hence, The Given points are collinear.



Question 14.

Using the method of slopes show that the following points are collinear:

A(16, – 18), B(3, – 6), C(– 10, 6)


Answer:

We have three points given A(16, – 18), B(3, – 6), C(– 10, 6)

To Prove: Given Points are collinear


Proof: AB[(16, – 18),(3, – 6)], BC[(3, – 6),(– 10, 6)], CA[(– 10, 6),(16, – 18)]


Formula used: The slope of the line =


The slope of line AB =


AB =


The slope of line BC =


BC =


The slope of line CA =


CA =


Here, AB = BC = CA


Hence, The Given points are collinear.



Question 15.

What is the value of y so that the line through (3, y) and (2, 7) is parallel to the line through (– 1, 4) and (0, 6) ?


Answer:

We have given coordinates of two lines (3, y) and (2, 7), (– 1, 4) and (0, 6)

To Find: Value of y?


The concept used: Slopes of the parallel line are always equal.


The formula used: The slope of line =


Now, The slope of the line whose coordinates are (3, y) and (2, 7).


M1 = …… (1)


And, Now, The slope of the line whose coordinates are (– 1, 4) and (0, 6).


M2 =


M2 = …… (2)


On equating the equation (1) and (2), we get



7 – y = 2(– 1)


– y = – 2 – 7


Y = 9


Hence, The value of y is 9.



Question 16.

What can be said regarding a line if its slope is

(i) zero

(ii) positive

(iii) negative


Answer:

(i) If the slope of the line is zero it means

M = tan θ


M = tan 0


Since, m = 0


So, The line is parallel to x - axis .


(ii) If the slope of the line is positive it means


Since θ is an acute


So, The line makes an acute angle with the positive x - axis .


(iii) If the slope of the line is positive it means


Since, θ is an obtuse


So, The line makes an obtuse angle with positive x - axis .



Question 17.

Show that the line joining (2, – 3) and (– 5, 1) is parallel to the line joining (7, – 1) and (0, 3).


Answer:

To Prove: The given line is parallel to another line.


Proof: Let Assume the coordinate A(2, – 3) and B(– 5, 1), C(7, – 1) and D(0,3).


The concept used: Slopes of the parallel lines are equal.


The formula used: The slope of the line, m =


Now, The slope of AB =


The Slope of AB =


Now, The slope of CD =


The Slope of AB =


So, The slope of AB = The slope of CD


Hence, The given Lines are parallel to each other.



Question 18.

Show that the line joining (2, – 5) and (– 2, 5) is perpendicular to the line joining (6, 3) and (1,1).


Answer:

To Prove: The Given line is perpendicular to each other.


Proof: Let Assume the coordinate A(2, – 5) and B(– 2, 5) joining the line AB, C(6,3) and D(1,1) joining the line CD.


The concept used: The product of the slopes of lines always – 1.


The formula used: The slope of the line, m =


Now, The slope of AB =


The Slope of AB =


Now, The slope of CD =


The Slope of AB =


So, AB × CD =


AB × CD = – 1


Hence, The given Lines are perpendicular to each other.



Question 19.

Without using Pythagoras theorem, show that the points A(0, 4), B(1, 2), C(3, 3) are the vertices of a right – angled triangle.


Answer:

We have given three points of a triangle.

Given: A(0, 4), B(1, 2), C(3, 3)


To Prove: Given points are the vertices of Right – angled Triangle.


Proof: We have A(0, 4), B(1, 2), C(3, 3)


The concept used: If the two lines are perpendicular to each other then it will be a right – angled triangle.


Now, Joining the points to make a line as AB, BC, and CA


The formula used: The slope of the line, m =


Now, The slope of line mAB =


The slope of mAB =


and, The slope of line BC =


The slope of mBc =


Now, mAB × mBc =


mAB × mBc = – 1


Since, AB is perpendicular to BC, it means B =


Hence, ABC is a right angle Triangle.



Question 20.

Prove that the points (– 4, – 1), (– 2, – 4), (4, 0) and (2, 3) are the vertices of a rectangle.


Answer:


To Prove: Given vertices are of the rectangle.


Explanation: We have given points A(– 4, – 1), B(– 2, – 4), C(4, 0) and D(2, 3)


The points are joining in the form of AB, BC, CD, and AD


The formula used: The slope of the line, m =


Now, The slope of Line AB, mAB


mAB =


The slope of BC, mBC =


mBC =


Now, The slope of Line CD, mCD =


mCD =


The slope of AD, mAD =


mAD =


Here, We can see that, mAB = mCD and mBC = mAD


i.e, AB CD and BC AD


And, mAB×mBC =


MCD×mAD =


So, that ABBC and CDAD


Hence, ABCD is a Rectangle.



Question 21.

If three points A(h, 0), P(a, b) and B(0, k) lie on a line, show that:



Answer:

If these three points lie on a line, the slope will be equal.


So, slope of A(h, 0) and P(a, b) = Slope of A(h, 0) and B(0, k)


Slope of AP


Slope of AB


Now,




bh = - ka + kh


ak + bh = kh


Dividing both sides by kh, we get,




Question 22.

The slope of a line is double of the slope of another line. If tangents of the angle between them is , find the slopes of the other line.


Answer:

Given, The tangent of the angle between them is

To Find Slope of the other line.


Assumption: The slope of line m1 = x, and m2 = 2x


Formula used:


Explanation: We have tan given, then



Case 1:



2x2 + 1 = 3x – 6x


2x2 + 3x + 1 = 0


2x2 + 2x + x + 1 = 0


2x(x + 1) + 1(x + 1) = 0


(2x + 1)(x + 1) = 0


x = – 1,


Case 2:



2x2 + 1 = 3x


2x2 – 3x + 1 = 0


2x2 – 2x – x + 1 = 0


2x(x – 1) – 1(x – 1) = 0


(2x – 1)(x – 1) = 0


x = 1,


Hence, The slope of other line is either 1, or – 1, .



Question 23.

Consider the following population and year graph:



Find the slope of the line AB and using it, find what will be the population in the year 2010.


Answer:

For the given graph,



Slope of line AB


Now, Slope of AB = Slope of AC


Therefore,


Slope of AC


5p – 460 = 25


5p = 485


P = 97 Crores.



Question 24.

Without using the distance formula, show that points (– 2, – 1), (4, 0), (3, 3) and (– 3, 2) are the vertices of a parallelogram.


Answer:


To Prove: Given points are of Parallelogram.


Explanation: Let us Assume that we have points, A (– 2, – 1), B(4, 0), C(3, 3) and D(– 3, 2), are joining the sides as AB, BC, CD, and AD.


The formula used: The slope of the line, m =


Now, The slope of Line AB, mAB =


mAB =


The slope of BC, mBC =


mBC =


Now, The slope of Line CD, mCD =


mCD =


The slope of AD, mAD =


mAD =


Here, We can see that, mAB = mCDand mBC= mAD


i.e, AB CD and BC AD


We know, If opposite side of a quadrilateral are parallel that it is parallelogram.


Hence, ABCD is a Parallelogram.



Question 25.

Find the angle between the X - axis and the line joining the points (3, – 1) and (4, – 2).


Answer:

Given, (3, – 1) and (4, – 2)

To find: Find the angle between x - axis and the line.


Explanation: We have two points A(3, – 1) and B(4, – 2).


The formula used: The slope of the line, m =


Now, The slope of line AB, mAB =


mAB = – 1


and, we know, The slope of x - axis is always 0


Now, the angle between x - axis and slope of line AB is,





θ = tan – 1 – 1


θ = 135o


Hence, The angle between the x - axis and the line is 1350.



Question 26.

The line through the points (– 2, 6) and 94, 8) is perpendicular to the line through the points (8, 12) and (x, 24). Find the value of x.


Answer:

To Find: Find the value of x ?


The concept used: If two line is perpendicular then, the product of their slopes is – 1.


Explanation: We have two lines having point A(– 2,6) and B(4,8) and other line having points C(8,12) and D(x,24).


The formula used: The slope of the line, m =


Now, The slope of Line AB is, mAB =


mAB =


and, The slope of Line CD is, mCD =


mCD =


We know the product of the slopes of perpendicular line is always – 1. Then,


mAB × mCD = – 1


= – 1



x – 8 = – 4


x = – 4 + 8


x = 4


Hence, The value of x is 4.



Question 27.

Find the value of x for which the points (x, – 1), (2, 1) and (4, 5) are collinear.


Answer:

The given points (x, – 1), (2, 1) and (4, 5) are collinear.

To Find: The value of x.


Concept Used: It is given that points are collinear, SO the area of the triangle formed by the points must be zero.


Formula used: The area of triangle = x1(y1 – y3) + x2(y3 – y1) + x3(y1 – y2)


Explanation: Let be points of triangle A(x, – 1), B(2, 1) and C(4, 5)


Now, The points are collinear than, Area of a triangle is zero.


Here, Put the given values in formula and we get,


x(1 – 5) + (2)(5 – (– 1)) + 4(– 1 – 1) = 0


x – 5x + 12 – 8 = 0


– 4x + 4 = 0


4x = 4


x = 1


Hence, The value of x is 1.



Question 28.

Find the angle between X - axis and the line joining the points (3, – 1) and (4, – 2).


Answer:

Given, (3, – 1) and (4, – 2)

To find: Find the angle between x - axis and the line.


Explanation: We have two points A(3, – 1) and B(4, – 2).


The formula used: The slope of the line, m =


Now, The slope of line AB, mAB =


mAB = – 1


and, we know, The slope of x - axis is always 0


Now, the angle between x - axis and slope of line AB is,





θ = tan – 1 – 1


θ = 135o


Hence, The angle between the x – axis and the line is 1350.



Question 29.

By using the concept of slope, show that the points (– 2, – 1), (4, 0), (3, 3) and (– 3, 2) vertices of a parallelogram.


Answer:

To Prove: Given points are of Parallelogram.


Explanation: Let us Assume that we have points, A (– 2, – 1), B(4, 0), C(3, 3) and D(– 3, 2), are joining the sides as AB, BC, CD, and AD.


The formula used: The slope of the line, m =


Now, The slope of Line AB, mAB =


mAB =


The slope of BC, mBC =


mBC =


Now, The slope of Line CD, mCD =


mCD =


The slope of AD, mAD =


mAD =


Here, We can see that, mAB = mCDand mBC= mAD


i.e, AB CD and BC AD


We know, If opposite side of a quadrilateral are parallel that it is a parallelogram.


Hence, ABCD is a Parallelogram.



Question 30.

A quadrilateral has vertices (4, 1), (1, 7), (– 6, 0) and (– 1, – 9). Show that the mid – points of the sides of this quadrilateral form a parallelogram.


Answer:

Given, A quadrilateral has vertices (4, 1), (1, 7), (– 6, 0) and (– 1, – 9).


To Prove: Mid – Points of the quadrilateral form a parallelogram.


The formula used: Mid point formula =


Explanation: Let ABCD is a quadrilateral


E is the midpoint of AB


F is the midpoint of BC


G is the midpoint of CD


H is the midpoint of AD


Now, Find the Coordinates of E, F,G and H using midpoint Formula


Coordinate of E =


Coordinate of F =


Coordinate of G =


Coordinate of H =


Now, EFGH is a parallelogram if the diagonals EG and FH have the same mid – point


Coordinate of mid – point of EG =


Coordinate of mid – point of FH =


Since Diagonals are equals then EFGH is a parallelogram.


Hence, EFGH is a parallelogram.




Exercise 23.10
Question 1.

Find the point of intersection of the following pairs of lines:

2x – y + 3 = 0 and x + y – 5 = 0


Answer:

Given:


The equations of the lines are as follows:


2x − y + 3 = 0 … (1)


x + y − 5 = 0 … (2)


Concept Used:


Point of intersection of two lines.


To find:


Point of intersection of pair of lines.


Explanation:


Solving (1) and (2) using cross - multiplication method:




⇒ x = 2/3 and y = 13/3


Hence, the point of intersection is



Question 2.

Find the point of intersection of the following pairs of lines:

bx + ay = ab and ax + by = ab


Answer:

Given:


The equations of the lines are as follows:


bx + ay = ab


To find:


Point of intersection of pair of lines.


Concept Used:


Point of intersection of two lines.


Explanation:


⇒ bx + ay − ab = 0 … (1)


ax + by = ab ⇒ ax + by − ab = 0 … (2)


Solving (1) and (2) using cross - multiplication method:




and


Hence, the point of intersection is



Question 3.

Find the point of intersection of the following pairs of lines:



Answer:

Given:


The equations of the lines are


and


To find:


Point of intersection of pair of lines.


Concept Used:


Point of intersection of two lines.


Explanation:


Thus, we have:


………(1)


……….(2)


Solving (1) and (2) using cross - multiplication method:



,


and


Hence, the point of intersection is or



Question 4.

Find the coordinates of the vertices of a triangle, the equations of whose sides are :

x + y – 4 = 0, 2x – y + 3 0 and x – 3y + 2 = 0


Answer:

Given:


x + y − 4 = 0, 2x − y + 3 = 0 and x − 3y + 2 = 0


To find:


Point of intersection of pair of lines.


Concept Used:


Point of intersection of two lines.


Explanation:


x + y − 4 = 0 … (1)


2x − y + 3 = 0 … (2)


x − 3y + 2 = 0 … (3)


Solving (1) and (2) using cross - multiplication method:



⇒ x = 1/3, y = 11/3


Solving (1) and (3) using cross - multiplication method:



⇒ x = 5/2, y = 3/2


Similarly, solving (2) and (3) using cross - multiplication method:



⇒ x = - 7/5, y = 1/5


Hence, the coordinates of the vertices of the triangle are



Question 5.

Find the coordinates of the vertices of a triangle, the equations of whose sides are :

y(t1 + t2) = 2x + 2at1t2. y(t2 + t3) = 2x + 2at2t3 and, y(t3 + t1) = 2x + 2at1t3.


Answer:

Given:


y (t1 + t2) = 2x + 2a t1t2, y (t2 + t3) = 2x + 2a t2t3 and y (t3 + t1) = 2x + 2a t1t3


To find:


Point of intersection of pair of lines.


Concept Used:


Point of intersection of two lines.


Explanation:


2x − y (t1 + t2) + 2a t1t2 = 0 … (1)


2x − y (t2 + t3) + 2a t2t3 = 0 … (2)


2x − y (t3 + t1) + 2a t1t3 = 0 … (3)


Solving (1) and (2) using cross - multiplication method:





Solving (1) and (3) using cross - multiplication method:





Solving (2) and (3) using cross - multiplication method:





Hence, the coordinates of the vertices of the triangle are



Question 6.

Find the area of the triangle formed by the lines

y = m1x + c1, y = m2x + c2 and x = 0


Answer:

Given:


y = m1x + c1 … (1)


y = m2x + c2 … (2)


x = 0 … (3)


Explanation:


In triangle ABC, let equations (1), (2) and (3) represent the sides AB, BC and CA, respectively. Solving (1) and (2):



Thus, AB and BC intersect at B


Solving (1) and (3):


x = 0, y = c1


Thus, AB and CA intersect at A 0,c1.


Similarly, solving (2) and (3):


x = 0, y = c2


Thus, BC and CA intersect at C 0,c2.


∴ Area of triangle ABC =




Question 7.

Find the area of the triangle formed by the lines

y = 0, x = 2 and x + 2y = 3


Answer:

Given:


y = 0 … (1)


x = 2 … (2)


x + 2y = 3 … (3)


Assuming:


In triangle ABC, let equations (1), (2) and (3) represent the sides AB, BC and CA, respectively.


Concept Used:


Point of intersection of two lines.


Explanation:


Solving (1) and (2):


x = 2, y = 0


Thus, AB and BC intersect at B (2, 0).


Solving (1) and (3):


x = 3, y = 0


Thus, AB and CA intersect at A (3, 0).


Similarly, solving (2) and (3):


x = 2, y = 12


Thus, BC and CA intersect at C(2, 12).


∴ Area of triangle ABC =


Hence, area of triangle ABC is .



Question 8.

Find the area of the triangle formed by the lines

x + y – 6 = 0, x – 3y – 2 = 0 and 5x – 3y + 2 = 0


Answer:

Given:


x + y − 6 = 0 … (1)


x − 3y − 2 = 0 … (2)


5x − 3y + 2 = 0 … (3)


Assuming:


In triangle ABC, let equations (1), (2) and (3) represent the sides AB, BC and CA, respectively.


Concept Used:


Point of intersection of two lines.


Explanation:


Solving (1) and (2):


x = 5, y = 1


Thus, AB and BC intersect at B (5, 1).


Solving (1) and (3):


x = 2, y = 4


Thus, AB and CA intersect at A (2, 4).


Similarly, solving (2) and (3):


x = −1, y = −1


Thus, BC and CA intersect at C (−1, −1).


∴ Area of triangle ABC =


Hence, area of triangle ABC is .



Question 9.

Find the equations of the medians of a triangle, the equations of whose sides are :

3x + 2y + 6 = 0, 2x – 5y + 4 = 0 and x – 3y – 6 = 0


Answer:

Given: equations are as follows:


3x + 2y + 6 = 0 … (1)


2x − 5y + 4 = 0 … (2)


x − 3y − 6 = 0 … (3)


Assuming:


In triangle ABC, let equations (1), (2) and (3) represent the sides AB, BC and CA, respectively.


Concept Used:


Point of intersection of two lines.


Explanation:


Solving (1) and (2):


x = −2, y = 0


Thus, AB and BC intersect at B (−2, 0).


Solving (1) and (3):


x = - 6/11, y = - 24/11


Thus, AB and CA intersect at


Similarly, solving (2) and (3):


x = −42, y = −16


Thus, BC and CA intersect at C (−42, −16).


Let D, E and F be the midpoints the sides BC, CA and AB, respectively. Then,


Then, we have:





Now, the equation of the median AD is



⇒ 16x – 59y – 120 = 0


The equation of the median BE is



⇒ 25x – 53y + 50 = 0


And, the equation of median CF is



⇒ 41 x – 112 y – 70 = 0



Question 10.

Prove that the lines , y = 4 and form an equilateral triangle.


Answer:

Given: equations are as follows:


y = √3 x + 1……(1)


y = 4 …….(2)


y = - √3 x + 2…….(3)


Assuming:


In triangle ABC, let equations (1), (2) and (3) represent the sides AB, BC and CA, respectively.


To prove:


Lines y = √3 x + 1, y = 4 and y = - √3 x + 2 form an equilateral triangle.


Explanation:


Solving (1) and (2):


x = √3 , y = 4


Thus, AB and BC intersect at B(√3,4)


Solving (1) and (3):



Thus, AB and CA intersect at A(,)


Similarly, solving (2) and (3):


, y = 4


Thus, BC and AC intersect at C(,4)


Now, we have:





Hence Proved, the given lines form an equilateral triangle



Question 11.

Classify the following pairs of lines as coincident, parallel or intersecting:

(i) 2x + y – 1 = 0 and 3x + 2y + 5 = 0

(ii) x – y = 0 and 3x – 3y + 5 = 0

(iii) 3x + 2y – 4 = 0 and 6x + 4y – 8 = 0


Answer:

Let a1x + b1y + c1 = 0 and a2x + b2y + c2 = 0 be the two lines.


(a) The lines intersect if is true.


(b) The lines are parallel if is true.


(c) The lines are coincident if is true.


(i) Given: 2x + y − 1 = 0 and 3x + 2y + 5 = 0


Explanation:


Here,


Therefore, the lines 2x + y − 1 = 0 and 3x + 2y + 5 = 0 intersect.


(ii) Given: x − y = 0 and 3x − 3y + 5 = 0


Explanation:


Here,


Therefore, the lines x − y = 0 and 3x − 3y + 5 = 0 are parallel.


(iii) Given: 3x + 2y − 4 = 0 and 6x + 4y − 8 = 0


Explanation:


Here,


Therefore, the lines 3x + 2y − 4 = 0 and 6x + 4y − 8 = 0 are coincident.



Question 12.

Find the equation of the line joining the point (3, 5) to the point of intersection of the lines 4x + y – 1 = 0 and 7x – 3y – 35 = 0.


Answer:

Given:


4x + y − 1 = 0 … (1)


7x − 3y − 35 = 0 … (2)


Concept Used:


Point of intersection of two lines.


Explanation:


Solving (1) and (2) using cross - multiplication method:



⇒ x = 2 ,y = - 7


Thus, the point of intersection of the given lines is (2, - 7).


So, the equation of the line joining the points (3, 5) and (2, - 7) is



⇒ y - 5 = 12x - 36


⇒ 12x – y – 31 = 0


Hence, the required equation of line is 12x – y – 31 = 0



Question 13.

Find the equation of the line passing through the point of intersection of the lines 4x – 7y – 3 = 0 and 2x – 3y + 1 = 0 that has equal intercepts on the axes.


Answer:

Given:

4x − 7y − 3 = 0 … (1)


2x − 3y + 1 = 0 … (2)


To find:


Equation of line passing through the point of intersection of lines.


Concept Used:


Point of intersection of two lines.


Explanation:


Solving (1) and (2) using cross - multiplication method:



⇒ x = - 8 , y = - 5


Thus, the point of intersection of the given lines is ( - 8, - 5).


Now, the equation of a line having equal intercept as a is


This line passes through ( - 8, - 5)



⇒ - 8 - 5 = a


⇒ a = - 13


Hence, the equation of the required line is or x + y + 13 = 0



Question 14.

Show that the area of the triangle formed by the lines y = m1x, y = m2x and y = c is equal to , where m1, m2 are the roots of the equation .


Answer:

Given: lines are as follows:


y = m1 x … (1)


y = m2 x … (2)


y = c … (3)


To prove:


The area of the triangle formed by the lines y = m1x, y = m2x and y = c is equal to .


Concept Used:


Point of intersection of two lines.


Explanation:


Solving (1) and (2), we get (0, 0) as their point of intersection.


Solving (1) and (3), we get as their point of intersection.


Similarly, solving (2) and (3), we get as their point of intersection.


∴ Area of the triangle formed by these lines =


It is given that m1 and m2 are the roots of the







∴ AREA =



Hence Proved.



Question 15.

If the straight line passes through the point of intersection of the lines x + y = 3 and 2x – 3y = 1 and is parallel to x – y – 6 = 0, find a and b.


Answer:

Given: lines are x + y = 3 and 2x − 3y = 1.


To find:


a and b.


Concept Used:


Point of intersection of two lines.


Explanation:


x + y − 3 = 0 … (1)


2x − 3y − 1 = 0 … (2)


Solving (1) and (2) using cross - multiplication method:



⇒ x = 2 , y = 1


Thus, the point of intersection of the given lines is (2, 1).


It is given that the line passes through (2, 1).


……(3)


It is also given that the line is parallel to the line x − y − 6 = 0.


Hence, Slope of is equal to the slope of x − y − 6 = 0 or, y = x – 6


∴ - b/a = 1


⇒ b = - a … (4)


From (3) and (4):



⇒ a = 1


From (4):


b = −1


∴ a = 1,


b = −1


Hence, a = 1, b = - 1



Question 16.

Find the orthocenter of the triangle the equations of whose sides are x + y = 1, 2x + 3y = 6 and 4x – y + 4 = 0.


Answer:

Given: Sides of triangle are are x + y = 1, 2x + 3y = 6 and 4x – y + 4 = 0.


Assuming: AB, BC and AC be the sides of triangle whose equation is are x + y = 1, 2x + 3y = 6 and 4x – y + 4 = 0.


To find:


Orthocenter of triangle.


Concept Used:


Point of intersection of two lines.


Explanation:



x + y – 1 = 0 …… (i)


2x + 3y – 6 = 0 …… (ii)


4x – y + 4 = 0. …… (iii)


By solving equation (i) and (ii) By cross multiplication



⇒ x = - 3 ,y = 4


B( - 3, 4)


By Solving equation (i) and (iii) By cross multiplication



⇒ x , y


A


Equation of BC is 2x + 3y = 6


Altitude AD is perpendicular to BC,


Therefore, equation of AD is x + y + k = 0


AD is passing through A



⇒ k = - 1


Equation of AD is x + y – 1 = 0 …… (iv)


Altitude BE is perpendicular to AC.


⇒ Let the equation of DE be x – 2y = k


BE is passing through D( - 3, 4)


⇒ - 3 – 8 = k


⇒ k = - 11


Equation of BE is x – 2y = - 11 …… (v)


By solving equation (iv) and (v),


We get, x = - 3 and y = 4


Hence, the orthocenter of triangle is ( - 3, 4).



Question 17.

Three sides AB, BC and CA of a triangle ABC are 5x – 3y + 2 = 0, x – 3y – 2 = 0 and x + y – 6 = 0 respectively. Find the equation of the altitude through the vertex A.


Answer:

Given:


The sides AB, BC and CA of a triangle ABC are as follows:


5x − 3y + 2 = 0 … (1)


x − 3y − 2 = 0 … (2)


x + y − 6 = 0 … (3)


To find:


The equation of the Altitude through the vertex A.


Concept Used:


Point of intersection of two lines.


Explanation:


Solving (1) and (3):


x = 2 , y = 4



Thus, AB and CA intersect at A (2, 4).


Let AD be the altitude.


AD⊥BC


∴ Slope of AD × Slope of BC = −1


Here, slope of BC = slope of the line (2) = 1/3


∴ Slope of AD×1/3 = - 1 ⇒ Slope of AD = - 3


Hence, the equation of the altitude AD passing through A (2, 4) and having slope −3 is y - 4 = - 3x - 2⇒ 3x + y = 10



Question 18.

Find the coordinates of the orthocenter of the triangle whose vertices are ( - 1, 3), (2, - 1) and (0, 0).


Answer:

Given: coordinates of the orthocenter of the triangle whose vertices are ( - 1, 3), (2, - 1) and (0, 0).


Assuming:


A (0, 0), B (−1, 3) and C (2, −1) be the vertices of the triangle ABC.


Let AD and BE be the altitudes.


To find:


Orthocenter of the triangle.


Explanation:



AD⊥BC and BE⊥AC


∴ The slope of AD × Slope of BC = −1


The slope of BE × Slope of AC = −1


Here, the slope of BC =


and slope of AC =


∴ slope of AD × ( - 4/3) = - 1 and slope of BE × ( - 1/2) = - 1


⇒ slope of AD and slope of BE = 2


The equation of the altitude AD passing through A (0, 0) and having slope is


y - 0 ( x - 0)


⇒ y x …..(1)


The equation of the altitude BE passing through B (−1, 3) and having slope 2 is


y - 3 = 2(x + 1)


⇒ 2x – y + 5 = 0 …….(2)


Solving (1) and (2):


x = − 4, y = − 3


Hence, the coordinates of the orthocentre is (−4, −3).



Question 19.

Find the coordinates of the incentre and centroid of the triangle whose sides have the equations 3x – 4y = 0, 12y + 5x = 0 and y – 15 = 0.


Answer:

Given: lines are as follows:


3x − 4y = 0 … (1)


12y + 5x = 0 … (2)


y − 15 = 0 … (3)


Assuming:


In triangle ABC, let equations (1), (2) and (3) represent the sides AB, BC and CA, respectively.


Concept Used:


Point of intersection of two lines.


Explanation:



Solving (1) and (2):


x = 0, y = 0


Thus, AB and BC intersect at B (0, 0).


Solving (1) and (3):


x = 20 , y = 15


Thus, AB and CA intersect at A (20, 15).


Solving (2) and (3): x = −36 , y = 15


Thus, BC and CA intersect at C (−36, 15).


Let us find the lengths of sides AB, BC and CA.





Here, a = BC = 39, b = CA = 56 and c = AB = 25


Also, x1, y1 = A (20, 15), x2, y2 = B (0, 0) and x3, y3 = C (−36, 15)




AND incentre





Hence, coordinate of incenter and centroid are ( - 1, 8)



Question 20.

Prove that the lines , , and form a rhombus.


Answer:

Given: lines are as follows:


To prove:


lines form a rhombus.


Assuming:


In quadrilateral ABCD, let equations (1), (2), (3) and (4) represent the sides AB, BC, CD and


DA, respectively.


Explanation:


Lines (1) and (3) are parallel and lines (2) and (4) are parallel.


Solving (1) and (2):


x = 0, y = 0.


Thus, AB and BC intersect at B (0, 0).


Solving (1) and (4):


x , y


Thus, AB and DA intersect A


Solving (3) and (2):


x , y =


Thus, BC and CD intersect at C


Solving (3) and (4):


x , y


Thus, DA and CD intersect at D


Let us find the lengths of sides AB, BC and CD and DA.


AB


BC


CB


DA = 1


Hence Proved, the given lines form a rhombus.



Question 21.

Find the equation of the line passing through the intersection of the lines 2x + y = 5 and x + 3y + 8 = 0 and parallel to the line 3x + 4y = 7.


Answer:

Given: Line passing through point of intersection of lines 2x + y = 5 and x + 3y + 8 = 0 and parallel to the line 3x + 4y = 7.

To find:


The equation of the required line.


Concept Used:


Point of intersection of two lines.


Explanation:


2x + y – 5 = 0 …… (i)


x + 3y + 8 = 0 …… (ii)


By solving equation (i) and (ii) ,By cross multiplication,



⇒ x , y


Point of intersection


Equation of line parallel to 3x + 4y – 7 = 0 is


3x + 4y + k = 0 …… (iii)


Equation (iii) passing through point of intersection of line.



⇒ k = 3


Hence, required equation of line is 3x + 4y + 3 = 0.



Question 22.

Find the equation of the straight line passing through the point of intersection of the lines 5x – 6y – 1 = 0 and 3x + 2y + 5 = 0 and perpendicular to the line 3x – 5y + 11 = 0.


Answer:

Given: the point of intersection of the lines 5x – 6y – 1 = 0 and 3x + 2y + 5 = 0 and perpendicular to the line 3x – 5y + 11 = 0.

To find:


The equation of the required line.


Concept Used:


Point of intersection of two lines.


Explanation:


5x – 6y – 1 = 0 …… (i)


3x + 2y + 5 = 0 …… (ii)


By solving equation (i) and (ii) ,By cross multiplication,



⇒ x , y


Point of intersection ( - 1, - 1)


Now, the slope of the line 3x – 5y + 11 = 0 or yis


Now, we know that rhe product of the slope of two perpendicular lines is - 1.


Assuming: the slope of required line is m




Now, the equation of the required line passing through ( - 1, - 1) and having slope is given by,


Y + 1


⇒ 3y + 3 = - 5x – 5


⇒ 5x + 3y + 8 = 0


Hence, equation of required line is 5x + 3y + 8 = 0.




Exercise 23.11
Question 1.

Prove that the following sets of three lines are concurrent:

15x – 18y + 1 = 0, 12x + 10y – 3 = 0 and 6x + 66y – 11 = 0


Answer:

Given:


15x – 18y + 1 = 0 …… (i)


12x + 10y – 3 = 0 …… (ii)


6x + 66y – 11 = 0 …… (iii)


To prove:


Sets of given three lines are concurrent.


Explanation:


Now, consider the following determinant:


= 15(- 110 + 198) + 18(-132 + 18) + 1(792 – 60)


⇒ 1320 – 2052 + 732 = 0


Hence proved, the given lines are concurrent.



Question 2.

Prove that the following sets of three lines are concurrent:

3x – 5y – 11 = 0, 5x + 3y – 7 = 0 and x + 2y = 0


Answer:

Given:
3x − 5y − 11 = 0 …… (i)


5x + 3y − 7 = 0 …… (ii)


x + 2y = 0 …… (iii)


To prove:


Sets of given three lines are concurrent.


Explanation:


Now, consider the following determinant:


= 3 × 14 + 5 × 7 – 11 × 7 = 0


Hence, the given lines are concurrent.



Question 3.

Prove that the following sets of three lines are concurrent:

and y = x.


Answer:

Given:
bx + ay – ab = 0 … (1)


ax + by – ab = 0 … (2)


x − y = 0 … (3)


To prove:


Sets of given three lines are concurrent.


Explanation:


Now, consider the following determinant:


-b × ab - a × ab – ab × (-a – b) = 0


Hence proved, the given lines are concurrent.



Question 4.

For what value of λ are the three lines 2x – 5y + 3 = 0, 5x – 9y + λ = 0 and x – 2y + 1 = 0 concurrent?


Answer:

Given:
2x − 5y + 3 = 0 … (1)


5x − 9y + λ = 0 … (2)


x − 2y + 1 = 0 … (3)


To find:


Value of λ.


Concept Used:


Determinant of equation is zero.


Explanation:


It is given that the three lines are concurrent.



⇒ 2(-9 + 2λ) + 5(5 – λ) + 3(-10 + 9) = 0


⇒ -18 + 4λ + 25 – 5λ – 3 = 0


⇒ λ = 4


Hence, λ = 4.



Question 5.

Find the conditions that the straight lines y = m1x + c1, y = m2x + c2 and y = m3x + c3 may meet in a point.


Answer:

Given:


The given lines can be written as follows:


m1x – y + c1 = 0 … (1)


m2x – y + c2 = 0 … (2)


m3x – y + c3 = 0 … (3)


To find:


Conditions that the straight lines y = m1x + c1, y = m2x + c2 and y = m3x + c3 may meet in a point.


Concept Used:


Determinant of equation is zero.


Explanation:


It is given that the three lines are concurrent.



⇒ m1(-c3 + c2) + 1(m2c3-m3c2) + c1(-m2 + m3) = 0


⇒ m1(c2-c3) + m2(c3-c1) + m3(c1-c2) = 0


Hence, the required condition is m1(c2-c3) + m2(c3-c1) + m3(c1-c2) = 0



Question 6.

If the lines p1x + q1y = 1, p2x + q2y = 1 and p3x + q3y = 1 be concurrent, show that the points (p1, q1), (p2, q2) and (p3, q3) are collinear.


Answer:

Given:


p1x + q1y = 1


p2x + q2y = 1


p3x + q3y = 1


To prove:


The points (p1, q1), (p2, q2) and (p3, q3) are collinear.


Concept Used:


If three lines are concurrent then determinant of equation is zero.


Explanation:


The given lines can be written as follows:


p1 x + q1 y – 1 = 0 … (1)


p2 x + q2 y – 1 = 0 … (2)


p3 x + q3 y – 1 = 0 … (3)


It is given that the three lines are concurrent.





Hence proved, This is the condition for the collinearity of the three points, (p1, q1), (p2, q2) and (p3, q3).



Question 7.

Show that the straight lines L1 = (b + c)x + ay + 1 = 0, L2 = (c + a)x + by + 1 = 0 and L3 = (a + b)x + cy + 1 = 0 are concurrent.


Answer:

Given:


L1 = (b + c)x + ay + 1 = 0


L2 = (c + a)x + by + 1 = 0


L3 = (a + b)x + cy + 1 = 0


To prove:


The straight lines L1 = (b + c)x + ay + 1 = 0, L2 = (c + a)x + by + 1 = 0 and L3 = (a + b)x + cy + 1 = 0 are concurrent.


Concept Used:


If three lines are concurrent then determinant of equation is zero.


Explanation:


The given lines can be written as follows:


(b + c) x + ay + 1 = 0 … (1)


(c + a) x + by + 1 = 0 … (2)


(a + b) x + cy + 1 = 0 … (3)


Consider the following determinant.



Applying the transformation C1→C1 + C2,




⇒ ⇒


Hence proved, the given lines are concurrent.



Question 8.

If the three lines ax + a2y + 1 = 0, bx + b2y + 1 = 0 and cx + c2y + 1 = 0 are concurrent, show that at least two of three constant a, b, c are equal.


Answer:

Given:


ax + a2y + 1 = 0


bx + b2y + 1 = 0


cx + c2y + 1 = 0


To prove:


At least two of three constant a, b, c are equal.


Concept Used:


If three lines are concurrent then determinant of equation is zero.


Explanation:


The given lines can be written as follows:


ax + a2y + 1 = 0 … (1)


bx + b2y + 1 = 0 … (2)


cx + c2y + 1 = 0 … (3)


The given lines are concurrent.



Applying the transformation R1→R1-R2 and R2→R2-R3:




⇒ (a – b)(b – c)(c – a) = 0


⇒ a – b = 0 or b – c = 0 or c – a = 0


⇒ a = b or b = c or c = a


Hence proved, atleast two of the constants a,b,c are equal .



Question 9.

If a, b, c are in A. P., prove that the straight lines ax + 2y + 1 = 0, bx + 3y + 1 = 0 and cx + 4y + 1 = 0 are concurrent.


Answer:

Given:


ax + 2y + 1 = 0


bx + 3y + 1 = 0


cx + 4y + 1 = 0


To prove:


The straight lines ax + 2y + 1 = 0, bx + 3y + 1 = 0 and cx + 4y + 1 = 0 are concurrent.


Concept Used:


If three lines are concurrent then determinant of equation is zero.


Explanation:


The given lines can be written as follows:


ax + 2y + 1 = 0 … (1)


bx + 3y + 1 = 0 … (2)


cx + 4y + 1 = 0 … (3)


Consider the following determinant.


Applying the transformation R1→R1-R2 and R2→R2-R3,



(-a + b + b – c) = 2b – a – c


Given:
2b = a + c


a + c –a – c = 0


Hence proved, the given lines are concurrent, provided 2b = a + c.



Question 10.

Show that the perpendicular bisectors of the sides of a triangle are concurrent.


Answer:

To prove:


Perpendicular bisectors of the sides of a triangle are concurrent.


Assuming:


ABC be a triangle with vertices A (x1, y1), B (x2, y2) and C (x3, y3).


Let D, E and F be the midpoints of the sides BC, CA and AB, respectively.


Explanation:


Thus, the coordinates of D, E and F are


Let mD, mE and mF be the slopes of AD, BE and CF respectively.


∴ Slope of BC × mD = -1



⇒ mD


Thus, the equation of AD





Similarly, the respective equations of BE and CF are




Let L1, L2 and L3 represent the lines (1), (2) and (3), respectively.
Adding all the three lines,


We observe:
1 ⋅ L1 + 1⋅L2 + 1⋅L3 = 0


Hence proved, the perpendicular bisectors of the sides of a triangle are concurrent.




Exercise 23.12
Question 1.

Find the equation of a line passing through the point (2, 3) and parallel to the line 3x– 4y + 5 = 0.


Answer:

Given: equation is parallel to 3x − 4y + 5 = 0 and pass through (2, 3)


To find:


Equation of required line.


Explanation:


The equation of the line parallel to 3x − 4y + 5 = 0 is


3x – 4y + λ = 0,


Where, λ is a constant.


It passes through (2, 3).


∴6 – 12 + λ = 0


⇒ λ = 6


Hence, the required line is 3x − 4y + 6 = 0.



Question 2.

Find the equation of a line passing through (3, -2) and perpendicular to the line x – 3y + 5 = 0.


Answer:

Given: equation is perpendicular to x – 3y + 5 = 0 and passes through (3,-2)


To find:


Equation of required line.


Explanation:


The equation of the line perpendicular to x − 3y + 5 = 0 is


3x + y + λ = 0,


Where λ is a constant.


It passes through (3, − 2).


9 – 2 + λ = 0


⇒ λ = - 7


Substituting λ = − 7 in 3x + y + λ = 0,


Hence, we get 3x + y – 7 = 0, which is the required line.



Question 3.

Find the equation of the perpendicular bisector of the line joining the points (1, 3) and (3, 1).


Answer:

Given: A (1, 3) and B (3, 1) be the points joining the perpendicular bisector


To find:


The equation of the perpendicular bisector of the line joining the points (1, 3) and (3, 1).


Explanation:


Let C be the midpoint of AB.


∴ coordinates of c


Slope of AB


∴ Slope of the perpendicular bisector of AB = 1


Thus, the equation of the perpendicular bisector of AB is


y – 2 = 1(x – 2)


⇒ x – y = 0


Or, y = x


Hence, the equation is y = x.



Question 4.

Find the equations of the altitudes of a ΔABC whose vertices are A (1, 4), B(-3, 2) and C(-5, -3).


Answer:

Given: The vertices of ∆ABC are A (1, 4), B (− 3, 2) and C (− 5, − 3).


To find:


The equations of the altitudes of a ΔABC whose vertices are A (1, 4), B (-3, 2) and C (-5, -3).


Explanation:


Diagram:



Slope of AB


Slope of BC


Slope of CA


Thus, we have:


Slope of CF = -2


Slope of AD


Slope of BE


Hence,


Equation of CF is: y + 3 = -2(x + 5)


⇒ 2x + y + 13 = 0


Equation of AD is: y – 4(x – 1)


⇒ 2x + 5y – 22 = 0


Equation of BE is: y – 2(x + 3)


⇒ 6x + 7y + 4 = 0



Question 5.

Find the equation of a line which is perpendicular to the line and which cuts off an intercept of 4 units with the negative direction of y-axis.


Answer:

Given: equation is perpendicular to equation and cuts off an intercept of 4 units with the negative direction of y-axis


To find:


The equation of a line which is perpendicular to the line and which cuts off an intercept of 4 units with the negative direction of y-axis.


Explanation:


The line perpendicular to is


It is Given that the line cuts off an intercept of 4 units with the negative direction of the y-axis.


This means that the line passes through (0,-4).




Substituting the value of λ, we get , which is the equation of the required line.



Question 6.

If the image of the point (2, 1) with respect to a line mirror is (5, 2), find the equation of the mirror.


Answer:

Given: image of (2 1) is (5,2)


To find:


The equation of the mirror.


Explanation:


Let the image of A (2, 1) be B (5, 2).


Also, let M be the midpoint of AB.


∴ Coordinates of M



Diagram:



Let CD be the mirror.


Line AB is perpendicular to the mirror CD.


∴ Slope of AB × Slope of CD = − 1


Slope of CD = -1


⇒ Slope of CD = -3


Equation of the mirror CD



⇒ 2y – 3 = -6 x + 21


⇒ 6x + 2y – 24 = 0


⇒ 3x + y -12 = 0


Hence, the equation of mirror is 3x + y -12 = 0



Question 7.

Find the equation of the straight line through the point (α, β) and perpendicular to the line lx + my + n = 0.


Answer:

Given: equation is perpendicular to lx + my + n = 0 and passing through (α, β)


To find:


The equation of the straight line through the point (α, β) and perpendicular to the line lx + my + n = 0.


Explanation:


The line perpendicular to lx + my + n = 0 is


mx – ly + λ = 0


This line passes through (α, β).


∴ mα-lβ + λ = 0 ⇒ λ = lβ-mα


Substituting the value of λ:


mx – ly + lβ-mα = 0


⇒ mx – α = ly – β


Hence, equation of the required line is mx – α = ly – β



Question 8.

Find the equation of the straight line perpendicular to 2x – 3y = 5 and cutting off an intercept 1 on the positive direction of the x-axis.


Answer:

Given: equation is perpendicular to 2x − 3y = 5 and cutting off an intercept 1 on the positive direction of the x-axis.


Explanation:


The line perpendicular to 2x − 3y = 5 is


3x + 2y + λ = 0


It is given that the line 3x + 2y + λ = 0 cuts off an intercept of 1 on the positive direction of the x axis.


This means that the line 3x + 2y + λ = 0 passes through the point (1, 0).


∴ 3 + 0 + λ = 0


⇒ λ = -3


Substituting the value of λ, we get 3x + 2y – 3 = 0,


Hence, equation of the required line.



Question 9.

Find the equation of the straight line perpendicular to 5x – 2y = 8 and which passes through the mid-point of the line segment joining (2, 3) and (4, 5).


Answer:

Given: equation is perpendicular to 5x -2y = 8 and pass through mid-point of the line segment joining (2, 3) and (4, 5).


To find:


The equation of the straight line perpendicular to 5x – 2y = 8 and which passes through the mid-point of the line segment joining (2, 3) and (4, 5).


Explanation:


The line perpendicular to 5x − 2y = 8 is 2x + 5y + λ = 0


Coordinates of the mid points of (2,3) and (4,5)


∴ 6 + 20 + λ = 0


⇒ λ = -26


Substituting the value of λ,


We get 2x + 5y-26 = 0,


Hence, the required equation of line is 2x + 5y-26 = 0.



Question 10.

Find the equation of the straight line which has y-intercept equal to 4/3 and is perpendicular to 3x – 4y + 11 = 0.


Answer:

Given: equation is perpendicular to 3x – 4y + 11 = 0 and has y-intercept equal to


To find:


The equation of the straight line which has y-intercept equal to and is perpendicular to 3x – 4y + 11 = 0.


Explanation:


The line perpendicular to 3x − 4y + 11 = 0 is 4x + 3y + λ = 0


It is given that the line 4x + 3y + λ = 0 has y – intercept equal to


This means that the line passes through


∴ 0 + 4 + λ = 0


⇒ λ = -4


Substituting the value of λ,


We get 4x + 3y – 4 = 0,


Hence, equation of the required line is 4x + 3y – 4 = 0



Question 11.

Find the equation of the right bisector of the line segment joining the points (a, b) and (a1, b1).


Answer:

Given: A (a, b) and B (a1, b1) be the given points


To find:


Equation of the right bisector of the line segment joining the points (a, b) and (a1, b1).


Explanation:


Let C be the midpoint of AB.


∴ coordinates of C


And, slope of AB


So, the slope of the right bisector of AB is


Thus, the equation of the right bisector of the line segment joining the points (a, b) and (a1, b1) is



⇒ 2 (a1-a)x + 2y(b1-b) + (a2 + b2) – (a12 + b12) = 0


Hence, equation of the required line 2 (a1 – a)x + 2y(b1- b) + (a2 + b2) – (a12 + b12) = 0



Question 12.

Find the image of the point (2, 1) with respect to the line mirror x + y – 5 = 0.


Answer:

Given: (2,1) is given point and line mirror is x + y – 5 = 0


To find:


Image of the point with respect to mirror line.


Explanation:


Let the image of A (2, 1) be B (a, b).


Let M be the midpoint of AB.


∴ Coordinates of M are


Diagram:



The point M lies on the line x + y − 5 = 0



⇒ a + b = 7 … (1)


Now, the lines x + y − 5 = 0 and AB are perpendicular.


∴ Slope of AB × Slope of CD = − 1



⇒ a – 2 = b – 1 ……(2)


Adding eq (1) and eq (2):


2a = 8


⇒ a = 4


Now, from equation (1):


4 + b = 7


⇒ b = 3


Hence, the image of the point (2, 1) with respect to the line mirror x + y − 5 = 0 is (4, 3).



Question 13.

If the image of the point (2, 1) with respect to the line mirror be (5, 2), find the equation of the mirror.


Answer:

Given: image of (2,1) is (5,2)


To find:


The equation of the mirror.


Explanation:


Let the image of A (2, 1) be B (5, 2).


Let M be the midpoint of AB.


Coordinates of M



Diagram:



Let CD be the mirror.


The line AB is perpendicular to the mirror CD.


∴ Slope of AB × Slope of CD = − 1


⇒ Slope of CD = -3


Thus, the equation of the mirror CD is



⇒ 2y – 3 = -6x + 21


⇒ 6x + 2y -24 = 0


⇒ 3x + y – 12 = 0


Hence, the equation of mirror is 3x + y – 12 = 0.



Question 14.

Find the equation to the straight line parallel to 3x – 4y + 6 = 0 and passing through the middle point of the join of points (2, 3) and (4, -1).


Answer:

Given: equation parallel to 3x – 4y + 6 = 0 and passing through the middle point of the join of points (2, 3) and (4, -1).


To find:


The equation to the straight line parallel to 3x – 4y + 6 = 0 and passing through the middle point of the join of points (2, 3) and (4, -1).


Explanation:


Let the Given points be A (2, 3) and B (4, − 1). Let M be the midpoint of AB.


∴ Coordinates of M


The equation of the line parallel to 3x − 4y + 6 = 0 is 3x – 4y + λ = 0


This line passes through M (3, 1).


∴ 9 – 4 + λ = 0


⇒ λ = -5


Substituting the value of λ in 3x – 4y + λ = 0, we get 3x – 4y – 5 = 0


Hence, the equation of the required line is 3x – 4y – 5 = 0.



Question 15.

Prove that the lines 2x – 3y + 1 = 0, x + y = 3, 2x – 3y = 2 and x + y = 4 form a parallelogram.


Answer:

Given: 2x – 3y + 1 = 0,


x + y = 3,


2x – 3y = 2


x + y = 4 are given equation


To prove:


The lines 2x – 3y + 1 = 0, x + y = 3, 2x – 3y = 2 and x + y = 4 form a parallelogram.


Explanation:


The given lines can be written as


… (1)


… (2)


… (3)


… (4)


The slope of lines (1) and (3) is and that of lines (2) and (4) is − 1.


Thus, lines (1) and (3), and (2) and (4) are two pair of parallel lines.


If both pair of opposite sides are parallel then, we can say that it is a parallelogram.


Hence proved, the given lines form a parallelogram.



Question 16.

Find the equation of a line drawn perpendicular to the line through the point where it meets the y-axis.


Answer:

Given: equation is perpendicular to and it meets the y-axis.


To find:


The equation of a line drawn perpendicular to the line through the point where it meets the y-axis.


Explanation:


Let us find the intersection of the line with y-axis.


At x = 0,



⇒ y = 6


Thus, the given line intersects y-axis at (0, 6).


The line perpendicular to the line is



This line passes through (0, 6).



⇒ λ


Now, substituting the value of λ, we get:



⇒ 2x -3y + 18 = 0


Hence, the equation of the required line is 2x -3y + 18 = 0



Question 17.

The perpendicular from the origin to the line y = mx + c meets it at the point (-1, 2). Find the values of m and c.


Answer:

Given: perpendicular from the origin and meets at the point (-1, 2)


Explanation:


The given line is y = mx + c which can be written as mx –y + c = 0 … (1)


The slope of the line perpendicular to y = mx + c is


So, the equation of the line with slope and passing through the origin is



x + my = 0 … (2)


Solving eq(1) and eq(2) by cross multiplication, we get




Thus, the point of intersection of the perpendicular from the origin to the line y = mx + c is


It is given that the perpendicular from the origin to the line y = mx + c meets it at the point (-1, 2)


and


⇒ m2 + 1 = mc and m2 + 1


⇒ mc


⇒ m


Now, substituting the value of m in m2 + 1 = mc, we get



⇒ c


Hence, m and c



Question 18.

Find the equation of the right bisector of the line segment joining the points (3, 4) and (-1, 2).


Answer:

Given: A (3, 4) and B (− 1, 2) be the given points


To find:


Equation of the right bisector of the line segment joining the points (3, 4) and (-1, 2).


Explanation:


Let C be the midpoint of AB.


∴ C


∵ Slope of AB


∴ Slope of the perpendicular bisector of AB = -2


Thus, the equation of the perpendicular bisector of AB is


y-3 = -2(x-1)


⇒ 2x + y – 5 = 0


Hence, the required line is 2x + y – 5 = 0.



Question 19.

The line through (h, 3) and (4, 1) intersects the line 7x – 9y – 19 = 0 at right angle. Find the value of h.


Answer:

Given: A (h,3) and B (4,1) be the points intersect At right angle at the line 7x – 9y – 19 = 0


To find:


The value of h.


Explanation:


The line 7x − 9y − 19 = 0 can be written as



So, the slope of this line is


It is given that the line joining the points A (h,3) and B (4,1) is perpendicular to the line 7x − 9y − 19 = 0.



⇒ 9h – 36 = -14


⇒ 9h = 22


⇒ h


Hence, the value of h is



Question 20.

Find the image of the point (3, 8) with respect to the line x + 3y = 7 assuming the line to be a plane mirror.


Answer:

Given: (3, 8) is given point and line mirror is x + 3y – 7 = 0.


To find:


Image of point with respect to mirror line.


Explanation:


Let the image of A (3,8) be B (a,b).


Also, let M be the midpoint of AB.


∴ Coordinates of M


Diagram:



Point M lies on the line x + 3y = 7



⇒ a + 3b + 13 = 0 … (1)


Lines CD and AB are perpendicular


∴ Slope of AB × Slope of CD = − 1



⇒ b – 8 = 3a – 9


⇒ 3a-b-1 = 0 … (2)


Solving (1) and (2) by cross multiplication, we get:



⇒ a = -1, b = -4


Hence, the image of the point (3, 8) with respect to the line mirror x + 3y = 7 is


(− 1, − 4).



Question 21.

Find the coordinates of the foot of the perpendicular from the point (-1, 3) to the line 3x – 4y – 16 = 0.


Answer:

Given: equation is perpendicular from the point (-1, 3) to the line 3x – 4y – 16 = 0.


To find:


The coordinates of the foot of the perpendicular from the point (-1, 3) to the line 3x – 4y – 16 = 0.


Explanation:


Let A (− 1, 3) be the given point.


Also, let M (h, k) be the foot of the perpendicular drawn from A (− 1, 3) to the line 3x − 4y − 16 = 0


Diagram:



Point M (h, k) lies on the line 3x − 4y − 16 = 0


3h − 4k − 16 = 0 … (1)


Lines 3x − 4y − 16 = 0 and AM are perpendicular.



⇒ 4h + 3k – 5 = 0 … (2)


Solving eq (1) and eq (2) by cross multiplication, we get:




Hence, the coordinates of the foot of perpendicular are



Question 22.

Find the projection of the point (1, 0) on the line joining the points (-1, 2) and (5, 4).


Answer:

Given:


The points (-1, 2) and (5, 4).


To find:


The projection of the point (1, 0) on the line joining the points (-1, 2) and (5, 4).


Explanation:


Let A (− 1, 2) be the given point whose projection is to be evaluated and C (− 1, 2) and D (5, 4) be the other two points.


Also, let M (h, k) be the foot of the perpendicular drawn from A (− 1, 2) to the line joining the points C (− 1, 2) and D (5, 4).


Diagram:



Clearly, the slope of CD and MD are equal.



⇒ h – 3k + 7 = 0…….(1)


The lines segments AM and CD are perpendicular.



⇒ 3h + k – 3 = 0 …….(2)


Solving (1) and (2) by cross multiplication, we get:




Hence, the projection of the point (1, 0) on the line joining the points (-1, 2) and (5, 4) is



Question 23.

Find the equation of a line perpendicular to the line and at a distance of 3 units from the origin.


Answer:

Given:


Line and distance of 3 units from the origin.


To find:


The equation of a line perpendicular to the line and at a distance of 3 units from the origin.


Explanation:


The line perpendicular to is


It is given that the line is at a distance of 3 units from the origin.



⇒ λ = ± 6


Substituting the value of λ,


We get ,


Hence, equation of the required line is



Question 24.

The line 2x + 3y = 12 meets the x-axis at A and y-axis at B. The line through (5, 5) perpendicular to AB meets the x-axis and the line AB at C and E respectively. If O is the origin of coordinates, find the area of figure OCEB.


Answer:

Given:


Line 2x + 3y = 12 meets the x-axis at A and y-axis at B


To find:


The area of figure OCEB.


Explanation:


The given line is 2x + 3y = 12, which can be written as


……(1)


So, the coordinates of the points A and B are (6, 0) and (0, 4), respectively.


Diagram:



The equation of the line perpendicular to line (1) is



This line passes through the point (5, 5).




Now, substituting the value of λ in , we get:



…….(2)


Thus, the coordinates of intersection of line (1) with the x-axis is C


To find the coordinates of E, let us write down equations (1) and (2) in the following manner:


2x + 3y – 12 = 0 … (3)


3x – 2y – 5 = 0 … (4)


Solving (3) and (4) by cross multiplication, we get:



⇒ x = 3, y = 2


Thus, the coordinates of E are (3, 2).


From the figure,


EC


EA


Now,


Area(OCEB) = Area(∆OAB) –Area(∆CAE)


⇒ Area(OCEB)


sq units


Hence, area of figure OCEB is sq units



Question 25.

Find the equation of the straight line which cuts off intercepts on x-axis twice that on y-axis and is at a unit distance from the origin.


Answer:

To find:


The equation of the straight line which cuts off intercepts on x-axis twice that on y-axis and is at a unit distance from the origin.


Assuming:


Intercepts on x-axis and y-axis be 2a and a, respectively.


Explanation:


So, the equation of the line with intercepts 2a on x-axis and a on y-axis be



⇒ x + 2y = 2a … (1)


Let us change equation (1) into normal form.




Thus, the length of the perpendicular from the origin to the line (1) is


Given:


P = 1




Required equation of the line:


x + 2y


⇒ x + 2y + = 0


Hence, equation of required line is x + 2y + = 0.



Question 26.

The equations of perpendicular bisectors of the sides AB and AC of a triangle ABC are x – y + 5 = 0 and x + 2y = 0 respectively. If the point A is (1, -2), find the equation of the line BC.


Answer:

Given:


Sides AB and AC of a triangle ABC are x – y + 5 = 0 and x + 2y = 0.


To find:


The equation of the line BC.


Explanation:


Diagram:



Let the perpendicular bisectors x − y + 5 = 0 and x + 2y = 0 of the sides AB and AC intersect at D and E, respectively.


Let (x1,y1) and (x2,y2) be the coordinates of points B and C.


Coordinates of D


And coordinates of E


Point D lies on the line x − y + 5 = 0



⇒ x1 – y1 + 13 = 0 … (1)


Point E lies on the line x + 2y = 0



⇒ x2 + 2y2 – 3 = 0 … (2)


Side AB is perpendicular to the line x − y + 5 = 0



⇒ x1 + y1 + 1 … (3)


Similarly, side AC is perpendicular to the line x + 2y = 0



⇒ 2x2 - y2 - 4 = 0… (4)


Now, solving eq (1) and eq (3) by cross multiplication, we get:



⇒ x1 = -7, y1 = 6


Thus, the coordinates of B are (-7, 6)


Similarly, solving (2) and (4) by cross multiplication, we get:




Thus, coordinates of C are


Therefore, equation of line BC is




⇒ 14x + 23 y – 40 = 0


Hence, the equation of line BC is 14x + 23 y – 40 = 0




Exercise 23.13
Question 1.

Find the angles between each of the following pairs of straight lines :

3x + y + 12 = 0 and x + 2y – 1 = 0


Answer:

Given:


The equations of the lines are


3x + y + 12 = 0 … (1)


x + 2y − 1 = 0 … (2)


To find:


Angles between two lines.


Explanation:


Let m1 and m2 be the slopes of these lines.


m1 = -3, m2


Let θ be the angle between the lines.
Then,




Hence, the acute angle between the lines is 45°



Question 2.

Find the angles between each of the following pairs of straight lines :

3x – y + 5 = 0 and x – 3y + 1 = 0


Answer:

Given:


The equations of the lines are


3x − y + 5 = 0 … (1)


x − 3y + 1 = 0 … (2)


To find:


Angles between two lines.


Explanation:


Let m1 and m2 be the slopes of these lines.


m1 = 3, m2


Let θ be the angle between the lines.
Then,




Hence, the acute angle between the lines is



Question 3.

Find the angles between each of the following pairs of straight lines :

3x + 4y – 7 = 0 and 4x – 3y + 5 = 0


Answer:

Given:


The equations of the lines are


3x + 4y − 7 = 0 … (1)


4x − 3y + 5 = 0 … (2)


To find:


Angles between two lines.


Explanation:


Let m1 and m2 be the slopes of these lines.


m1m2


∵m1m2


Hence, the given lines are perpendicular.
Therefore, the angle between them is 90°.



Question 4.

Find the angles between each of the following pairs of straight lines :

x – 4y = 3 and 6x – y = 11


Answer:

Given:


The equations of the lines are


x − 4y = 3 … (1)


6x − y = 11 … (2)


To find:


Angles between two lines.


Explanation:


Let m1 and m2 be the slopes of these lines.


m1m2 = 6


Let θ be the angle between the lines.
Then,




Hence, the acute angle between the lines is



Question 5.

Find the angles between each of the following pairs of straight lines :

(m2 – mn) y = (mn + n2)x + n3 and (mn + m2)y = (mn – n2)x + m3.


Answer:

Given:


The equations of the lines are


(m2 − mn) y = (mn + n2) x + n3 … (1)


(mn + m2) y = (mn − n2) x + m3 … (2)


To find:


Angles between two lines.


Explanation:


Let m1 and m2 be the slopes of these lines.



Let θ be the angle between the lines.


Then,






Then,




Hence, the acute angle between the lines is



Question 6.

Find the acute angle between the lines 2x – y + 3 = 0 and x + y + 2 = 0.


Answer:

Given:


The equations of the lines are


2x − y + 3 = 0 … (1)


x + y + 2 = 0 … (2)


To find:


Angles between two lines.


Explanation:


Let m1 and m2 be the slopes of these lines.


m1 = 2, m2 = -1


Let θ be the angle between the lines.
Then,




Hence, the acute angle between the lines is



Question 7.

Prove that the points (2, -1), (0, 2), (2, 3) and (4, 0) are the coordinates of the vertices of a parallelogram and find the angle between its diagonals.


Answer:

To prove:


The points (2, -1), (0, 2), (2, 3) and (4, 0) are the coordinates of the vertices of a parallelogram


To find:


The angle between diagonals of parallelogram.


Assuming:


A(2, − 1), B(0, 2), C(2, 3) and D(4, 0) be the vertices.


Explanation:


Slope of AB


Slope of BC


Slope of CD


Slope of DA


Thus, AB is parallel to CD and BC is parallel to DA.


Therefore, the given points are the vertices of a parallelogram.


Now, let us find the angle between the diagonals AC and BD.


Let m1 and m2 be the slopes of AC and BD, respectively.




Thus, the diagonal AC is parallel to the y-axis.


∴∠ODB


In triangle MND,


∠DMN


Hence proved, the acute angle between the diagonal is .



Question 8.

Find the angle between the line joining the points (2, 0), (0, 3) and the line x + y = 1.


Answer:

Given:


Points (2, 0), (0, 3) and the line x + y = 1.


Assuming:


Let A (2, 0), B (0, 3) be the given points.


To find:


Angle between the line joining the points (2, 0), (0, 3) and the line x + y = 1.


Explanation:


Slope of AB = m1


Slope of the line x + y = 1 is -1


∴ m2 = -1


Let θ be the angle between the line joining the points (2, 0), (0, 3) and the line x + y = 1




Hence, the acute angle between the line joining the points (2, 0), (0, 3) and the line x + y = 1 is .



Question 9.

If θ is the angle which the straight line joining the points (x1, y1) and (x2, y2) subtends at the origin, prove that and .


Answer:

To prove:


and


Assuming:


A (x1, y1) and B (x2, y2) be the given points and O be the origin.


Explanation:



Slope of OA = m1 = y1x1


Slope of OB = m2 = y2x2


It is given that θ is the angle between lines OA and OB.





Now,


As we know that





Hence proved.



Question 10.

Prove that the straight lines (a + b)x + (a – b)y = 2ab, (a – b)x + (a + b)y = 2ab and x + y = 0 form an isosceles triangle whose vertical angle is .


Answer:

Given:


The given lines are


(a + b) x + (a − b) y = 2ab … (1)


(a − b) x + (a + b) y = 2ab … (2)


x + y = 0 … (3)


To prove:


The straight lines (a + b)x + (a – b)y = 2ab, (a – b)x + (a + b)y = 2ab and x + y = 0 form an isosceles triangle whose vertical angle is


Assuming:


Let m1, m2 and m3 be the slopes of the lines (1), (2) and (3), respectively.


Explanation:


Now,


Slope of the first line = m1


Slope of the second line = m2


Slope of the third line = m3 = -1


Let θ1 be the angle between lines (1) and (2), θ2 be the angle between lines (2) and (3) and θ3 be the angle between lines (1) and (3).














Here,
θ2 = θ3 and


Hence proved, the given lines form an isosceles triangle whose vertical angle is



Question 11.

Find the angle between the lines x = a and by + c = 0.


Answer:

Given:


x = a


by + c = 0


To find:


Angle between the lines x = a and by + c = 0.


Concept Used:


Angle between two lines.


Explanation:


The given lines can be written as


x = a … (1)


y = -cb … (2)


Hence, Lines (1) and (2) are parallel to the y-axis and x-axis, respectively. Thus, they intersect at right angle, i.e. at 90°.



Question 12.

Find the tangent of the angle between the lines which have intercepts 3, 4 and 1, 8 on the axes respectively.


Answer:

Given:


Intercepts 3, 4 and 1, 8 on the axes


To find:


Tangent of the angle between the lines


Concept Used:


Angle between two lines.


Explanation:


The respective equations of the lines having intercepts 3, 4 and 1, 8 on the axes are


… (1)


… (2)


Assuming:


m1 and m2 be the slope of the lines (1) and (2), respectively.


∴m1, m2 = -8


Let θ be the angle between the lines (1) and (2).




Hence, the tangent of the angles between the lines is .



Question 13.

Show that the line a2x + ay + 1 = 0 is perpendicular to the line x – ay = 1 for all non-zero real values of a.


Answer:

Given:


Line a2x + ay + 1 = 0 is perpendicular to the line x – ay = 1


To prove:


The line a2x + ay + 1 = 0 is perpendicular to the line x – ay = 1 for all non-zero real values of a.


Concept Used:


Product of slope of perpendicular line is -1.


Explanation:


The given lines are


a2x + ay + 1 = 0 … (1)


x − ay = 1 … (2)


Let m1 and m2 be the slopes of the lines (1) and (2).


m1m2 = -


Hence proved, line a2x + ay + 1 = 0 is perpendicular to the line x − ay = 1 for all non-zero real values of a.



Question 14.

Show that the tangent of an angle between the lines and is .


Answer:

Given:


……(i)


……(ii)


To prove:


The tangent of an angle between the lines and is


Concept Used:


Angle between two lines.


Assuming:


m1 and m2 be the slope of the lines (1) and (2), respectively.


Explanation:


∴m1, m2


Let θ be the angle between the lines (1) and (2).







Hence proved, the tangent of the angles between the lines is .




Exercise 23.14
Question 1.

Find the values of α so that the point P(α 2, α) lies inside or on the triangle formed by the lines x – 5y + 6 = 0, x – 3y + 2 = 0 and x – 2y – 3 = 0.


Answer:

Given: x – 5y + 6 = 0, x – 3y + 2 = 0 and x – 2y – 3 = 0 forming a triangle and point P(α2, α) lies inside or on the triangle


To find: value of α


Explanation:


Let ABC be the triangle of sides AB, BC and CA whose equations are x − 5y + 6 = 0, x − 3y + 2 = 0 and x − 2y − 3 = 0, respectively.
On solving the equations,


We get A (9, 3), B (4, 2) and C (13, 5) as the coordinates of the vertices.


Diagram:




It is given that point P (α2, α) lies either inside or on the triangle. The three conditions are given below.


(i) A and P must lie on the same side of BC.


(ii) B and P must lie on the same side of AC.


(iii) C and P must lie on the same side of AB.


If A and P lie on the same side of BC, then


(9 – 9 + 2)(α2 – 3α + 2) ≥0


⇒ (α – 2)(α – 1) ≥ 0


⇒ α ∈ (- ∞, 1 ] ∪ [ 2, ∞) … (1)


If B and P lie on the same side of AC, then


(4 – 4 – 3) (α2 – 2α – 3) ≥ 0


⇒ (α – 3)(α + 1) ≤ 0


⇒ α ∈ [- 1, 3] … (2)


If C and P lie on the same side of AB, then


(13 – 25 + 6)(α2 – 5α + 6) ≥0


⇒ (α – 3)(α – 2) ≤ 0


⇒ α ∈ [ 2, 3] … (3)


From (1), (2) and (3), we get:


α∈ [ 2, 3]


Hence, α∈ [2, 3]



Question 2.

Find the values of the parameter a so that the point (a, 2) is an interior point of the triangle formed by the lines x + y – 4 = 0, 3x – 7y – 8 = 0 and 4x – y – 31 = 0.


Answer:

Given:


x + y – 4 = 0, 3x – 7y – 8 = 0 and 4x – y – 31 = 0 forming a triangle and point (a, 2)is an interior point of the triangle


To find:


Value of a


Explanation:


Let ABC be the triangle of sides AB, BC and CA whose equations are x + y − 4 = 0, 3x − 7y − 8 = 0 and 4x − y − 31 = 0, respectively.


On solving them, we get A (7, - 3), B and C as the coordinates of the vertices.
Let P (a, 2) be the given point.


Diagram:



It is given that point P (a, 2) lies inside the triangle. So, we have the following:


(i) A and P must lie on the same side of BC.


(ii) B and P must lie on the same side of AC.


(iii) C and P must lie on the same side of AB.


Thus, if A and P lie on the same side of BC, then


21 + 21 – 8 – 3a – 14 – 8 > 0


⇒ a > … (1)


If B and P lie on the same side of AC, then



⇒ a < … (2)


If C and P lie on the same side of AB, then




⇒ a > 2 … (3)


From (1), (2) and (3), we get:


A ∈


Hence, A ∈



Question 3.

Determine whether the point (-3, 2) lies inside or outside the triangle whose sides are given by the equations x + y – 4 = 0, 3x – 7y + 8 = 0, 4x – y – 31 = 0.


Answer:

Given:


x + y – 4 = 0, 3x – 7y + 8 = 0, 4x – y – 31 = 0 forming a triangle and point (-3, 2)


To find:


Point (-3, 2) lies inside or outside the triangle


Explanation:


Let ABC be the triangle of sides AB, BC and CA, whose equations x + y − 4 = 0, 3x − 7y + 8 = 0 and 4x − y − 31 = 0, respectively.


On solving them, we get A (7, - 3), B (2, 2) and C (9, 5) as the coordinates of the vertices.


Let P (− 3, 2) be the given point.


Diagram:



The given point P (− 3, 2) will lie inside the triangle ABC, if


(i) A and P lies on the same side of BC


(ii) B and P lies on the same side of AC


(iii) C and P lies on the same side of AB


Thus, if A and P lie on the same side of BC, then


21 + 21 + 8 – 9 – 14 + 8 > 0


⇒ 50 × - 15 > 0


⇒ - 750 > 0,


Which is false


Hence, the point (− 3, 2) lies outside triangle ABC.




Exercise 23.15
Question 1.

Find the distance of the point (4, 5) from the straight line 3x – 5y + 7 = 0.


Answer:

Given:


Line 3x – 5y + 7 = 0


Concept Used:


Distance of a point from a line.


To find:


The distance of the point (4, 5) from the straight line 3x – 5y + 7 = 0.


Explanation:


Comparing ax + by + c = 0 and 3x − 5y + 7 = 0, we get:


a = 3, b = − 5 and c = 7


So, the distance of the point (4, 5) from the straight line 3x − 5y + 7 = 0 is


d


⇒ d


Hence, the required distance is



Question 2.

Find the perpendicular distance of the line joining the points (cosθ, sinθ) and (cosϕ, sinϕ) from the origin.


Answer:

Given: points (cosθ, sinθ) and (cosϕ, sinϕ) from the origin.


To find:


The perpendicular distance of the line joining the points (cosθ, sinθ) and (cosϕ, sinϕ) from the origin


Concept Used:


Distance of a point from a line.


Explanation:


The equation of the line joining the points (cos θ, sin θ) and (cos ϕ, sin ϕ) is given below:





Let d be the perpendicular distance from the origin to the line


-sinθ2 + cosϕ-cosθ2


⇒ d


⇒ d


⇒ d


⇒ d


⇒ d


⇒ d


⇒ d


Hence, the required distance is



Question 3.

Find the length of the perpendicular from the origin to the straight line joining the two points whose coordinates are (a cos α, a sin α) and (a cos β, a sin β).


Answer:

Given:


Coordinates are (a cos α, a sin α) and (a cos β, a sin β).


To find:


The length of the perpendicular from the origin to the straight line joining the two points whose coordinates are (a cos α, a sin α) and (a cos β, a sin β).


Concept Used:


Distance of a point from a line.


Explanation:


Equation of the line passing through (acosα, asinα) and (acosβ, asinβ) is








The distance of the line from the origin is








Hence, the required distance is



Question 4.

Show that the perpendicular let fall from any point on the straight line 2x + 11y – 5 = 0 upon the two straight lines 24x + 7y = 20 and 4x – 3y – 2 = 0 are equal to each other.


Answer:

Given:


Lines 24x + 7y = 20 and 4x – 3y – 2 = 0


To prove:


The perpendicular let fall from any point on the straight line 2x + 11y – 5 = 0 upon the two straight lines 24x + 7y = 20 and 4x – 3y – 2 = 0 are equal to each other.


Concept Used:


Distance of a point from a line.


Assuming:


P(a, b) be any point on 2x + 11y − 5 = 0


Explanation:


∴ 2a + 11b − 5 = 0


⇒ b ……..(i)


Let d1 and d2 be the perpendicular distances from point P
on the lines 24x + 7y = 20 and 4x − 3y − 2 = 0, respectively.


d1


⇒ d1


From (1)


⇒ d1


Similarly,


d2


⇒ d2


From (1)


⇒ d2


∴ d1 = d2


Hence proved.



Question 5.

Find the distance of the point of intersection of the lines 2x + 3y = 21 and 3x – 4y + 11 = 0 from the line 8x + 6y + 5 = 0.


Answer:

Given:


Lines 2x + 3y = 21 and 3x – 4y + 11 = 0


To find:


The distance of the point of intersection of the lines 2x + 3y = 21 and 3x – 4y + 11 = 0 from the line 8x + 6y + 5 = 0.


Concept Used:


Distance of a point from a line.


Explanation:


Solving the lines 2x + 3y = 21 and 3x − 4y + 11 = 0 we get:



⇒ x = 3, y = 5


So, the point of intersection of 2x + 3y = 21 and 3x − 4y + 11 = 0 is (3, 5).


Now, the perpendicular distance d of the line 8x + 6y + 5 = 0 from the point (3, 5) is
d


Hence, distance is .



Question 6.

Find the length of the perpendicular from the point (4, -7) to the line joining the origin and the point of intersection of the lines 2x – 3y + 14 = 0 and 5x + 4y – 7 = 0.


Answer:

Given:


Lines 2x – 3y + 14 = 0 and 5x + 4y – 7 = 0.


To find:


The length of the perpendicular from the point (4, -7) to the line joining the origin and the point of intersection of the lines 2x – 3y + 14 = 0 and 5x + 4y – 7 = 0.


Concept Used:


Distance of a point from a line.


Explanation:


Solving the lines 2x − 3y + 14 = 0 and 5x + 4y − 7 = 0 we get:



⇒ x, y


So, the point of intersection of 2x − 3y + 14 = 0 and 5x + 4y − 7 = 0 is


The equation of the line passing through the origin and the point is



⇒ y


⇒ y


⇒ 12x + 5y = 0


Let d be the perpendicular distance of the line 12x + 5y = 0 from the point (4, − 7)


∴d


Hence, Length of perpendicular is 1.



Question 7.

What are the points on X-axis whose perpendicular distance from the straight line is a ?


Answer:

Given:


The points on x-axis whose perpendicular distance from the straight line is a


To find:


Points on x-axis


Concept Used:


Distance of a point from a line.


Explanation:


Let (t, 0) be a point on the x-axis.


It is given that the perpendicular distance of the line from a point is a.








⇒ t


Hence, the required points on the x-axis are and



Question 8.

Show that the product of perpendicular on the line from the points is b2.


Answer:

Given:


and points


To prove:


The product of perpendicular on the line from the points is b2.


Concept Used:


Distance of a point from a line.


Explanation:


Let d1 and d2 be the perpendicular distances of line from points and respectively.


∴d1


Similarly,


d1 = -


Now,


d1d2


⇒ d1d2


⇒ d1d2


⇒ d1d2


Hence proved.



Question 9.

Find the perpendicular distance from the origin of the perpendicular from the point (1, 2) upon the straight line.


Answer:

Given:


Line


To find:


The perpendicular distance from the origin of the perpendicular from the point (1, 2) upon the straight line .


Concept Used:


Distance of a point from a line.


Explanation:


The equation of the line perpendicular to is
This line passes through (1, 2).


+ 2 + λ = 0


⇒ λ


Substituting the value of λ, we get


Let d be the perpendicular distance from the origin to the line


d


Hence, the required perpendicular distance is



Question 10.

Find the distance of the point (1, 2) from the straight line with slope 5 and passing through the point of intersection of x + 2y = 5 and x – 3y = 7.


Answer:

Given:


Lines x + 2y = 5 and x – 3y = 7, slope = 5.


To find:


The distance of the point (1, 2) from the straight line with slope 5 and passing through the point of intersection of x + 2y = 5 and x – 3y = 7.


Concept Used:


Distance of a point from a line.


Explanation:


To find the point intersection of the lines x + 2y = 5 and x − 3y = 7, let us solve them.



⇒ x y


So, the equation of the line passing through with slope 5 is



⇒ 5y + 2 = 25x – 145


⇒ 25x – 5y – 147 = 0


Let d be the perpendicular distance from the point (1, 2) to the line 25x – 5y – 147 = 0


∴d


Hence, the required perpendicular distance is



Question 11.

What are the points on y-axis whose distance from the line is 4 units?


Answer:

Given:


Distance from the line is 4.


To find:


Points on y-axis


Concept Used:


Distance of a point from a line.


Explanation:


Let (0, t) be a point on the y-axis.


It is given that the perpendicular distance of the line from the point (0, t) is 4 units.






⇒ t


⇒ t


⇒ t


Hence, the required points on the y-axis are and .



Question 12.

In the triangle ABC with vertices A(2, 3), B(4, -1) and C(1, 2) find the equation and the length of the altitude from the vertex A.


Answer:

Given:


A(2, 3), B(4, -1) and C(1, 2).


To find:


The equation and the length of the altitude from the vertex A.


Concept Used:


Distance of a point from a line.


Explanation:


Equation of side BC:



⇒ x + y – 3 = 0


The equation of the altitude that is perpendicular to x + y – 3 = 0 is x – y + λ = 0.


Line x – y + λ = 0 passes through (2, 3).


∴ 2 – 3 + λ = 0


⇒ λ = 1


Thus, the equation of the altitude from the vertex A (2, 3) is x – y + 1 = 0.


Let d be the length of the altitude from A (2, 3).


d


⇒ d


Hence, the required distance is.



Question 13.

Show that the path of a moving point such that its distances from two lines 3x – 2y = 5 and 3x + 2y = 5 are equal is a straight line.


Answer:

Given:


Two lines 3x – 2y = 5 and 3x + 2y = 5


To prove:


The path of a moving point such that its distances from two lines 3x – 2y = 5 and 3x + 2y = 5 are equal is a straight line


Concept Used:


Distance of a point from a line.


Explanation:


Let P(h, k) be the moving point such that it is equidistant from the lines 3x − 2y = 5 and 3x + 2y = 5



⇒ |3h – 2k – 5| = |3h + 2k – 5|


⇒ 3h – 2k – 5 = ±(3h + 2k – 5)


⇒ 3h – 2k – 5 = 3h + 2k – 5 and 3h – 2k – 5 = -3h + 2k – 5


⇒ k = 0 and 3h = 5


Hence proved, the path of the moving points are 3x = 5 or y = 0. These are straight lines.



Question 14.

If sum of perpendicular distances of a variable point P(x, y) from the lines x + y – 5 = 0 and 3x – 2y + 7 = 0 is always 10. Show that P must move on a line.


Answer:

Given:


Sum of perpendicular distances of a variable point P(x, y) from the lines x + y – 5 = 0 and 3x – 2y + 7 = 0 is always 10


To prove:


P must move on a line.


Concept Used:


Distance of a point from a line.


Explanation:


It is given that the sum of perpendicular distances of a variable point P (x, y) from the lines x + y − 5 = 0 and 3x − 2y + 7 = 0 is always 10





It is a straight line.


Hence proved.



Question 15.

If the length of the perpendicular from the point (1, 1) to the line ax – by + c = 0 be unity, Show that .


Answer:

Given:


Line ax – by + c = 0 and point (1, 1)


To prove:



Concept Used:


Distance of a point from a line.


Explanation:


The distance of the point (1, 1) from the straight line ax − by + c = 0 is 1


∴ 1


⇒ a2 + b2 + c2 – 2ab + 2ac – 2bc = a2 + b2


⇒ ab + bc – ac


Dividing both the sides by abc, we get:



Hence proved.




Exercise 23.16
Question 1.

Determine the distance between the following pair of parallel lines:

4x – 3y – 9 = 0 and 4x – 3y – 24 = 0


Answer:

Given: The parallel lines are


4x − 3y − 9 = 0 … (1)


4x − 3y − 24 = 0 … (2)


To find:


Distance between the givens parallel lines


Explanation:


Let d be the distance between the given lines.



Hence, distance between givens parallel line is



Question 2.

Determine the distance between the following pair of parallel lines:

8x + 15y – 34 = 0 and 8x + 15y + 31 = 0


Answer:

Given: The parallel lines are


8x + 15y − 34 = 0 … (1)


8x + 15y + 31 = 0 … (2)


To find:


Distance between the givens parallel lines


Explanation:


Let d be the distance between the given lines.



Hence, distance between givens parallel line is



Question 3.

Determine the distance between the following pair of parallel lines:

y = mx + c and y = mx + d


Answer:

Given: The parallel lines are


y = mx + c and y = mx + d


To find:


Distance between the givens parallel lines


Explanation:


The parallel lines can be written as


mx − y + c = 0 … (1)


mx − y + d = 0 … (2)


Let d be the distance between the given lines.



Hence, distance between givens parallel line is



Question 4.

Determine the distance between the following pair of parallel lines:

4x + 3y – 11 = 0 and 8x + 6y = 15


Answer:

Given: The parallel lines are


4x + 3y – 11 = 0 and 8x + 6y = 15


To find:


Distance between the givens parallel lines


Explanation:


The given parallel lines can be written as


4x + 3y − 11 = 0 … (1)


… (2)


Let d be the distance between the given lines.



Hence, distance between givens parallel line is



Question 5.

The equations of two sides of a square are 5x – 12y – 65 = 0 and 5x – 12y + 26 = 0. Find the area of the square.


Answer:

Given: Two side of square are 5x – 12y – 65 = 0 and 5x – 12y + 26 = 0


To find: area of the square


Explanation:


The sides of a square are


5x − 12y − 65 = 0 … (1)


5x − 12y + 26 = 0 … (2)


We observe that lines (1) and (2) are parallel.


So, the distance between them will give the length of the side of the square.


Let d be the distance between the given lines.



∴ Area of the square = 72
= 49 square units



Question 6.

Find the equation of two straight lines which are parallel to x + 7y + 2 = 0 and at unit distance from the point (1, -1).


Answer:

Given: equation is parallel to x + 7y + 2 = 0 and at unit distance from the point (1, -1)


To find: equation of two straight lines


Explanation:


The equation of given line is


x + 7y + 2 = 0 … (1)


The equation of a line parallel to line x + 7y + 2 = 0 is given below:


X + 7y + λ = 0 … (2)


The line x + 7y + λ = 0 is at a unit distance from the point (1, − 1).


∴ 1 = 1 – 7 + λ1 + 49




Hence, the required lines:


and



Question 7.

Prove that the lines 2x + 3y = 19 and 2x + 3y + 7 = 0 are equidistant from the line 2x + 3y = 6.


Answer:

Given: lines A 2x + 3y = 19 and B 2x + 3y + 7 = 0 also a line C 2x + 3y = 6.


To prove:


Line A and B are equidistant from the line C


Proof:


Let d1 be the distance between lines 2x + 3y = 19 and 2x + 3y = 6,



While d2 is the distance between lines 2x + 3y + 7 = 0 and 2x + 3y = 6




Hence proved, the lines 2x + 3y = 19 and 2x + 3y + 7 = 0 are equidistant from the line 2x + 3y = 6



Question 8.

Find the equation of the line mid-way between the parallel lines 9x + 6y – 7 = 0 and 3x + 2y + 6 = 0.


Answer:

Given:


9x + 6y – 7 = 0 and 3x + 2y + 6 = 0 are parallel lines


To find:


The equation of the line mid-way between the givens line


Explanation:


The given equations of the lines can be written as:


… (1)


3x + 2y + 6 = 0 … (2)


Let the equation of the line midway between the parallel lines (1) and (2) be


3x + 2y + λ = 0 … (3)


The distance between (1) and (3) and the distance between (2) and (3) are equal.






Equation of the required line:



⇒ 18x + 12y + 11 = 0


Hence, equation of required line is 18x + 12y + 11 = 0.



Question 9.

Find the ratio in which the line 3x + 4y + 2 = 0 divides the distance between the lines 3x + 4y + 5 = 0 and 3x + 4y – 5 = 0


Answer:

Given:


Lines A: 3x + 4y + 5 = 0 and B: 3x + 4y – 5 = 0


And also C: 3x + 4y + 2 = 0


To find:


Ratio in which line C divides the distance between the lines A and B


Explanation:


The distance between two parallel line ax + by + c1 = 0 and ax + by + c2 = 0 is



Therefore the distance between two parallel line 3x + 4y + 5 = 0 and 3x + 4y – 5 = 0 is



The distance between the line 3x + 4y + 2 = 0 and 3x + 4y – 5 = 0 is



Hence, the required ratio




Exercise 23.17
Question 1.

Prove that the area of the parallelogram formed by the lines

a1x + b1y + c1 = 0, a1x + b1y + d1 = 0, a2x + b2y + c2 = 0, a2x + b2y + d2 = 0 is sq. units.

Deduce the condition for these lines to form a rhombus.


Answer:

Given:


The given lines are


a1x + b1y + c1 = 0 … (1)


a1x + b1y + d1 = 0 … (2)


a2x + b2y + c2 = 0 … (3)


a2x + b2y + d2 = 0 … (4)


To prove:


The area of the parallelogram formed by the lines


a1x + b1y + c1 = 0, a1x + b1y + d1 = 0, a2x + b2y + c2 = 0, a2x + b2y + d2 = 0 is sq. units.


Explanation:


The area of the parallelogram formed by the lines a1x + b1y + c1 = 0, a1x + b1y + d1 = 0, a2x + b2y + c2 = 0 and a2x + b2y + d2 = 0 is given below:


Area


a1b2 – a2b1


∴ Area


If the given parallelogram is a rhombus, then the distance between the pair of parallel lines are equal.



Hence proved.



Question 2.

Prove that the area of the parallelogram formed by the lines 3x – 4y + a = 0, 3x – 4y + 3a = 0, 4x – 3y – a = 0 and 4x – 3y – 2a = 0 is sq. units.


Answer:

Given:


The given lines are


3x − 4y + a = 0 … (1)


3x − 4y + 3a = 0 … (2)


4x − 3y − a = 0 … (3)


4x − 3y − 2a = 0 … (4)


To prove:


The area of the parallelogram formed by the lines 3x – 4y + a = 0, 3x – 4y + 3a = 0, 4x – 3y – a = 0 and 4x – 3y – 2a = 0 is sq. units.


Explanation:


From Above solution, We know that


Area of the parallelogram


⇒ Area of the parallelogramsquare units


Hence proved.



Question 3.

Show that the diagonals of the parallelogram whose sides are lx + my + n = 0, lx + my + n’ = 0, mx + ly + n = 0 and mx + ly + n’ = 0 include an angle.


Answer:

Given:


The given lines are


lx + my + n = 0 … (1)


mx + ly + n’ = 0 … (2)


lx + my + n’ = 0 … (3)


mx + ly + n = 0 … (4)



To prove:


The diagonals of the parallelogram whose sides are lx + my + n = 0, lx + my + n’ = 0, mx + ly + n = 0 and mx + ly + n’ = 0 include an angle.


Explanation:


Solving (1) and (2), we get,


B


Solving (2) and (3), we get,


C


Solving (3) and (4), we get,


D


Solving (1) and (4), we get,


A


Let m1 and m2 be the slope of AC and BD.




∴m1m2 = -1


Hence proved, diagonals of the parallelogram intersect at an angle




Exercise 23.18
Question 1.

Find the equation of the straight lines passing through the origin and making an angle of 450 with the straight line.


Answer:

Given:


Equation passes through (0, 0) and make an angle of 45° with the line


To find:


Equation of given line


Explanation:


We know that, the equations of two lines passing through a point x1,y1 and making an angle α with the given line y = mx + c are



Here,
x1 = 0, y1 = 0, α = 45° and m =


So, the equations of the required lines are






Hence, Equation of given line is



Question 2.

Find the equations to the straight lines which pass through the origin and are inclined at an angle of 750 to the straight line.


Answer:

Given:


Equation passes through (0,0) and make an angle of 75° with the line


To find:


Equation of given line


Explanation:


We know that the equations of two lines passing through a point x1,y1 and making an angle α with the given line y = mx + c are



Here,
Equation of the given line is,





Comparing this equation with y = mx + c


We get,


∴ x1 = 0, y1 = 0, α = 75°, m


and tan75∘ = 2 +


So, the equations of the required lines are






Hence, Equation of given line is



Question 3.

Find the equations of straight lines passing through (2, -1) and making an angle of 450 with the line 6x + 5y – 8 = 0.


Answer:

Given: equation passes through (2,-1) and make an angle of 45° with the line 6x + 5y – 8 = 0


To find: equation of given line


Explanation:


We know that the equations of two lines passing through a point x1,y1 and making an angle α with the given line y = mx + c are



Here,
Equation of the given line is,


6x + 5y – 8 = 0


⇒ 5y = - 6x + 8



Comparing this equation with y = mx + c


we get, m


x1 = 2, y1 = - 1, α = 45°, m


So, the equations of the required lines are





⇒ x + 11y + 9 = 0 and 11x – y – 23 = 0


Hence, Equation of given line is x + 11y + 9 = 0 and 11x – y – 23 = 0



Question 4.

Find the equations to the straight lines which pass through the point (h, k) and are inclined at angle tan-1 m to the straight line y = mx + c.


Answer:

Given: equation passes through (h, k) and make an angle of tan-1 m with the line y = mx + c


To find: equation of given line


Explanation:


We know that the equations of two lines passing through a point x1,y1 and making an angle α with the given line y = m’x + c are


m’ = m so,



Here,
x1 = h, y1 = k, α m’ = m.


So, the equations of the required lines are




and y = k


Hence, Equation of given line is and y = k



Question 5.

Find the equations to the straight lines passing through the point (2, 3) and inclined at an angle of 450 to the lines 3x + y – 5 = 0.


Answer:

Given: equation passes through (2,3) and make an angle of 450with the line 3x + y – 5 = 0.


To find: equation of given line


Explanation:


We know that the equations of two lines passing through a point x1,y1 and making an angle α with the given line y = mx + c are



Here,
Equation of the given line is,


3x + y – 5 = 0


⇒ y = - 3x + 5


Comparing this equation with y = mx + c we get, m = - 3
x1 = 2, y1 = 3, α = 45∘, m = - 3.


So, the equations of the required lines are





⇒ x + 2y – 8 = 0 and 2x – y – 1 = 0


Hence, Equation of given line is x + 2y – 8 = 0 and 2x – y – 1 = 0



Question 6.

Find the equations to the sides of an isosceles right angled triangle the equation of whose hypotenuse is 3x + 4y = 4 and the opposite vertex is the point (2, 2).


Answer:

Given: hypotenuse is 3x + 4y = 4 of isosceles right angled triangle the opposite vertex is the point (2, 2).


To find: equation of side of isosceles right angle triangle


Explanation:


Here,


we are given △ABC is an isosceles right angled triangle .


∠A + ∠B + ∠C = 180°


⇒ 90° + ∠B + ∠B = 180°


⇒ ∠B = 45°, ∠C = 45°


Diagram:



Now, we have to find the equations of the sides AB and AC, where 3x + 4y = 4 is the equation of the hypotenuse BC.


We know that the equations of two lines passing through a point x1,y1 and making an angle α with the given line y = mx + c are



Here,
Equation of the given line is,


3x + 4y = 4


⇒ 4y = - 3x + 4



Comparing this equation with y = mx + c


we get,



x1 = 2, y1 = 2, α = 45∘, m


So, the equations of the required lines are





⇒ x – 7y + 12 = 0 and 7x + y – 16 = 0


Hence, Equation of given line is x – 7y + 12 = 0 and 7x + y – 16 = 0



Question 7.

The equation of one side of an equilateral triangle is x – y = 0 and one vertex is . Prove that a second side is and find the equation of the third side.


Answer:

Given: equation of one side of an equilateral triangle is x – y = 0 and one vertex is


To prove: second side is


To find: the equation of the third side.


Explanation:


Let A be the vertex of the equilateral triangle ABC and x − y = 0 be the equation of BC.
Here, we have to find the equations of sides AB and AC, each of which makes an angle of 60∘ with the line x − y = 0


We know the equations of two lines passing through a point x1,y1 and making an angle α with the line whose slope is m.



Here,
x1, y1 = 5, α = 60∘, m = 1


So, the equations of the required sides are


and


and


and


and


Hence, the second side is and the equation of the third side is




Question 8.

Find the equations of the two straight lines through (1, 2) forming two sides of a square of which 4x + 7y = 12 is one diagonal.


Answer:

Given: 4x + 7y = 12 is one diagonal and opposite vertex is (1,2)


To find: equation of straight line


Explanation:


Let A (1, 2) be the vertex of square ABCD and BD be the diagonal that is along the line 4x + 7y = 12


Diagram:



Here, we have to find the equations of sides AB and AD, each of which makes an angle of 45∘ with line 4x + 7y = 12


We know that the equations of two lines passing through a point x1,y1 and making an angle α with the line whose slope is m.



Equation of given line is


4x + 7y = 9



m, x1 = 1, y1 = 2, α = 45∘


So, the equations of the required sides are




⇒ 3x – 11y + 19 = 0 and 11x + 3y – 17 = 0


Hence, equation of straight line 3x – 11y + 19 = 0 and 11x + 3y – 17 = 0



Question 9.

Find the equations of two straight lines passing through (1, 2) and making an angle of 600 with the lines x + y = 0. Find also the area of the triangle formed by the three lines.


Answer:

Given: equation passes through (1,2) and make an angle of 60° with the line x + y = 0


To find: equation of given line and the area of the triangle formed by the three lines


Explanation:


Let A(1, 2) be the vertex of the triangle ABC and x + y = 0 be the equation of BC.


Diagram:



Here, we have to find the equations of sides AB and AC, each of which makes an angle of 60∘ with the line x + y = 0.


We know the equations of two lines passing through a point x1,y1 and making an angle α with the line whose slope is m.



Here,
x1 = 1, y1 = 2, α = 60∘, m = - 1


So, the equations of the required sides are


and





Solving x + y = 0 and , we get:




AB = BC = AD units


∴ Area of the required triangle square units


Hence, area of the required triangle square units



Question 10.

Two sides of an isosceles triangle are given by the equations 7x – y + 3 = 0 and x + y – 3 = 0 and its third side passes through the point (1, -10). Determine the equation of the third side.


Answer:

Given: two side of an isosceles triangle are 7x – y + 3 = 0 and x + y – 3 = 0 and its third side passes through the point (1, -10)


To find: third side of isosceles triangle


Explanation:


Let ABC be the isosceles triangle, where 7x − y + 3 = 0 and x + y − 3 = 0 represent the sides AB and AC, respectively.
Let AB = BC


Diagram:



∵ AB = BC


∴ tan B = tan C


Here,


Slope of AB = 7


Slope of AC = − 1


Let m be the slope of BC.


Then,



Taking the positive sign, we get:


m2 – 8m + 7 = 7m2 + 8m + 1




Now, taking the negative sign, we get:


(m – 7) (m – 1) = - (7m + 1)(m + 1)


⇒ m2 – 8m + 7 = - 7m2 – 8m – 1


⇒ m2 = - 1 (not possible)


Equations of the third side is


y + 10 = - 3(x – 1) and


⇒ 3x + y + 7 = 0 and x – 3y – 31 = 0


Hence, third side of isosceles triangle is 3x + y + 7 = 0 and x – 3y – 31 = 0



Question 11.

Show that the point (3, -5) lies between the parallel lines 2x + 3y – 7 = 0 and 2x + 3y + 12 = 0 and find the equation of lines through (3, -5) cutting the above lines at an angle of 45°.


Answer:

Given:


Parallel lines 2x + 3y – 7 = 0 and 2x + 3y + 12 = 0 and


To prove:


The point (3, -5) lies between the parallel lines 2x + 3y – 7 = 0 and 2x + 3y + 12 = 0


To find:


Lines through (3, -5) cutting the above lines at an angle of 450.


Explanation:


We observed that (0, -4) lies on the line 2x + 3y + 12 = 0


If (3, 5) lies between the lines 2x + 3y – 7 = 0 and 2x + 3y + 12 = 0, then we have,


(ax1 + by1 + c1)(ax2 + by2 + c1) > 0


Here, x1 = 0, y1 = -4, x2 = 3, y2 = -5, a = 2, b = 3, c1 = -7


Now,


(ax1 + by1 + c1)(ax2 + by2 + c1) = (2 × 0 – 3 × 4 – 7) (2 × 3 – 3 × 5 – 7)


(ax1 + by1 + c1)(ax2 + by2 + c1) = -19 × (-16) > 0


Thus, point (3, -5) lies between the given parallel lines.


The equation of the lines passing through (3, -5) and making angle of 45 with the given parallel lines is given below:



Here,
x1 = 3, y1 = - 5, α = 45∘, m


So, the equations of the required sides are



and


and


x – 5y – 28 = 0 and 5x + y – 10 = 0


Hence, equation of required line is x – 5y – 28 = 0 and 5x + y – 10 = 0


Hence proved.



Question 12.

The equation of the base of an equilateral triangle is x + y = 2 and its vertex is (2, -1). Find the length and equations of its sides.


Answer:

Given: equation of the base of an equilateral triangle is x + y = 2 and its vertex is (2, -1)


To find: length and equations of its sides


Explanation:


Let A (2, − 1) be the vertex of the equilateral triangle ABC and x + y = 2 be the equation of BC.


Diagram:



Here, we have to find the equations of the sides AB and AC, each of which makes an angle of 60∘ with the line x + y = 2


The equations of two lines passing through point x1,y1 and making an angle α with the line whose slope is m is given below:



Here,
x1 = 2, y1 = - 1, α = 60∘, m = -1


So, the equations of the required sides are


and


and





Solving x + y = 2 and , we get:




∴ AB = BC = AC =
Hence, equations of its sides are given below: ,



Question 13.

If two opposites vertices of a square are (1, 2) and (5, 8), find the coordinates of its other two vertices and the equations of its sides.


Answer:

Given: two opposites vertices of square are (1,2) and (5,8)


To find: opposite’s vertices of a square and equation of sides.


Explanation:


Let A (1, 2) be the vertex of square ABCD and BD be the diagonal that is along the line 8x − 15y = 0


Equation of the given line is, 8x – 15y = 0


⇒ - 15y = - 8x



Comparing this equation with y = mx + c


We get, m


So, the slope of BD will be .
Here, we have to find the equations of sides AB and AD.


We know that the equations of two lines passing through a point x1,y1 and making an angle α with the line whose slope is m.



Here,


m, x1 = 1, y1 = 2, α = 45∘


So, the equations of the required sides are




⇒ 23x – 7y – 9 = 0 and 7x + 23y – 53 = 0


Hence, equation of sides is 23x – 7y – 9 = 0 and 7x + 23y – 53 = 0




Exercise 23.19
Question 1.

Find the equation of a straight line through the point of intersection of the lines 4x – 3y = 0 and 2x – 5y + 3 = 0 and parallel to 4x + 5 y + 6 = 0.


Answer:

Given:


Lines 4x – 3y = 0 and 2x – 5y + 3 = 0 and parallel to 4x + 5 y + 6 = 0


To find:


The equation of a straight line through the point of intersection of the lines


Explanation:


The equation of the straight line passing through the points of intersection of 4x − 3y = 0 and 2x − 5y + 3 = 0 is given below:


4x − 3y + λ (2x − 5y + 3) = 0


⇒ (4 + 2λ)x + (− 3 − 5λ)y + 3λ = 0


⇒ y


The required line is parallel to 4x + 5y + 6 = 0 or, y


⇒ λ


Hence, the required equation is


⇒ 28x + 35y – 48 = 0



Question 2.

Find the equation of a straight line passing through the point of intersection of x + 2y + 3 = 0 and 3x + 4y + 7 = 0 and perpendicular to the straight line x – y + 9 = 0.


Answer:

Given:


x + 2y + 3 = 0 and 3x + 4y + 7 = 0


To find:


The equation of a straight line passing through the point of intersection of x + 2y + 3 = 0 and 3x + 4y + 7 = 0 and perpendicular to the straight line x – y + 9 = 0.


Explanation:


The equation of the straight line passing through the points of intersection of x + 2y + 3 = 0 and 3x + 4y + 7 = 0 is


x + 2y + 3 + λ(3x + 4y + 7) = 0


⇒ (1 + 3λ)x + (2 + 4λ)y + 3 + 7λ = 0


⇒ y


The required line is perpendicular to x − y + 9 = 0 or, y = x + 9



⇒ λ = -1


Required equation is given below:


(1 − 3)x + (2 − 4)y + 3 − 7 = 0


⇒ x + y + 2 = 0


Hence, required equation is x + y + 2 = 0



Question 3.

Find the equation of the line passing through the point of intersection of 2x – 7y + 11 = 0 and x + 3y – 8 = 0 and is parallel to (i) x = axis (ii) y-axis.


Answer:

Given:


2x – 7y + 11 = 0 and x + 3y – 8 = 0


To find:


The equation of the line passing through the point of intersection of 2x – 7y + 11 = 0 and x + 3y – 8 = 0 and is parallel to (i) x = axis (ii) y-axis.


Explanation:


The equation of the straight line passing through the points of intersection of 2x − 7y + 11 = 0 and x + 3y − 8 = 0 is given below:


2x − 7y + 11 + λ(x + 3y − 8) = 0


⇒ (2 + λ)x + (− 7 + 3λ)y + 11 − 8λ = 0
(i) The required line is parallel to the x-axis. So, the coefficient of x should be zero.


∴ 2 + λ = 0


⇒ λ = -2


Hence, the equation of the required line is


0 + (− 7 − 6)y + 11 + 16 = 0


⇒ 13y − 27 = 0


(ii) The required line is parallel to the y-axis. So, the coefficient of y should be zero.


∴ -7 + 3λ = 0


⇒ λ


Hence, the equation of the required line is



⇒ 13x – 23 = 0



Question 4.

Find the equation of the straight line passing through the point of intersection of 2x + 3y + 1 = 0 and 3x – 5y – 5 = 0 and equally inclined to the axes.


Answer:

Given:


2x + 3y + 1 = 0 and 3x – 5y – 5 = 0


To find:


The equation of the straight line passing through the point of intersection of 2x + 3y + 1 = 0 and 3x – 5y – 5 = 0 and equally inclined to the axes.


Explanation:


The equation of the straight line passing through the points of intersection of 2x + 3y + 1 = 0 and 3x − 5y − 5 = 0 is


2x + 3y + 1 + λ(3x − 5y − 5) = 0


⇒ (2 + 3λ)x + (3 − 5λ)y + 1 − 5λ = 0
⇒ y


The required line is equally inclined to the axes. So, the slope of the required line is either 1 or − 1.


and


⇒ -2 – 3λ = 3 – 5λ and 2 + 3λ = 3 – 5λ


⇒ λ and


Substituting the values of λ in (2 + 3λ)x + (3 − 5λ)y + 1 − 5λ = 0, we get the equations of the required lines.



⇒ 19x – 19y – 23 = 0 and 19x + 19y + 3 = 0


Hence, required equation is 19x – 19y – 23 = 0 and 19x + 19y + 3 = 0



Question 5.

Find the equation of the straight line drawn through the point of intersection of the lines x + y = 4 and 2x – 3y = 1 and perpendicular to the line cutting off intercepts 5, 6 on the axes.


Answer:

Given:


lines x + y = 4 and 2x – 3y = 1


To find:


The equation of the straight line drawn through the point of intersection of the lines x + y = 4 and 2x – 3y = 1 and perpendicular to the line cutting off intercepts 5, 6 on the axes.


Explanation:


The equation of the straight line passing through the point of intersection of x + y = 4 and 2x − 3y = 1 is


x + y − 4 + λ(2x − 3y − 1) = 0


⇒ (1 + 2λ)x + (1 − 3λ)y − 4 − λ = 0 … (1)


⇒ y


The equation of the line with intercepts 5 and 6 on the axis is


… (2)


The slope of this line is


The lines (1) and (2) are perpendicular.



⇒ λ


Substituting the values of λ in (1), we get the equation of the required line.



⇒ 25x – 30y – 23 = 0


Hence, required equation is 25x – 30y – 23 = 0



Question 6.

Prove that the family of lines represented by x(1 + λ) + y(2 – λ) + 5 = 0, λ being arbitrary, pass through a fixed point. Also, find the fixed point.


Answer:

Given:


Lines represented by x(1 + λ) + y(2 – λ) + 5 = 0, λ being arbitrary


To prove:


The family of lines represented by x(1 + λ) + y(2 – λ) + 5 = 0, λ being arbitrary, pass through a fixed point. Also, find the fixed point.


Explanation:


The given family of lines can be written as


x + 2y + 5 + λ (x − y) = 0


This line is of the form L1 + λL2 = 0, which passes through the intersection of L1 = 0 and L2 = 0.
⇒ x + 2y + 5 = 0
⇒ x − y = 0


Now, solving the lines:
This is a fixed point.


Hence proved.



Question 7.

Show that the straight lines given by (2 + k)x + (1 + k)y = 5 + 7k for different values of k pass through a fixed point. Also, find that point.


Answer:

Given:


lines given by (2 + k)x + (1 + k)y = 5 + 7k


To prove:


The straight lines given by (2 + k)x + (1 + k)y = 5 + 7k for different values of k pass through a fixed point


Explanation:


The given straight line (2 + k)x + (1 + k)y = 5 + 7k can be written in the following way:


2x + y − 5 + k (x + y − 7) = 0


This line is of the form L1 + kL2 = 0, which passes through the intersection of the lines
L1 = 0 and L2 = 0, i.e. 2x + y − 5 = 0 and x + y − 7 = 0.


Solving 2x + y − 5 = 0 and x + y − 7 = 0, we get (− 2, 9), which is the fixed point.


Hence proved.



Question 8.

Find the equation of the straight line passing through the point of intersection of 2x + y – 1 = 0 and x + 3y – 2 = 0 and making with the coordinate axes a triangle of area 3/8 sq. units.


Answer:

Given:


2x + y – 1 = 0 and x + 3y – 2 = 0


To find:


The equation of the straight line passing through the point of intersection of 2x + y – 1 = 0 and x + 3y – 2 = 0 and making with the coordinate axes a triangle of area 3/8 sq. units.


Explanation:


The equation of the straight line passing through the point of intersection of 2x + y − 1 = 0 and x + 3y − 2 = 0 is given below:


2x + y − 1 + λ (x + 3y − 2) = 0


⇒ (2 + λ)x + (1 + 3λ)y − 1 − 2λ = 0



So, the points of intersection of this line with the coordinate axes are and


It is given that the required line makes an area of square units with the coordinate axes.



⇒ 3 |3λ2 + 7λ + 2| = 4 |4λ2 + 4λ + 1|


⇒ 9λ2 + 21λ + 6 = 16λ2 + 16λ + 4


⇒ 7λ2 – 5λ – 2 = 0


⇒ λ = 1,


Hence, the equations of the required lines are


3x + 4y – 1 – 2 = 0 and


⇒ 3x + 4y – 3 = 0 and 12x + y – 3 = 0



Question 9.

Find the equation of the straight line which passes through the point of intersection of the lines 3x – y = 5 and x + 3y = 1 and makes equal and positive intercepts on the axes.


Answer:

Given:


Lines 3x – y = 5 and x + 3y = 1


To find:


The equation of the straight line which passes through the point of intersection of the lines 3x – y = 5 and x + 3y = 1 and makes equal and positive intercepts on the axes.


Explanation:


The equation of the straight line passing through the point of intersection of 3x − y = 5 and x + 3y = 1 is


3x − y − 5 + λ(x + 3y − 1) = 0


⇒ (3 + λ)x + (− 1 + 3λ)y − 5 − λ = 0 … (1)
⇒ y


The slope of the line that makes equal and positive intercepts on the axis is − 1.


From equation (1), we have:



⇒ λ = 2


Substituting the value of λ in (1), we get the equation of the required line.


⇒ 3 + 2x + -1 + 6y – 5 – 2 = 0


⇒ 5x + 5y – 7 = 0


Hence, equation of required line is 5x + 5y – 7 = 0



Question 10.

Find the equations of the lines through the point of intersection of the lines x – 3y + 1 = 0 and 2x + 5y – 9 = 0 and whose distance from the origin is.


Answer:

Given:


Lines x – 3y + 1 = 0 and 2x + 5y – 9 = 0


To find:


The equations of the lines through the point of intersection of the lines x – 3y + 1 = 0 and 2x + 5y – 9 = 0 and whose distance from the origin is.


Explanation:


The equation of the straight line passing through the point of intersection of x − 3y + 1 = 0 and 2x + 5y − 9 = 0 is given below:


x − 3y + 1 + λ(2x + 5y − 9) = 0


⇒ (1 + 2λ)x + (− 3 + 5λ)y + 1 − 9λ = 0 … (1)


The distance of this line from the origin is



⇒ 1 + 81λ2 – 18λ = 145λ2 – 130λ + 50


⇒ 64λ2 – 112λ + 49 = 0


⇒ (8λ – 7)2 = 0


⇒ λ


Substituting the value of λ in (1), we get the equation of the required line.



⇒ 22x + 11y – 55 = 0


⇒ 2x + y – 5 = 0


Hence, equation of required line is 2x + y – 5 = 0.



Question 11.

Find the equations of the lines through the point of intersection of the lines x – y + 1 = 0 and 2x – 3y + 5 = 0 whose distance from the point (3, 2) is .


Answer:

Given:


Lines x – y + 1 = 0 and 2x – 3y + 5 = 0


To find:


The equations of the lines through the point of intersection of the lines x – y + 1 = 0 and 2x – 3y + 5 = 0 whose distance from the point (3, 2) is 7/5.


Explanation:


The equation of the straight line passing through the point of intersection of x – y + 1 = 0 and 2x – 3y + 5 = 0 is given below:


x − y + 1 + λ(2x – 3y + 5) = 0


⇒ (1 + 2λ)x + (− 3λ – 1)y + 5λ + 1 = 0 … (1)


The distance of this line from the point is given by




⇒ 25(5λ + 2)2 = 49(13λ2 + 10λ + 2)


⇒ 6λ2 – 5λ – 1 = 0


⇒ λ


Substituting the value of λ in (1), we get the equation of the required line.


⇒ 3x – 4y + 6 = 0 and 4x – 3y + 1 = 0


Hence, equation of required line is 3x – 4y + 6 = 0 and 4x – 3y + 1 = 0.




Exercise 23.2
Question 1.

Find the equation of the parallel to x–axis and passing through (3, – 5).


Answer:

Given, A line which is parallel to x–axis and passing through (3, – 5)

To Find: The equation of the line.


Formula used: The equation of line is [y – y1 = m(x – x1)]


Explanation: Here, The line is parallel to the x–axis,


So, The parallel lines have equal slopes,


And, the slope of x–axis is always 0, then


The slope of line, m = 0


Coordinates of line are (x1, y1) = (3, – 5)


The equation of line = y – y1 = m(x – x1)


By putting the values, we get


y – (– 5) = 0(x – 3)


y + 5 = 0


Hence, The equation of line is y + 5 = 0



Question 2.

Find the equation of the line perpendicular to x–axis and having intercept – 2 on x–axis.


Answer:

Given, A line which is perpendicular to x–axis and having intercept – 2.

To Find: The equation of the line.


Formula used: The equation of line is [y – y1 = m(x – x1)]


Explanation: Here, The line is perpendicular to the x–axis, then x is 0 and y is – 1.


So, The slope of line is, m =


m =


Since, It is given that x–intercept is – 2, so, y is 0.


Coordinates of line are (x1, y1) = (– 2, 0)


The equation of line = y – y1 = m(x – x1)


By putting the values, we get


y – 0 = (x – (– 2))


x + 2 = 0


Hence, The equation of line is x + 2 = 0



Question 3.

Find the equation of the line parallel to x–axis and having intercept – 2 on y – axis.


Answer:

Given, A line which is parallel to x–axis and having intercept – 2 on y – axis.

To Find: The equation of the line.


Formula used: The equation of line is [y – y1 = m(x – x1)]


Explanation Here, The line is parallel to the x–axis,


So, The parallel lines have equal slopes,


And, the slope of x–axis is always 0, then


The slope of line, m = 0


Since, It is given that intercept is – 2, on y – axis then


Coordinates of line are (x1, y1) = (0, – 2)


The equation of line is y – y1 = m(x – x1) – – – – (1)


By putting the values in equation (1), we get


y – (– 2) = 0 (x – 0)


y + 2 = 0


Hence, The equation of line is y + 2 = 0



Question 4.

Draw the lines x = – 3, x = 2, y = – 2, y = 3 and write the coordinates of the vertices of the square so formed.


Answer:

Given, x = – 3, x = 2, y = – 2 and y = 3


Coordinates of the square are : A(2, 3), B(2, –2), C(–3, 3), and D(–3, –2).



Question 5.

Find the equations of the straight lines which pass through (4, 3) and are respectively parallel and perpendicular to the x–axis.


Answer:

Given, A line which is perpendicular and parallel to x–axis respectively and passing through (4, 3)

To Find: Find the equation of that line.


Formula used: The equation of line is [y – y1 = m(x – x1)]


Explanation:


Case 1 : When Line is parallel to x–axis


So, The parallel lines have equal slopes,


And, the slope of x–axis is always 0, then


The slope of line, m = 0


Coordinates of line are (x1, y1) = (4, 3)


The equation of line is y – y1 = m(x – x1) – – – – (1)


By putting the values in equation (1), we get


y – (3) = 0(x – 4)


y – 3 = 0


Case 2: when line is perpendicular to x–axis


Here, The line is perpendicular to the x–axis, then x is 0 and y is – 1.


So, The slope of the line is, m =


m =


Coordinates of line are (x1, y1) = (4, 3)


The equation of line = y – y1 = m(x – x1)


By putting the values, we get



x = 4


Hence, The equation of line when it is parallel to x –axis is y = 3 and it is perpendicular is x = 4.



Question 6.

Find the equation of the line which is equidistant from the lines x = –2 and x = 6.


Answer:

To Find: The equation of the line


Formula Used: The equation of line is [y – y1 = m(x – x1)]


Explanation:


Let us plot the lines



Now the line must be at the centre of the lines (–2, 0) and (6, 0).


The Midpoint formula =


Therefore,


Midpoint =


Midpoint = (2, 0)


Hence, the equation of the line is x = 2.



Question 7.

Find the equation of a line equidistant from the lines y = 10 and y = – 2.


Answer:

A line which is equidistant from the lines y = 10 and y = – 2

To Find: The equation of the line


Formula used: The equation of line is [y – y1 = m(x – x1)]


Explanation: A line which is equidistant from, two other lines,


So, the slopes must be the same .


Therefore, The slope of line y = 10 and y = – 2 is 0, because lines are parallel to the x–axis.


Since, The required line will pass from the midpoint of the line joining (0, – 2) and (0, 10)


The Midpoint formula =


So, The coordinates of the point will be (0, 4)


Since The equation of the line is :


y – 4 = 0(x – 0)


y = 4


Hence, The equation of the line is y = 4




Very Short Answer
Question 1.

Write an equation representing a pair of lines through the point (a, b) and parallel to the coordinates axes.


Answer:

Given:


Point (a, b)


To find:


Equation representing a pair of lines through the point (a, b) and parallel to the coordinates axes.


Explanation:


The lines passing through (a, b) and parallel to the x-axis and y-axis are y = b and x = a, respectively.


Therefore, their combined equation is given below:


(x – a)(y – b) = 0



Question 2.

Write the coordinates of the orthocenter of the triangle formed by the lines x2 – y2 = 0 and x + 6y = 18.


Answer:

Given:


Lines x2 – y2 = 0 and x + 6y = 18.


To find:


The coordinates of the orthocenter of the triangle formed by the lines x2 – y2 = 0 and x + 6y = 18.


Explanation:


The equation x2 − y2 = 0 represents a pair of straight line, which can be written in the following way:


(x + y)(x − y) = 0


So, the lines can be written separately in the following manner:


x + y = 0 … (1)


x − y = 0 … (2)


The third line is


x + 6y = 18 … (3)


Lines (1) and (2) are perpendicular to each other as their slopes are − 1 and 1, respectively
⇒ − 1 × 1 = − 1


Therefore, the triangle formed by the lines (1), (2) and (3) is a right-angled triangle.


Thus, the orthocentre of the triangle formed by the given lines is the intersection of x + y = 0 and x − y = 0, which is (0, 0).



Question 3.

If the centroid of a triangle formed by the points (0, 0), (cos θ, sin θ) and (sinθ, - cosθ) lies on the line y = 2x, then write the value of tanθ.


Answer:

Given:


The points (0, 0), (cos θ, sin θ) and (sinθ, - cosθ) lies on the line y = 2x


To find:


The value of tanθ.


Explanation:


The centroid of a triangle with vertices (x1,y1), (x2, y2) and (x3, y3)is given below:



Therefore, the centre of the triangle having vertices (0, 0,), (cos θ, sin θ) and (sin θ, − cos θ) is


This point lies on the line y = 2x.



⇒ sinθ – cosθ = 2cosθ + 2sinθ


⇒ tanθ = -3


∴ tanθ = − 3


Hence, tanθ = − 3



Question 4.

Write the value of for which area of the triangle formed by points O(0, 0), A(a cos θ, b sin θ) and (a cos θ, - b sin θ) is maximum.


Answer:

Given:


Points O(0, 0), A(a cos θ, b sin θ) and (a cos θ, - b sin θ)


To find:


The value of for which area of the triangle formed by points O(0, 0), A(a cos θ, b sin θ) and (a cos θ, - b sin θ) is maximum.


Explanation:


Let A be the area of the triangle formed by the points O (0,0), A (acosθ, bsinθ) and B (acosθ, − bsinθ)


A


⇒ A


⇒ A = ab sinθcosθ


Now,
∴Amax, when sin2θ = 1


⇒ ∴Amax, when 2θ = π/2


⇒ θ


Hence, the area of the triangle formed by the given points is maximum when



Question 5.

Write the distance the lines 4x + 3y – 11 = 0 and 8x + 6y – 15 = 0.


Answer:

Given:


Lines 4x + 3y – 11 = 0 and 8x + 6y – 15 = 0


To find:


Distance between lines.


Explanation:


The distance between the two parallel lines ax + by + c1 = 0 and ax + by + c2 = 0 is
the given lines can be written as


4x + 3y − 11 = 0 … (1)


8x + 6y – 15 = 0


… (2)


Let d be the distance between the lines (1) and (2).



Hence,



Question 6.

Write the coordinates of the orthocenter of the triangle formed by the lines xy = 0 and x + y = 1


Answer:

Given:


Lines xy = 0 and x + y = 1


To find:


The coordinates of the orthocenter of the triangle formed by the lines xy = 0 and x + y = 1


Explanation:


The equation xy = 0 represents a pair of straight lines.


The lines can be written separately in the following way:


x = 0 … (1)


y = 0 … (2)


The third line is


x + y = 1 … (3)


Lines (1) and (2) are perpendicular to each other as they are coordinate axes.


Therefore, the triangle formed by the lines (1), (2) and (3) is a right-angled triangle.


Thus, the orthocentre of the triangle formed by the given lines is the intersection of x = 0 and y = 0, which is (0, 0).



Question 7.

If the lines x + ay + a = 0, bx + y + b = 0 and cx + cy + 1 = 0 are concurrent then write the value of 2abc – ab – bc – ca.


Answer:

Given:


Lines x + ay + a = 0, bx + y + b = 0 and cx + cy + 1 = 0


To find:


The value of 2abc – ab – bc – ca.


Explanation:


The given lines are


x + ay + a = 0 … (1)


bx + y + b = 0 … (2)


cx + cy + 1 = 0 … (3)


It is given that the lines (1), (2) and (3) are concurrent.



⇒ (1 – bc) – a(b – bc) + a(bc – c) = 0


⇒ 1 – bc – ab + abc + abc – ac = 0


⇒ 2abc – ab – bc – ca = -1


Hence, the value of 2abc − ab − bc − ca is − 1



Question 8.

Write the area of the triangle formed by the coordinate axes and the line (sec θ – tan θ)x + (sec θ + tan θ) y = 2.


Answer:

Given:


Line (sec θ – tan θ)x + (sec θ + tan θ) y = 2.


To find:


The area of the triangle formed by the coordinate axes and the line (sec θ – tan θ)x + (sec θ + tan θ) y = 2.


Explanation:


The point of intersection of the coordinate axes is (0, 0).
Let us find the intersection of the line (sec θ − tan θ) x + (sec θ + tan θ) y = 2 and the coordinate axis.


For x-axis:


y = 0, x


For y-axis:


x = 0, y


Thus, the coordinates of the triangle formed by the coordinate axis and the line (sec θ − tan θ) x + (sec θ + tan θ) y = 2 are (0, 0), and .


Let A be the area of the required triangle.


∴ A


⇒ A


⇒ A


Hence, the area of the triangle is 2 square units.



Question 9.

If the diagonals of the quadrilateral formed by the lines l1x + m1y + n1 = 0, l2x + m2y + n2 = 0, l1x + m1y + n’1 = 0 and l2x + m2y + n2’ = 0 are perpendicular, then write the value of l12 – l22 + m12 – m22.


Answer:

Given:


l1x + m1y + n1 = 0 … (1)


l2x + m2y + n2 = 0 … (2)


l1x + m1y + n’1 = 0 … (3)


l2x + m2y + n2‘ = 0 … (4)


To find:


The value of l12 – l22 + m12 – m22.


Explanation:


Assuming:


(1), (2), (3) and (4) represent the sides AB, BC, CD and DA, respectively.


The equation of diagonal AC passing through the intersection of (2) and (3) is given by
l1x + m1y + n’1+ λ(l2x + m2y + n2) = 0


⇒ (l1 + λl2)x + (m1 + λm2)y + (n1’ + λn2) = 0


⇒ Slope of diagonal AC


Also, the equation of diagonal BD, passing through the intersection of (1) and (2), is given by
l1x + m1y + n1 + μ(l2x + m2y + n2) = 0


⇒ l1 + μl2x + m1 + μm2y + n1 + μn2 = 0


⇒ Slope of diagonal BD


The diagonals are perpendicular to each other.



⇒ (l1 + λl2)(l1 + λl2) = (-m1 + λm2)(m1 + μm2)


Let λ = -1, μ = 1


⇒ (l1 – l2)(l1 + l2) = (-m1-m2)(m1 + m2)


⇒ (l12-l22) = (-m12-m22)


⇒ (l12-l22) + (m12-m22) = 0


Hence, (l12-l22) + (m12-m22) = 0



Question 10.

Write the coordinates of the image of the point (3, 8) in the line x + 3y – 7 = 0.


Answer:

Given:


Line x + 3y – 7 = 0, point (3, 8).


To find:


The coordinates of the image.


Explanation:


Let the given point be A(3,8) and its image in the line x + 3y − 7 = 0 is B(h,k).


The midpoint of AB is that lies on the line x + 3y − 7 = 0.



h + 3k + 13 = 0 … (1)


AB and the line x + 3y − 7 = 0 are perpendicular.


∴Slope of AB × Slope of the line = - 1



⇒ 3h – k – 1 = 0 … (2)


Solving (1) and (2), we get:
(h, k) = (− 1, − 4)


Hence, the image of the point (3,8) in the line x + 3y − 7 = 0 is (− 1, − 4).



Question 11.

Write the integral values of m for which the x-coordinate of the point of intersection of the lines y = mx + 1 and 3x + 4y = 9 is an integer.


Answer:

Given:


Lines y = mx + 1 and 3x + 4y = 9


To find:


The integral values of m


Explanation:


The given lines can be written as


mx – y + 1 = 0 … (1)


3x + 4y – 9 = 0 … (2)


Solving (1) and (2) by cross multiplication, we get:



⇒ x


y
For x to be integer we have, 4m + 3 = 1, -1, 5 and -5


⇒ m,-1,and -2
Hence, the integral values of m are -1 and -2.



Question 12.

If a ≠ b ≠ c, write the condition for which the equation (b – c)x + (c – a) y + (a – b) = 0 and (b3 – c3)x + (c3 – a3)y + (a3 – b3) = 0 represent the same line


Answer:

Given:


The equation (b – c)x + (c – a) y + (a – b) = 0 and (b3 – c3)x + (c3 – a3)y + (a3 – b3) = 0


To find:


The condition for which the equation (b – c)x + (c – a) y + (a – b) = 0 and (b3 – c3)x + (c3 – a3)y + (a3 – b3) = 0 represent the same line


Explanation:


The given lines are


(b − c)x + (c − a)y + (a − b) = 0 … (1)


(b3 − c3)x + (c3 − a3)y + (a3 − b3) = 0 … (2)


The lines (1) and (2) will represent the same lines if





∵ (a ≠ b ≠ c)


⇒ b2 + bc + c2 = c2 + ac + a2 and c2 + ac + a2 = a2 + ab + b2


⇒ (a – b) (a + b + c) = 0 and (b – c) (b + c + a) = 0


⇒ a + b + c = 0 ∵(a ≠ b ≠ c)


Hence, the given lines will represent the same lines if a + b + c = 0.



Question 13.

If a, b, c are in G.P. write the area of the triangle formed by the line ax + by + c = 0 with the coordinates axes.


Answer:

Given:


a, b, c are in G.P.


To find:


Area of the triangle formed by the line ax + by + c = 0 with the coordinates axes.


Explanation:


The point of intersection of the line ax + by + c = 0 with the coordinate axis are (–c/a,0) and (0,-c/b).


So, the vertices of the triangle are (0, 0),


Let A be the area of the required triangle.


A


A


It is given that a, b and c are in GP.


∴ b2 = ac


⇒ A


Hence, area A



Question 14.

Write the area of the figure formed by the lines a|x| + b|y| + c = 0


Answer:

Given:


a x + b y + c = 0; x, y ≥ 0 … (1)


-a x + b y + c = 0; x < 0 y ≥ 0 … (2)


-a x – b y + c = 0; x < 0 y < 0 … (3)


a x – b y + c = 0; x ≥ 0 y < 0 … (4)


To find:


The area of the figure formed by the lines a|x| + b|y| + c = 0


Explanation:


The given lines can be written separately in the following way:


a x + b y + c = 0; x, y ≥ 0 … (1)


-a x + b y + c = 0; x < 0 y ≥ 0 … (2)


-a x – b y + c = 0; x < 0 y < 0 … (3)


a x – b y + c = 0; x ≥ 0 y < 0 … (4)


The lines and the region enclosed between them is shown below.



So, the area of the figures formed by the lines a |x| + b |y| + c = 0 is


Square units



Question 15.

Write the locus of a point the sum of whose distances from the coordinate’s axes is unity.


Answer:

Given:


Distances from the coordinate’s axes is unity.


To find:


The locus of a point the sum of whose distances from the coordinate’s axes is unity.


Assuming:


(h, k) be the locus.


Explanation:


It is given that the sum of distances of (h, k) from the coordinate axis is unity.


∴ |h| + |k| = 1


Taking locus of (h, k), we get:


|x| + |y| = 1


Hence, this represents a square.



Question 16.

If a, b, c are in A.P., then the line ax + by + c = 0 passes through a fixed point. Write the coordinates of that point.


Answer:

Given:


a, b, c are in A.P.


To find:


The coordinates of that point.


Explanation:


If a, b, c are in A.P., then


a + c = 2b


⇒ a – 2b + c = 0


Comparing the coefficient of ax + by + c = 0 and a – 2b + c = 0, we get x = 1 and y = -2


Hence, the coordinate of that point is (1, -2).



Question 17.

Write the equation of the line passing through the point (1, -2) and cutting off equal intercepts from the axes.


Answer:

Given:


Line passing through the point (1, -2) and cutting off equal intercepts from the axes.


To find:


The equation of the line


Explanation:


Let the required equation of the line is:



Now, passes through (1, -2)



⇒ a = -1


Hence, the required equation is:



⇒ x + y + 1 = 0


Hence, equation of required line is x + y + 1 = 0.



Question 18.

Find the locus of the mid-points of the portion of the line x sin θ + y cos θ = p intercepted between the axes.


Answer:

Given:


Line x sin θ + y cos θ = p


To find:


The locus of the mid-points of the portion of the line x sin θ + y cos θ = p intercepted between the axes.


Explanation:


If the equation of the given line is


x sin θ + y cos θ = p, then the solution is shown below:


The line


x sin θ + y cos θ = p intercepts the axes.


Thus, the coordinate of the poin where the line intercepts x – axis is



Thus, the coordinate of the poin where the line intercepts y – axis is



The midpoint R of the line is given by


R(h, k)


⇒ h , k


Eliminating the sine and cosine terms, we get




⇒ p2(h2 + k2) = 4h2k2


Thus, the locus is given by


p2(x2 + y2) = 4x2y2




Mcq
Question 1.

L is variable line such that the algebraic sum of the distances of the points (1, 1), (2, 0) and (0, 2) from the line is equal to zero. The line L will always pass through
A. (1, 1)

B. (2, 1)

C. (1, 2)

D. none of these


Answer:

Let ax + by + c = 0 be the variable line. It is given that the algebraic sum of the distances
of the points (1, 1), (2, 0) and (0, 2) from the line is equal to zero.



⇒ 3a + 3b + 3c = 0


⇒ a + b + c = 0


Substituting c = – a – b in ax + by + c = 0, we get:


ax + by – a – b = 0


⇒ a(x – 1) + b(y – 1) = 0



This line is of the form L1 + λL2 = 0, which passes through the intersection of L1 = 0 and L2 = 0, i.e. x – 1 = 0 and y – 1 = 0.


⇒ x = 1, y = 1


Question 2.

The acute angle between the medians drawn from the acute of a right angled isosceles triangle is
A.

B.

C.

D.


Answer:

Let the coordinates of the right-angled isosceles triangle be O(0, 0), A(a, 0) and B(0, a).



Here, BD and AE are the medians drawn from the acute angles B and A, respectively.


∴ Slope of BD = m1


Slope of AE = m2


Let θ be the angle between BD and AE.


tan θ


⇒ cos θ


⇒ cos θ =


⇒ θ


Hence, the acute angle between the medians is


Question 3.

The distance between the orthocenter and circumcentre of the triangle with vertices (1, 2) (2, 1) and is
A. 0

B.

C.

D. none of these


Answer:

Let A(1, 2), B(2, 1) and Cbe the given points.


∴AB


BC


AC


Thus, ABC is an equilateral triangle.


We know that the orthocentre and the circumcentre of an equilateral triangle are same.


So, the distance between the orthocentre and the circumcentre of the triangle
with vertices (1, 2), (2, 1) andis 0.


Question 4.

The equation of the straight line which passes through the point (-4, 3) such that the portion of the line between the axes is divided internally by the point in the ratio 5 : 3 is
A. 9x – 20y + 96 = 0

B. 9x + 20y = 24

C. 20x + 9y + 53 = 0

D. none of these


Answer:

Let the required line intersects the coordinate axis at (a, 0) and (0, b).



The point (− 4, 3) divides the required line in the ratio 5 : 3


∴ -4and


⇒ aand b


Hence, the equation of the required line is given below:




⇒ -9x + 20y = 96


⇒ 9x – 20y + 96 = 0


Question 5.

Which point which divides the join of (1, 2) and (3, 4) externally in the ratio of 1 : 1.
A. lies in the III quadrant

B. lies in the II quadrant

C. lies in the I quadrant

D. cannot be found


Answer:

The point which divides the join of (1, 2) and (3, 4) externally in the ratio 1 :1 is


which is not defined .


Therefore, it is not possible to externally divide the line joining two points in the ratio 1:1


Question 6.

A line passes through the point (2, 2) and is perpendicular to the line 3x + y = 3. Its y-intercepts is
A. 1/3

B. 2/3

C. 1

D. 4/3


Answer:

The equation of the line perpendicular to 3x + y = 3 is given below:


x – 3y + λ = 0


This line passes through (2, 2).


2 – 6 + λ = 0


⇒ λ = 4


So, the equation of the line will be


x – 3y + 4 = 0


⇒ y = 13x + 43


Hence, the y-intercept is .


Question 7.

If the lines ax + 12y + 1 = 0, bx + 13y + 1 = 0 and cx + 14y – 1 = 0 are concurrent, then a, b, c are in
A. H.P.

B. G.P.

C. A.P.

D. none of these


Answer:

The given lines are


ax + 12y + 1 = 0 … (1)


bx + 13y + 1 = 0 … (2)


cx + 14y + 1 = 0 … (3)


It is given that (1), (2) and (3) are concurrent.



⇒ a(13 – 14) – 12(b – c) + 14b – 13c = 0


⇒ -a – 12b + 12c + 14b – 13c = 0


⇒ -a + 2b – c = 0


⇒ 2b = a + c


Hence, a, b and c are in AP.


Question 8.

The number of real values of λ for which the lines x – 2y + 3 = 0, λx + 3y + 1 = 0 and 4x – λy + 2 = 0 are concurrent is
A. 0

B. 1

C. 2

D. infinite


Answer:

The given lines are


x − 2y + 3 = 0 … (1)


λx + 3y + 1 = 0 … (2)


4x − λy + 2 = 0 … (3)


It is given that (1), (2) and (3) are concurrent.



⇒ (6 + λ) + 2(2λ – 4) + 3(-λ2 – 12) = 0


⇒ 6 + λ + 4λ – 8 – 3λ2 – 36 = 0


⇒ 5λ – 3λ2 – 38 = 0


⇒ 3λ2 – 5λ + 38 = 0


The discriminant of this equation is 25-4 × 3 × 38 = -431


Hence, there is no real value of λ for which the lines x − 2y + 3 = 0, λx + 3y + 1 = 0 and 4x − λy + 2 = 0 are concurrent.


Question 9.

The equations of the sides AB, BC and CA of ΔABC are y – x = 2, x + 2y = 1 and 3x + y + 5 = 0 respectively. The equation of the altitude through B is
A. x – 3y + 1 0

B. x – 3y + 4 = 0

C. 3x – y + 2 = 0

D. none of these


Answer:

The equation of the sides AB, BC and CA of ∆ABC are y − x = 2, x + 2y = 1 and 3x + y + 5 = 0, respectively.


Solving the equations of AB and BC, i.e. y − x = 2 and x + 2y = 1, we get:


x = − 1, y = 1


So, the coordinates of B are (− 1, 1).


The altitude through B is perpendicular to AC.


∴ Slope of AC = -3


Thus, slope of the altitude through B is 13.


Equation of the required altitude is given below:


y – 1 = 13x + 1


⇒ x – 3y + 4 = 0


Question 10.

If p1 and p2 are the lengths of the perpendiculars form the origin upon the lines x sec θ + y cosec θ = a and x cos θ – y sin θ = a cos 2 θ respectively, then
A. 4p12 + p22 = a2

B. p12 + 4p22 = a2

C. p12 + p22 = a2

D. none of these


Answer:

The given lines are


x sec θ + y cosec θ = a … (1)


x cos θ − y sin θ = a cos 2 θ … (2)


p1 and p2 are the perpendiculars from the origin upon the lines (1) and (2), respectively.





⇒ 4p12 + p22 = a2 (sin2 2θ + cos2 2 θ) = a2


Question 11.

Area of the triangle formed by the points ((a + 3)(a + 4), a + 3), ((a + 2)(a + 3), (a + 2)) and ((a + 1)(a + 2), (a + 1)) is
A. 25a2

B. 5a2

C. 24a2

D. none of these


Answer:

The given points are (a + 3) (a + 4), a + 3, (a + 2) (a + 3), (a + 2) and (a + 1) (a + 2), (a + 1).


Let A be the area of the triangle formed by these points.


Then, A






Question 12.

If a + b + c = 0, then the family of lines 3ax + by + 2c = 0 pass through fixed point
A. (2. 2/3)

B. (2/3, 2)

C. (-2, 2/3)

D. none of these


Answer:

Given:


a + b + c = 0


Substituting c = − a − b in 3ax + by + 2c = 0, we get:


3ax + by – 2a – 2b = 0


⇒ a (3x – 2) + b (y – 2) = 0



This line is of the form L1 + λL2 = 0,


which passes through the intersection of the lines L1 and L2, i.e. 3x – 2 = 0 and y – 2 = 0 .


Solving 3 x – 2 = 0 and y – 2 = 0, we get:



Hence, the required fixed point is


Question 13.

The line segment joining the points (-3, -4) and (1, -2) is divided by y-axis in the ratio
A. 1 : 3

B. 2 : 3

C. 3 : 1

D. 3 : 2


Answer:

Let the points (− 3, − 4) and (1, − 2) be divided by y-axis at (0, t) in the ratio m:n.





Question 14.

The area of a triangle with vertices at (-4, -1), (1, 2) and (4, -3) is
A. 17

B. 16

C. 15

D. none of these


Answer:

Let A be the area of the triangle formed by the points (− 4, − 1), (1, 2) and (4, − 3).




⇒ A = 17


Question 15.

The line segment joining the points (1, 2) and (-2, 1) is divided by the line 3x + 4y = 7 in the ratio
A. 3 : 4

B. 4 : 3

C. 9 : 4

D. 4 : 9


Answer:

Let the line segment joining the points (1, 2) and (− 2, 1) be divided by the line 3x + 4y = 7 in the ratio m:n.


Then, the coordinates of this point will be that lie on the line 3x + 4y = 7



⇒ - 2 + 11 n = 7 m + 7 n


⇒ - 9 m = - 4 n


⇒ m: n = 4 : 9


Question 16.

If the point (5, 2) bisects the intercept of a line between the axes, then its equation is
A. 5x + 2y = 20

B. 2x + 5y = 20

C. 5x – 2y = 20

D. 2x – 5y = 20


Answer:

Let the equation of the line be


The coordinates of the intersection of this line with the coordinate axes are (a, 0) and (0, b).


The midpoint of (a, 0) and (0, b) is,


According to the question:




⇒ a = 10, b = 4


The equation of the required line is given below:



⇒ 2x + 5y = 20


Question 17.

A(6, 3), B(-3, 5), C(4, -2) and (x, 3x) are four points. If ΔDBC : ΔABC = 1 : 2, then x is equal to
A. 11/8

B. 8/11

C. 3

D. none of these


Answer:

The area of a triangle with vertices D (x, 3x), B (− 3, 5) and C (4, − 2) is given below:


Area of ∆DBC


⇒ Area of ∆DBC = 14x – 7 sq units


Similarly, the area of a triangle with vertices A (6, 3), B (− 3, 5) and C (4, − 2) is given below:




Given:


∆DBC: ∆ABC = 1: 2



⇒ 8x – 4 = 7



Question 18.

If p be the length of the perpendicular from the origin on the line x/a + y/b = 1, then
A. p2 = a2 + b2

B.

C.

D. none of these


Answer:

It is given that p is the length of the perpendicular from the origin on the line




Squaring both sides



Question 19.

If equation of the line passing through (1, 5) and perpendicular to the line 3x – 5y + 7 = 0 is
A. 5x + 3y – 20 = 0

B. 3x – 5y + 7 = 0

C. 3x – 5y + 6 = 0

D. none of these


Answer:

A line perpendicular to 3x − 5y + 7 = 0 is given by


5x + 3y + λ = 0


This line passes through (1, 5).


5 + 15 + λ = 0


⇒ λ = – 20


Therefore, the equation of the required line is 5x + 3y – 20 = 0


Question 20.

The figure formed by the lines ax ± by ± c = 0 is
A. a rectangle

B. a square

C. a rhombus

D. none of these


Answer:

The given lines can be written separately in the following manner:


ax + by + c = 0 … (1)


ax + by − c = 0 … (2)


ax − by − c = 0 … (3)


ax − by − c = 0 … (4)


Graph of the given lines is given below:


Diagram:



Clearly, AB = BC = CD = DA


Thus, the region formed by the given lines is ABCD, which is a rhombus


Question 21.

Two vertices of a triangle are (-2, -1) and (3, 2) and third vertex lies on the line x + y = 5. If the area of the triangle is 4 square units, then the third vertex is
A. (0, 5) or, (4, 1)

B. (5, 0) or, (1, 4)

C. (5, 0) or, (4, 1)

D. (0, 5) or, (1, 4)


Answer:

Let (h, k) be the third vertex of the triangle.


It is given that the area of the triangle with vertices (h, k), (− 2, − 1) and (3, 2) is 4 square units.



⇒ 3h – 5k + 1 = ± 8


Taking positive sign, we get,


3h – 5k + 1 = 8


3h – 5k – 7 = 0 … (1)


Taking negative sign, we get,


3h – 5k + 9 = 0 … (2)


The vertex (h, k) lies on the line x + y = 5.


H + k – 5 = 0 … (3)


On solving (1) and (3), we find (4, 1) to be the coordinates of the third vertex.


Similarly, on solving (2) and (3), we find (2, 3) to be the coordinates of the third vertex.


Question 22.

The inclination of the straight line passing through the point (-3, 6) and the mid-point of the line joining the point (4, -5) and (-2, 9) is
A.

B.

C.

D.


Answer:

The midpoint of the line joining the points (4, − 5) and (− 2, 9) is (1, 2).


Let θ be the inclination of the straight line passing through the points (− 3, 6) and (1, 2).


Then



Question 23.

Distance between the lines 5x + 3y – 7 = 0 and 15x + 9y + 14 = 0 is
A.

B.

C.

D.


Answer:

The given lines can be written as


5x + 3y − 7 = 0 … (1)


……(2)


Let d be the distance between the lines 5x + 3y − 7 = 0 and 15x + 9y + 14 = 0


Then, d



Question 24.

The angle between the lines 2x – y + 3 = 0 and x + 2y + 3 = 0 is
A. 90°

B. 60°

C. 45°

D. 30°


Answer:

Let m1 and m2 be the slope of the lines 2x − y + 3 = 0 and x + 2y + 3 = 0, respectively.


Let θ be the angle between them.


Here, m1 = 2 and m2


∵ m1m2 = − 1


Therefore, the angle between the given lines is 90°.


Question 25.

The value of λ for which the lines 3x + 4y = 5, 5x + 4y = 4 and λx + 4y = 6 meet at a point is
A. 2

B. 1

C. 4

D. 3


Answer:

It is given that the lines 3x + 4y = 5, 5x + 4y = 4 and λx + 4y = 6 meet at a point.


In other words, the given lines are concurrent.



⇒ 3(- 24 + 16) – 4 (- 30 + 4λ) – 5 (20 – 4 λ) = 0


⇒ - 24 + 120 – 16 λ – 100 + 20 λ = 0


⇒ 4 λ = 4


⇒ λ = 1


Question 26.

Three vertices of a parallelogram taken in order are (-1, -6), (2, -5) and (7, 2). The fourth vertex is
A. (1, 4)

B. (4, 1)

C. (1, 1)

D. (4, 4)


Answer:

Let A (− 1, − 6), B(2, − 5) and C(7, 2) be the given vertex.


Let D(h, k) be the fourth vertex.


The midpoints of AC and BD are and respectively.


We know that the diagonals of a parallelogram bisect each other .



⇒ h = 4 and k = 1


Question 27.

The centroid of a triangle is (2, 7) and two of its vertices are (4, 80 and (-2, 6). The third vertex is
A. (0, 0)

B. (4, 7)

C. (7, 4)

D. (7, 7)


Answer:

Let A(4, 8) and B(− 2, 6) be the given vertex. Let C (h, k) be the third vertex.


The centroid of △ABC is


It is given that the centroid of triangle ABC is (2, 7).



⇒ h = 4, k = 7


Thus, the third vertex is (4, 7).


Question 28.

If the lines x + q = 0, y – 2 = 0 and 3x + 2y + 5 + 0 are concurrent, then the value of q will be
A. 1

B. 2

C. 3

D. 5


Answer:

The lines x + q = 0, y − 2 = 0 and 3x + 2y + 5 = 0 are concurrent.




⇒ 3q = 9


⇒ q = 3


Question 29.

The medians AD and BE of a triangle with vertices A(0, b), B(0, 0) and C(a, 0) are perpendicular to each other, if
A.

B.

C. ab = 1

D.


Answer:

The midpoints of BC and AC are


Slope of AD


Slope of BE


It is given that the medians are perpendicular to each other.




Question 30.

The equation of the line with slope -3/2 and which is concurrent with the lines 4x + 3y – 7 = 0 and 8x + 5y – 1 = 0 is
A. 3x + 2y – 63 = 0

B. 3x + 2y – 2 = 0

C. 2y – 3x – 2 = 0

D. none of these


Answer:

Given:


4x + 3y − 7 = 0 … (1)


8x + 5y − 1 = 0 … (2)


The equation of the line with slope – 3/2 is given below:


y = - 32x + c


⇒ 32x + y - c = 0 … (3)


The lines (1), (2) and (3) are concurrent.






⇒ 8c = 8


⇒ c = 1


On substituting c = 1 in , we get:



⇒ 3x + 2y – 2 = 0


Question 31.

The vertices of a triangle are (6, 0), (0, 6) and (6, 6). The distance between its circumcentre and centroid is
A.

B. 2

C.

D. 1


Answer:

Let A(0, 6), B(6, 0) and C(6, 6) be the vertices of the given triangle.


Diagram:



Coordinates of N


= (6, 3)


Coordinates of P = (3, 6)


Equation of MN is y = 3


Equation of MP is x = 3


As we know that circumcentre of a triangle is the intersection of the perpendicular bisectors of any two sides .Therefore, coordinates of circumcentre is (3,3)


Thus, the coordinates of the circumcentre are (3, 3) and the centroid of the triangle is (4,4).


Let d be the distance between the circumcentre and the centroid.



Question 32.

A point equidistant from the line 4x + 3y + 10 = 0, 5x – 12y + 26 = 0 and 7x + 24y – 50 = 0 is
A. (1, -1)

B. (1, 1)

C. (0, 0)

D. (0, 1)


Answer:

Given equations are AB 4x + 3y + 10 = 0


Normalizing AB, we get


= > 4x + 3y + 10 = 0


Dividing by 5, we get


……(1)


Consider BC 5x - 12y + 26 = 0


Normalizing BC we get,


= > ……(2)


Consider AC 7x + 24y - 50 = 0


Normalizing AC we get


= > ……(3)


Adding (1) + (3), we get Angular bisector of A: ……(4)


Adding (2) + (3), we get Angular bisector of C: = 0 ……(5)


Finding point of intersection of lines (4) and (5), we get I(0, 0) which is the


Incenter of the given triangle which is the point equidistant from its sides of a triangle.


Question 33.

The ratio in which the line 3x + 4y + 2 = 0 divides the distance between the lines 3x + 4y + 5 = 0 and 3x + 4y – 5 = 0 is
A. 1 : 2

B. 3 : 7

C. 2 : 3

D. 2 : 5


Answer:

The distance between two parallel line 3x + 4y + 5 = 0 and 3x + 4y + 2 = 0 is



The distance between two parallel line 3x + 4y + 2 = 0 and 3x + 4y - 5 = 0 is



Thus required ratio is


Question 34.

The coordinates of the foot of the perpendicular from the point (2, 3) on the line x + y – 11 = 0 are
A. (-6, 5)

B. (3, 4)

C. (0, 0)

D. (6, 5)


Answer:

Let the coordinate of the foot of perpendicular from the point (2, 3) on the line x + y – 11 = 0 be (x, y)


Now, the slope of the perpendicular = -1


The equation of perpendicular is given by


y – 3 = 1(x – 2)


= > x – y + 1 = 0


Solving x + y – 11 = 0 and x – y + 1 = 0, we get


ð x = 5 and y = 6


Question 35.

The reflection of the point (4, -13) about the line 5x + y + 6 = 0 is
A. (-1, -14)

B. (3, 4)

C. (0, 0)

D. (1, 2)


Answer:

Given point (4, -13)


Line is 5x + y + 6 = 0 ……(i)


Let (h, k) be the image of A w.r.t (i)


We know that P(h, k) be the image of A(x1, y1) w.r.t ax + by + c = 0


Then,




, k + 13


h = -1, k = -14


Image of(4, -13) is (-1, -14).



Exercise 23.3
Question 1.

Find the equation of a line making an angle of 150° with the x–axis and cutting off an intercept 2 from y–axis.


Answer:

A line which makes an angle of 150o with the x–axis and cutting off an intercept at 2.

To Find: The equation of that line.


Formula used: The equation of a line is y = mx + c


Explanation: Here, angle, θ = 150o


SO, The slope of the line, m = tan θ


m = tan 150o


m =


Coordinate of y–intercept is (0, 2)


The required equation of the line is y = mx + c





Hence, The equation of line is .



Question 2.

Find the equation of a straight line:

(i) with slope 2 and y – intercept 3;

(ii) with slope – 1/ 3 and y – intercept – 4.

(iii) with slope – 2 and intersecting the x–axis at a distance of 3 units to the left of origin.


Answer:

(i) Here, The slope is 2 and the coordinates are (0, 3)

Now, The required equation of line is


y = mx + c


y = 2x + 3


(ii) Here, The slope is – 1/3 and the coordinates are (0, – 4)


Now, The required equation of line is


y = mx + c


y = x – 4


3y + x = – 12


(iii) Here, The slope is – 2 and the coordinates are (– 3, 0)


Now, The required equation of line is y – y1 = m (x – x1)


y – 0 = – 2(x + 3)


y = – 2x – 6


2x + y + 6 = 0



Question 3.

Find the equations of the bisectors of the angles between the coordinate axes.


Answer:

To Find: Equations of bisectors of the angles between coordinate axes.


Formula Used: The equation of line is y = mx + c


Diagram:



Explanation:


Co–ordinate axes make an angle of 90˚ with each other.


So the bisector of angles between co–ordinate axes will subtend


Now, we can see that there are two bisectors.


Angles subtended from x–axis are: 90˚ and 135˚


And there is no intercept, c = 0


Equations are:


y = tan45˚x and y = tan135˚x


y = x and y = –x


Hence, the equations of bisectors of angle between coordinate axis are y = x and y = –x



Question 4.

Find the equation of a line which makes an angle of tan – 1 (3) with the x–axis and cuts off an intercept of 4 units on the negative direction of y–axis.


Answer:

Given: The equation which makes an angle of tan – 1(3) with the x–axis and cuts off an intercept of 4 units on the negative direction of y–axis.

To Find: The equation of the line?


The formula used: The equation of the line is y = mx + c


Explanation: Here, angle θ = tan – 1(3)


So, tan θ = 3


The slope of the line is, m = 3


And, Intercept in the negative direction of y–axis is (0, -4)


Now, The required equation of the line is y = mx + c


y = 3x – 4


Hence, The equation of the line is y = 3x – 4.


Question 5.

Find the equation of a line that has y – intercept – 4 and is parallel to the line joining (2, – 5) and (1, 2).


Answer:

Given, A line segment joining (2, – 5) and (1, 2) if it cuts off an intercept – 4 from y–axis.

To Find: The equation of that line.


Formula used: The equation of line is y = mx + C


Explanation: Here, The required equation of line is y = mx + c


Now, c = – 4 (Given)


Slope of line joining (x1 – x2) and (y1 – y2) ,


So, Slope of line joining (2, – 5) and (1, 2),


Therefore, m = – 7


Now, The equation of line is y = mx + c


y = –7x – 4


y + 7x + 4 = 0


Hence, The equation of line is y + 7x + 4 = 0.



Question 6.

Find the equation of a line which is perpendicular to the line joining (4, 2) and (3 5) and cuts off an intercept of length 3 on y – axis.


Answer:

Given, A line segment joining (4, 2) and (3, 5) if it cuts off an intercept 3 from y–axis.

To Find: The equation of that line.


Formula used: The equation of line is y = mx + C


Explanation: Here, The required equation of line is y = mx + c


Now, c = 3 (Given)


Let m be slope of given line = – 1


Slope of line joining (x1 – x2) and (y1 – y2) ,


So, Slope of line joining (4, 2) and (3, 5),


Therefore,


Now, The equation of line is y = mx + c



x – 3y + 9 = 0


Hence, The equation of line is 2y + 5x + 6 = 0.



Question 7.

Find the equation of the perpendicular to the line segment joining (4, 3) and (– 1 1) if it cuts off an intercept – 3 from y – axis.


Answer:

Given, A line segment joining (4, 3) and (– 1, 1) if it cuts off an intercept – 3 from y–axis.

To Find: The equation of that line.


Formula used: The equation of line is y = mx + C


Explanation: Here, The required equation of line is y = mx + c


Now, c = – 3 (Given)


Let m be slope of given line = – 1


Slope of line joining (x1 – x2) and (y1 – y2) ,


So, Slope of line joining (4, 3) and (– 1, 1) ,


Therefore,


Now, The equation of the line is y = mx + c




2y + 5x + 6 = 0


Hence, The equation of line is 2y + 5x + 6 = 0.



Question 8.

Find the equation of the straight line intersecting y – axis at a distance of 2 units above the origin and making an angle of 300 with the positive direction of the x–axis.


Answer:

Given, A line which intersects at y–axis at a distance of 2 units and makes an angle of 30o with the positive direction of x–axis.

To Find: The equation of that line.


Formula used: The equation of line is [y – y1 = m(x – x1)]


Explanation: Here, Angle = 30o (Given)


So, The slope of the line, m = tan θ


m = tan 30o



Now, The coordinates are (x1, y1) = (0, 2)


The equation of line = y – y1 = m(x – x1)


y – 2 = (x – 0)




Hence, The equation of line is .




Exercise 23.4
Question 1.

Find the equation of the straight line passing through the point (6, 2) and having slope – 3.


Answer:

Given, A straight line passing through the point (6,2) and the slope is – 3

To Find: The equation of the straight line.


Formula used: The equation of line is [y – y1 = m(x – x1)]


Explanation: Here, The line is passing through (6,2)


The slope of line, m = – 3 (Given)


Coordinates of line are (x1,y1) = (6,2)


The equation of line = y – y1 = m(x – x1)


By putting the values, we get


y – 2 = – 3(x – 6)


y – 2 = – 3x + 18


y + 3x – 20 = 0


Hence, The equation of line is y + 3x – 20 = 0



Question 2.

Find the equation of the straight line passing through ( – 2, 3) and indicated at an angle of 45° with the x – axis.


Answer:

A line which is passing through ( – 2,3), the angle is 45o.

To Find: The equation of a straight line.


Formula used: The equation of line is [y – y1 = m(x – x1)]


Explanation: Here, angle, θ = 45o


SO, The slope of the line, m = tan θ


m = tan 45o


m = 1


The line passing through (x1,y1) = ( – 2,3)


The required equation of line is y – y1 = m(x – x1)


y – 3 = 1(x – ( – 2))


y – 3 = x + 2


x – y + 5 = 0


Hence, The equation of line is x – y + 5 = 0



Question 3.

Find the equation of the line passing through (0, 0) with slope m


Answer:

Given, A straight line passing through the point (0,0) and slope is m.

To Find: The equation of the straight line.


Formula used: The equation of line is [y – y1 = m(x – x1)]


Explanation: Here, The line is passing through (0,0)


The slope of line, m = m (Given)


Coordinates of line are (x1,y1) = (0,0)


The equation of line = y – y1 = m(x – x1)


By putting the values, we get


y – 0 = m(x – 0)


y = mx


Hence, The equation of line is y = mx.



Question 4.

Find the equation of the line passing through (2, 2) and inclined with x – axis at an angle of 750.


Answer:

A line which is passing through (2,2√3), the angle is 75o.

To Find: The equation of a straight line.


Formula used: The equation of line is [y – y1 = m(x – x1)]


Explanation: Here, angle, θ = 75o


SO, The slope of the line, m = tan θ


m = tan 75o


m = 3.73 = 2 +


The line passing through (x1,y1) = (2,2√3)


The required equation of the line is y – y1 = m(x – x1)


y – 2√3 = 2 + √3 (x – 2)


y – 2√3 = (2 + √3)x – 7.46


(2 + √3)x – y – 4 = 0


Hence, The equation of the line is (2 + √3)x – y – 4 = 0



Question 5.

Find the equation of the straight line which passes through the point (1,2) and makes such an angle with the positive direction of x – axis whose sine is .


Answer:

A line which is passing through (1,2)

To Find: The equation of a straight line.


Formula used: The equation of line is [y – y1 = m(x – x1)]


Explanation: Here,


We know,


According to Pythagoras theorem,


(Hypotenuse)2 = (Base)2 + (Perpendicular)2


(5)2 = (Base)2 + (3)2


(Base) =


(Base)2 =


Base = 4


Hence,


SO, The slope of the line, m = tan θ


m =


The line passing through (x1,y1) = (1,2)


The required equation of line is y – y1 = m(x – x1)


y – 2 = (x – 1)


4y – 8 = 3x – 3


3x – 4y + 5 = 0


Hence, The equation of line is 3x – 4y + 5 = 0



Question 6.

Find the equation of the straight line passing through (3, – 2) and making an angle of 60° with the positive direction of y – axis.


Answer:

A line which is passing through (3, – 2), the angle is 60o with the positive direction of the y – axis.

To Find: The equation of a straight line.


Formula used: The equation of line is [y – y1 = m(x – x1)]


Explanation: Here, angle, θ = 60o with the y – axis. So, it makes 300 with the positive direction of the x – axis.


SO, The slope of the line, m = tan θ


m = tan 30o


m =


The line passing through (x1,y1) = (3, – 2)


The required equation of line is y – y1 = m(x – x1)


y – ( – 2) = (x – 3)




Hence, The equation of the line is



Question 7.

Find the lines through the point (0, 2) making angles and with the x–axis. Also, find the lines parallel to them cutting the y–axis at a distance of 2 units below the origin.


Answer:

We know that equation of line having angle θ from x–axis and passing through (x1, y1) is given by,



Therefore,


Equation of first line,


(y – 2)


y – 2 = √3x


y – √3x – 2 = 0


The equation of line parallel to this line and passing through (0, –2),


(y + 2) = √3x


y – √3x + 2 = 0


Equation of second line,


(y – 2)


y – 2 = –√3x


y + √3x – 2 = 0


The equation of line parallel to this line and passing through (0, –2),


(y + 2) = –√3x


y + √3x + 2 = 0



Question 8.

Find the equations of the straight lines which cut off an intercept 5 from the y – axis and are equally inclined to the axes.


Answer:

A line which cut off an intercept 5 from the y – axis are equally to axes.

To Find: Find the equation


The formula used: The equation of the line is y = mx + c


Explanation: If a line is equally inclined to the axis, then


Angle θ = 45o and θ = (180 – 45) = 135o


Then, The slope of the line, m = tan θ


m = 1


Since, the intercept is 5, C = 5


Now, The equation of the line is y = mx + c


y = 1x + 5


y – x = 5


Hence, The equation of the line is y – x = 5.



Question 9.

Find the equation of the line which intercepts a length 2 on the positive direction of the x – axis and is inclined at an angle of 135° with the positive direction of the y – axis.


Answer:

Given, A line which cut off an intercept a length w from the x – axis.

To Find: Find the equation


Formula used: The equation of line is [(y – y1) = m(x – x1)]


Explanation: If a line is inclined at angle 135o on y – axis, then angle on the x – axis is


Angle θ = 135o and θ = (180 – 135) = 45o


Then, The slope of the line, m = tan θ


m = 1


Since the line passes through the point (2,0)


Now, The equation of line is (y – y1) = m(x – x1)


(y – 0) = 1(x – 2)


y = x – 2


x – y – 2 = 0


Hence, The equation of the line is x – y – 2 = 0.



Question 10.

Find the equation of the straight line which divides the join of the points (2, 3) and ( – 5, 8) in the ratio 3 : 4 and is also perpendicular to it.


Answer:

Given, A line which divides the join of the points (2,3) and ( – 5,8) in the ratio 3:4

To Find : The equation of the line.


Explanation: The coordinates of the point which divides the join of the points (2,3) and ( – 5,8) in the ratio 3:4 is given by (x,y).


Coordinate of x when line divides in ratio m:n


x =


x =


Coordinate of y when line divides in ratio m:n =


y =


y =


The slope of the line with two points is, m =


Now, The slope of joining the points (2,3) and ( – 5,8) =


m =


The equation of the line is


y


y





35y – 180 = 49x + 63


49x – 35y + 229 = 0


Hence, The equation of line is 49x – 35y + 229 = 0



Question 11.

Prove that the perpendicular drawn from the point (4, 1) on the join of (2, – 1) and (6 5) divides it in the ratio 5:8.


Answer:

Given, A perpendicular drawn from the point (4,1) on the join of (2, – 1) and (6,5)

To Prove: The perpendicular divides the line in the ratio 5:8.



Explanation: Let us Assume,The perpendicular drawn from point C(4,1) on a line joining A(2, – 1) and B(6,5) divide in the ratio k:1 at the point R.


Now, The coordinates of R are:


By using Sectional Formula, (x,y) =


R(x,y) = – – – (1)


The slope of the line with two points is, m =


The slope of AB =


The slope of CR =


And, PR is perpendicular to AB


Since, (Slope of CR)×(Slope of AB) = – 1






3(4k – 2) = – 2(2k – 2)


12k – 6 = – 4k + 4


16k = 10


K


So, The ratio is 5:8


Hence, R divides AB in the ratio 5:8.



Question 12.

Find the equations to the altitudes of the triangle whose angular points are A (2, – 2), B(1, 1), and C ( – 1, 0).


Answer:

A triangle is given with three angular points A (2, – 2), B(1, 1), and C ( – 1, 0)

To Find: Find the equation.


Formula Used: The equation of line is (y – y1) = m(x – x1)



Explanation: Here, AD, BE and CF are the three altitudes of the triangle.


Now,


We know, The slope of the line with two points is, m =


So, The slope of BC =


The slope of AC =


The slope of AB =


and, The product of two slopes of the perpendicular line is always – 1


So, (slope of AB) × (slope of CF) = – 1


The slope of CF =


(slope of BE)×(slope of AC) = – 1


The slope of BE =


(slope of AD)×(slope of BC) = – 1


The slope of AD =


So, The equation of line is (y – y1) = m(x – x1)


The equation of Line AD is


y – ( – 2) = – 2(x – 2)


y + 2 = – 2x + 2


2x + y – 2 = 0


The equation of Line BE is



2y – 2 = 3x – 3


2y – 3x + 1 = 0


The equation of Line CF is



x – 3y + 1 = 0


Hence, The equation of the three equation is calculated.



Question 13.

Find the equation of the right bisector of the line segment joining the points (3, 4) and ( – 1, 2).


Answer:

Given, The line segment joining the points (3,4) and ( – 1,2)

To Find: Find the equation of the line


Formula used: The equation of line is (y – y1) = m(x – x1)


Explanation: Here, The right bisector PQ of AB at C and is perpendicular to AB


Now, The coordinate of the mid – points =


The coordinates of point C are = = (1,3)


And, The slope of PQ =


The slope of PQ, m =


SO, The slope of PQ, m = – 2


The required equation of PQ is (y – y1) = m(x – x1)


y – 3 = – 2(x – 1)


y – 3 = – 2x + 2


y + 2x = 5


Hence, The equation of line is y + 2x = 5



Question 14.

Find the equation of the line passing through the point ( – 3, 5) and perpendicular to the line joining (2, 5) and ( – 3, 6).


Answer:

Given, A line which passes through the point ( – 3,5) and perpendicular to the line joining (2,5) and ( – 3,6)

To Find: Find the equation


Formula Used: The equation of line is (y – y1) = m(x – x1)


Explanation: Here, The line passes through the point ( – 3,5 ), Given


So, The coordinate (x1,y1) = ( – 3,5)


Now, The line is perpendicular to the line joining (2,5) and ( – 3,6),


We know, The slope of the line with two points is, m =


So, the slope of line joining (2, 5 ) and ( – 3,6) is =


m =


Therefore, The slope of the required line is, m =


So, m =


m = 5


Now, The equation of straight line is (y – y1) = m(x – x1)


y – 5 = 5 (x – ( – 3)


y – 5 = 5x + 15


5x – y + 20 = 0


Hence, The equation of line is 5x – y + 20 = 0



Question 15.

Find the equation of the right bisector of the line segment joining the points A(1, 0) and B(2, 3).


Answer:

Given, The line segment joining the points (1,0) and (2,3)

To Find: Find the equation of line


Formula used: The equation of line is (y – y1) = m(x – x1)


Explanation: Here, The right bisector PQ of AB at C and is perpendicular to AB


So, The slope of the line with two points is, m


The slope of the line AB


We know, The product of two slopes of the perpendicular line is always – 1


Therefore, (slope of AB) × (slope of PQ) = – 1


Since Slope of PQ


Now, The coordinate of the mid – points


The coordinates of point C are


The required equation of PQ is (y – y1) = m(x – x1)



6y – 9 = – 2x + 3


x + 3y = 6


Hence, The equation of line is x + 3y = 6




Exercise 23.5
Question 1.

Find the equation of the straight lines passing through the following pair of points:

(0, 0) and (2, - 2)


Answer:

Given:


( x1, y1) = (0, 0), ( x2, y2) = (2, -2)


Concept Used:


The equation of the line passing through the two points ( x1, y1) and ( x2, y2).


To find:


The equation of the straight line passing through a pair of points.


Explanation:


So, the equation of the line passing through the two points (0, 0) and (2, −2) is


The formula used:



⇒ y = -x


Hence, equation of line is y = -x



Question 2.

Find the equation of the straight lines passing through the following pair of points:

(a, b) and (a + c sin α, b + c cos α)


Answer:

Given:

( x1, y1) = (a, b), ( x2, y2) = (a + c sin, b + c cos)


Concept Used:


The equation of the line passing through the two points ( x1, y1) and ( x2, y2).


To find:


The equation of the straight line passing through a pair of points.


Explanation:


So, the equation of the line passing through the two points (0, 0) and (2, −2) is


The formula used:



⇒ y – b = cot (x - a)


Hence, equation of line is y – b = cot (x - a)



Question 3.

Find the equation of the straight lines passing through the following pair of points:

(0, - a) and (b, 0)


Answer:

Given:

( x1,y1) = (0,-a), ( x2, y2) = (b,0)


Concept Used:


The equation of the line passing through the two points ( x1, y1) and ( x2, y2)


To find:


Equation of straight line passing through pair of points.


Explanation:


So, the equation of the line passing through the two points is


The formula used:



⇒ ax – by = c


Hence, the equation of line is ax – by = c



Question 4.

Find the equation of the straight lines passing through the following pair of points:

(a, b) and (a + b, a - b)


Answer:

Given: ( x1,y1) = (a,b), ( x2, y2) = (a+b, a-b)

Concept Used:


The equation of the line passing through the two points ( x1, y1) and ( x2, y2).


To find:


The equation of the straight line passing through a pair of points.


Explanation:


So, the equation of the line passing through the two points is


The formula used:



⇒ by – b2 = (a – 2b)x – a2 + 2ab


⇒ (a – 2b)x – by + b2 + 2ab – a2 = 0


Hence, the equation of line is (a – 2b)x – by + b2 + 2ab – a2 = 0



Question 5.

Find the equation of the straight lines passing through the following pair of points:

(at1, a/t1) and (at2, a/t2)


Answer:

Given: ( x1,y1), ( x2, y2)

Concept Used:


The equation of the line passing through the two points ( x1, y1) and ( x2, y2).


To find:


The equation of straight line passing through a pair of points.


Explanation:


So, the equation of the line passing through the two points is


The formula used:





Hence, the equation of the line is



Question 6.

Find the equation of the straight lines passing through the following pair of points:

(a cos α, a sin α) and (a cos β, a sin β)


Answer:

Given: ( x1,y1) = (a cos α, a sin α), ( x2,y2) = (a cos β, a sin β)

Concept Used:


The equation of the line passing through the two points ( x1, y1) and ( x2, y2).


To find:


The equation of straight line passing through a pair of points.


Explanation:


So, the equation of the line passing through the two points is


The formula used:







Dividing by



Hence, the equation of the line is



Question 7.

Find the equations to the sides of the triangles the coordinates of whose angular points are respectively:

(1,4), (2, - 3) and (-1, - 2)


Answer:

Given:


Points A (1, 4), B(2, -3) and C(-1, -2).


Assuming:


m1, m2, and m3 be the slope of the sides AB, BC and CA, respectively.


Concept Used:


The slope of the line passing through the two points ( x1, y1) and ( x2, y2).


The equation of the line passing through the two points ( x1, y1) and ( x2, y2).


To find:


The equation of sides of the triangle.


Explanation:


m1, m2m3


m1 = -7, m2 and m3 = 3


So, the equation of the sides AB, BC and CA are


Formula used: y – y1= m (x – x1)


y – 4 = -7 (x – 1), and y + 2 = 3(x+1)


⇒ 7x + y =11, x+ 3y +7 =0 and 3x – y +1 = 0


Hence, equation of sides are 7x + y =11, x+ 3y +7 =0 and 3x – y +1 = 0



Question 8.

Find the equations to the sides of the triangles the coordinates of whose angular points are respectively:

(0,1), (2, 0) and (-1, - 2)


Answer:

Given:


Points A (0, 1), B(2, 0) and C(-1, -2).


Assuming:


m1, m2 and m3 be the slope of the sides AB, BC and CA, respectively.


Concept Used:


The slope of the line passing through the two points ( x1, y1) and ( x2, y2).


The equation of the line passing through the two points ( x1, y1) and ( x2, y2).


To find:


The equation of sides of the triangle.


Explanation:


m1m2, m3


m1, m2 and m3= 3


So, the equation of the sides AB, BC and CA are


Formula used: y – y1= m (x – x1)


, and y + 2 = 3(x+1)


⇒ x + 2y = 2, 2x – 3y =4 and 3x – y +1 = 0


Hence, equation of sides are x + 2y = 2, 2x – 3y =4 and 3x – y +1 = 0



Question 9.

Find the equations of the medians of a triangle, the coordinates of whose vertices are (-1, 6), (-3,-9) and (5, -8).


Answer:

Given:

A (−1, 6), B (−3, −9) and C (5, −8) be the coordinates of the given triangle.


Assuming:


D, E, and F be midpoints of BC, CA and AB, respectively. So, the coordinates of D, E and F are


To find:


The equation of median of a triangle.


Explanation:



Median AD passes through A (-1, 6) and D (1, -17/2)


So, its equation is


Formula used:



4y – 24 = -29x – 29


29x + 4y + 5 = 0


Median BE passes through B (-3,-9) and E (2,-1)


So, its equation is


Formula used:



5y + 45 = 8x + 24


8x – 5y – 21=0


Median CF passes through C (5,-8) and F(-2,-3/2)


So, its equation is


Formula used:



⇒ -14y – 112 = 13x – 65


⇒ 13x + 14y + 47 = 0


Hence, the equation of line is 13x + 14y + 47 = 0



Question 10.

Find the equations to the diagonals of the rectangle the equations of whose sides are x = a, x = a', y = b and y = b'.


Answer:

Given: The rectangle formed by the lines x = a, x = a’, y = b and y = b’

Concept Used:


The equation of the line passing through the two points ( x1, y1) and ( x2, y2)


To find:


The equation of diagonal of the rectangle.


Explanation:


Clearly, the vertices of the rectangle are A(a, b), B(a’, b), C(a,’ b’) and D(a, b’) .


The diagonal passing through A (a, b) and C (a’, b’) is


Formula used:



⇒ (a’ – a)y – b(a’ – a) = (b’ – b)x – a(b’ – b)


⇒ (a’ – a) – (b’ – b)x = ba’ – ab’


And, the diagonal passing through B(a’, b) and D(a, b’) is


Formula used:



⇒ (a’ – a)y – b(a – a’) = (b’ – b)x – a’(b’ – b)


⇒ (a’ – a) – (b’ – b)x = a’b’ – ab


Hence, the equation of diagonals are (a’ – a) – (b’ – b)x = ba’ – ab’ and (a’ – a) – (b’ – b)x = a’b’ – ab



Question 11.

Find the equation of the side BC of the triangle ABC whose vertices are A (-1, -2), B (0, 1) and C (2, 0) respectively. Also, find the equation of the median through A (-1, - 2).


Answer:

Given: The vertices of triangle ABC are A (-1, -2), B(0, 1) and C(2, 0).


Concept Used:


The equation of the line passing through the two points ( x1, y1) and ( x2, y2)


To find:


Equation of side BC of triangle ABC.


The equation of median through A.


Explanation:


So, the equation of BC is


Formula used:




⇒ x + 2y – 2 = 0


Let D be the midpoint of median AD is


So,


So, the equation of the median AD is


Formula used:



⇒ 4y + 8 = 5x + 5


⇒ 5x – 4y – 3 = 0


The equation of line BC is x + 2y – 2 = 0


Hence, the equation of median is 5x – 4y – 3 = 0



Question 12.

By using the concept of the equation of a line, prove that the three points (- 2, - 2), (8, 2) and (3, 0) are collinear.


Answer:

Given: points be A (-2, 2), B (8, 2) and C(3,0).

To prove:


Points (- 2, - 2), (8, 2) and (3, 0) are collinear.


Explanation:


The equation of the line passing through A (-2,-2) and B (8, 2) is


Formula used:



⇒ 5y + 10 = 2x + 4


⇒ 2x – 5y – 6 = 0


Clearly, point C (3, 0) satisfies the equation 2x – 5y – 6 = 0


Hence Proved, the given points are collinear.



Question 13.

Prove that the line y - x + 2 = 0 divides the join of points (3,-1) and (8, 9) in the ratio 2:3


Answer:

Assuming:

y – x +2 = 0 divides the line joining the points (3, -1) and (8, 9) at the point P in the ratio k : 1


To prove:


Line y - x + 2 = 0 divides the join of points (3,-1) and (8, 9) in the ratio 2:3


Explanation:


P


P lies on the y – x +2 = 0


Therefore,



⇒ -1 + 9k – 3 – 8k +2k + 2 = 0


⇒ 3k = 2


⇒ k


Hence Proved, the line y – x +2 = 0 divides the line joining the points (3, -1) and (8, 9) in the ratio 2 : 3



Question 14.

Find the equation to the straight line which bisects the distance between the points (a, b), (a', b') and also bisects the distance between the points (-a, b) and (a', - b').


Answer:

Given: points be A (a, b), B(a’, b’),C(-a, b) and D(a’, -b’)

Assuming:


P and Q be the mid points of AB and CD, respectively.


P


Q


Explanation:


The equation of the line passing through P and Q is


Formula used:




⇒ 2ay - 2b’x = ab – a’b’


Hence, the equation of the required straight line is 2ay - 2b’x = ab – a’b’



Question 15.

In what ratio is the line joining the points (2, 3) and (4, -5) divided by the line passing through the points (6, 8) and (- 3, -2).


Answer:

Given: the equation of the line joining the points (6, 8) and (-3, -2) is


To find: In what ratio line joining the points divided by a line.


Assuming: 10x – 9y +12 = 0 divide the line joining the points (2, 3) and (4, 5) at points P in the ratio k : 1


Explanation:


Formula used:



⇒ 10x – 9y +12 = 0


P


P lies on the 10x – 9y +12 = 0


Therefore,



⇒ 40k + 20 + 45k – 27 +12k + 12 = 0


⇒ 97k + 5 = 0


⇒ K


Hence, the joining the points (2, 3) and (4, 5) is divided by the line passing through the points (6, 8) and (-3, -2) in the ratio 5: 97 externally.



Question 16.

The vertices of a quadrilateral are A (-2, 6), B (1, 2), C (10, 4) and D (7, 8). Find the equations of its diagonals.


Answer:

Given: the two diagonals of the quadrilateral with vertex A (-2, 6), B(1, 2), C(10, 4) and D(7, 8) are

AC and BD.


Concept Used:


The equation of the line passing through the two points ( x1, y1) and ( x2, y2).


To find:


The equation of diagonal of the quadrilateral.


Explanation:


The equation of AC passing through A (-2, 6) and C (10, 4) is


Formula used:


y – 6 (x + 2)


⇒ x + 6y – 34 = 0


And the equation of AC passing through, B (1, 2) and D(7, 8) is


Formula used:


y – 2 (x – 1)


⇒ x – y + 1 = 0


Hence, the equation of the diagonal are x + 6y – 34 = 0 and x – y + 1 = 0



Question 17.

The length L (in centimeters) of a copper rod is a linear function of its Celsius temperature C. In an experiment if L =124.942 when C =20 and L =125.134 when C =110, express L in terms of C.


Answer:

Assuming:

C along the x-axis and L along the y-axis


Given:


Points (20, 124.942) and (110, 125.134) in CL plane.


Concept Used:


The equation of the line passing through the two points ( x1, y1) and ( x2, y2)


To find:


The equation of L in term of C.


Explanation:


L is a linear function of C, the equation of the line passing through (20, 124.942) and(110, 125.134) is


Formula used:








Hence, the equation of L in term of C is



Question 18.

The owner of a milk store finds that he can sell 980 liters milk each week at Rs. 14 per liter and 1220 liters of milk each week at Rs. 16 per liter. Assuming a linear relationship between selling price and demand, how many liters could he sell weekly at Rs. 17 per liter.


Answer:

Assuming:

x denotes the price per liter, and y denote the quality of the milk sold at this price.


Since there is a linear relationship between the price and the quality, the line representing this


Given:


Relationship passes through (14, 980) and (16, 1220).


To find:


How many liters could he sell weekly at Rs. 17 per liter.


Explanation:


So, the equation of the line passing through these points is


Formula used:



⇒ y – 980 = 120(x - 14)


⇒ 120 x - y -700 = 0


When x = 17 then we have,


120(17) – y – 700 = 0


⇒ y = 1340


Hence, the owner of the milk store can shell 1340 litres of milk at Rs. 17 per litre.



Question 19.

Find the equation of the bisector of angle A of the triangle whose vertices are A (4, 3), B (0, 0) and C (2,3).


Answer:

Given: the vertices of triangle ABC are A (4, 3), B (0, 0) and C (2, 3).


To find:


The equation of bisector of angle A.


Explanation:


Let us find the lengths of sides AB and AC.


AB = 5


AC = = 2


We know that the internal bisector AD of angle BAC divides BC in the ratio AB: AC, i.e. 5: 2


D


Thus, the equation of AD is


Formula used:




⇒ x – 3y +5 = 0


Hence, the equation of line is x – 3y +5 = 0



Question 20.

Find the equations to the straight lines which go through the origin and trisect the portion of the straight line 3 x + y = 12 which is intercepted between the axes of coordinates.


Answer:

To find:


The equation of the required line.


Assuming:


The line 3x + y = 12 intersect the x-axis and the y-axis at A and B, respectively.


y =m1x and m2x be the lines passing through the origin and trisect the line 3x + y =12 at P and Q.


Explanation:


At x = 0


0 + y =12


⇒ y =12


At y = 0


3x + 0 =12


⇒ x = 4


A (4, 0) and B (0,12)


y =m1x and m2x be the lines passing through the origin and trisect the line 3x + y =12 at P and Q.


AP = PQ = QB


Let us find the coordinates of P and Q.


P


Q


Clearly, P and Q lie on y = m1x and y = m2x, respectively.


4


⇒ m1 and m2 = 6


Hence, the required lines are y


⇒ 2y = 3x and y = 6x


Hence, the equation of line is 2y = 3x and y = 6x



Question 21.

Find the equations of the diagonals of the square formed by the lines x = 0, y = 0, x = 1 and y = 1.


Answer:

Given:

The square formed by the lines x = 0, x = 1, y = 0 and y = 1.


Concept Used:


The equation of the line passing through the two points ( x1, y1) and ( x2, y2)


To find:


The equation of diagonal of the square.


Explanation:


Clearly, the vertices of the square are A(0, 0), B(1, 0), C(1, 1) and D(0,1) .


The diagonal passing through A (0, 0) and C(1, 1) is


Formula used:



⇒ y = x


And, the diagonal passing through B(1, 0) and D(0, 1) is


Formula used:



⇒ y = -x + 1


⇒ x + y = 1


Hence, the equation of diagonals are y = x and x + y = 1.




Exercise 23.6
Question 1.

Find the equation to the straight line

cutting off intercepts 3 and 2 from the axes.


Answer:

Given:


Here, a = 3, b = 2


To find:


The equation of line cutoff intercepts from the axes.


Explanation:


So, the equation of the line is


Formula used:



⇒ 2x + 3y =6


Hence the equation of line cut off intercepts 3 and 2 from the axes is 2x + 3y = 6



Question 2.

Find the equation to the straight line

cutting off intercepts -5 and 6 from the axes.


Answer:

Given:


Here, a = -5, b=6


To find:


The equation of line cutoff intercepts from the axes.


Explanation:


So, the equation of the line is


Formula used:



⇒ 6x -5y =-30


Hence, the equation of line cut off intercepts -5 and 6 from the axes is 6x – 5y = -30



Question 3.

Find the equation of the straight line which passes through (1, -2) and cuts off equal intercepts on the axes.


Answer:

Given:


A line passing through (1, -2)


Assuming:


The equation of the line cutting equal intercepts at coordinates of length ‘ a ‘ is


Explanation:


Formula used:



⇒ x + y =a


The line x + y =a passes through (1, -2 ) So the point satisfy the equation


1 -2 =a


⇒ a = -1


Hence the equation of the line is x+ y = -1



Question 4.

Find the equation to the straight line which passes through the point (5, 6) and has intercepts on the axes

Equal in magnitude and both positive


Answer:

Given:


Here, a = b


To find:


The equation of line cutoff intercepts from the axes.


Explanation:


So, the equation of the line is


Formula used:




⇒ x + y = a


The line passes through the point (5, 6) So equation satisfy the points,


⇒ 5 + 6 = a


⇒ a = 11


Hence the equation of the line is x + y = 11



Question 5.

Find the equation to the straight line which passes through the point (5, 6) and has intercepts on the axes

Equal in magnitude but opposite in sign


Answer:

Given:


Here, b = -a


To find:


The equation of line cutoff intercepts from the axes.


Explanation:


So, the equation of the line is


Formula used:




⇒ x – y =a


The line passes through the point (5, 6) So equation satisfy the points,


⇒5 – 6 = a


⇒ a = -1


The equation of the line is x – y = -1



Question 6.

For what values of a and b the intercepts cut off on the coordinate axes by the line ax + by + 8 = 0 are equal in length but opposite in signs to those cut off by the line 2x – 3y + 6 = 0 on the axes.


Answer:

Given:


Intercepts cut off on the coordinate axes by the line ax + by +8 = 0 ……(i)


And are equal in length but opposite in sign to those cut off by the line


2x – 3y +6 = 0 ……(ii)


Explanation:


The slope of two lines are equal


The slope of the line (i) is


The slope of the line (ii) is



a


The length of the perpendicular from the origin to the line (i) is


The formula used: d


d1=


d1=


The length of the perpendicular from the origin to the line (ii) is


The formula used: d


d2=


Given: d1= d2



⇒ b = 4



Hence the value of a and b is



Question 7.

Find the equation to the straight line which cuts off equal positive intercepts on the axes and their product is 25


Answer:

Concept Used:


The equation of the line with intercepts a and b is


To find:


The equation of the line which cutoff intercepts on the axes.


Given:


Here a = b and ab = 25


Explanation:


∴ a2 = 25


⇒ a = 5 since we are to take only positive value of intercepts


Hence, the equation of the required line is


Formula used:



⇒ x + y = 5


Hence, the equation of line is x + y = 5



Question 8.

Find the equation of the line which passes through the point (-4, 3) and the portion of the line intercepted between the axes is divided internally in the ratio 5: 3 by this point.


Answer:

Concept Used:


The equation of the line with intercepts a and b is


Given:


The line intersects the axes (a,0) and (0,b).


Explanation:


So, (-4,3) divides the line segment AB and the ratio 5:3



⇒ a , b


So, the equation of the line is


⇒ 9x – 20y = -96


Hence, the equation of line is 9x – 20y = -96



Question 9.

A straight line passes through the point (α, β) and this point bisects the portion of the line intercepted between the axes. Show that the equation of the straight line is .


Answer:

Concept Used:


The equation of the line with intercepts a and b is


Given:


The line intersects the axis at A (a, 0) and B ( b, 0)


Explanation:


Here, ( α, β) is the midpoint of AB


⇒ α , β


⇒ α, β


Hence, The equation is



Question 10.

Find the equation of the line which passes through the point (3, 4) and is such that the portion of it intercepted between the axes is divided by the point in the ratio 2 : 3.


Answer:

Concept Used:


The equation of the line with intercepts a and b is


Assuming:


The line meets the coordinate axes at A and B, So the coordinates A (a, 0) and B (0, b )


AP : BP =2 : 3


Here p = (3, 4)



⇒ 3a = 15,2b = 20


⇒ a = 5, b = 10


Thus the equation of the line is


Formula used:



⇒ 2x + y = 10



Question 11.

Point R (h, k) divided line segments between the axes in the ratio 1 : 2. Find the equation of the line.


Answer:

Concept Used:


The equation of the line with intercepts a and b is


Given:


The line passes through R(h, k)


Explanation:


……(i)


The line intersects the coordinate axes at A(a, 0) and B( 0, b).


Here, AP : BP = 1 : 2



⇒a , b = 3k


Substituting, b = 3k in



⇒2kx + hy – 3hk = 0


Hence, the equation of the line is 2kx + hy – 3hk = 0



Question 12.

Find the equation of the straight line which passes through the point (-3, 8) and cuts off positive intercepts on the coordinate axes whose sum is 7


Answer:

Concept Used:


The equation of the line with intercepts a and b is


Given:


Here a + b = 7, b = 7 – a


Explanation:


The line is passing through (-3, 8).



Substituting b =7 – a, we get




⇒ a2 + 4a – 21 = 0


⇒ (a – 3)(a + 7)= 0


⇒ a = 3 ( since, a can only be positive )


Substituting a = 3 in equation (i) we get,


b = 7 – 3 = 4


Hence, the equation of the line is



Question 13.

Find the equation to the straight line which passes through the point (-4, 3) and is such that the portion of it between the axes is divided by the point in the ratio 5 : 3.


Answer:

Concept Used:

The equation of the line with intercepts a and b is


Given:


The line intersects the axes (a,0) and (0,b).


Explanation:


So, (-4,3) divides the line segment AB and the ratio 5:3



⇒ a , b


So, the equation of the line is


⇒ 9x – 20y = -96


Hence, the equation of line is 9x – 20y = -96



Question 14.

Find the equation of a line which passes through the point (22, -6) and is such that the intercept on x-axis exceeds the intercept on the y-axis by 5.


Answer:

Concept Used:


The equation of the line with intercepts a and b is


Given:


Here, a =b+5 ……..(1)


Explanation:


The line passing through the point (22, -6)


………(2)


Substituting a = b + 5 from equation (1 ) in equation (2)




(b – 5)(b – 6)=0


b = 5, 6


From equation (1)


When b = 5 then a = 10


When b = 6 then a = 11


Thus the equation of the required line is


and


Thus the equations are


x + 2y=10, 6x +11y=66


Hence, the equation of line is x + 2y=10, 6x +11y=66



Question 15.

Find the equation of the line, which passes through P(1, -7) and meets the axes at A and B respectively so that 4AP – 3BP = 0.


Answer:

Concept Used:


The equation of the line with intercepts a and b is


Assuming:


The line meets the coordinate axes at A and B, So the coordinates A (a, 0) and B (0, b )


Given:


4AP – 3BP = 0


Explanation:


⇒ AP : BP =3 : 4


Here p= (1, -7)



⇒4a = 7,3b = - 49


⇒a = , b =


Thus the equation of the line is




⇒ 28x – 3y = 49



Question 16.

Find the equation of the line passing through the point (2, 2) and cutting off intercepts on the axes whose sum is 9


Answer:

Concept Used:


The equation of the line with intercepts a and b is


Given:


Here, a+b = 9


Explanation:


⇒ b = 9 – a ……(i)


The line is passing through (2, 2).


……(ii)


From equation (i) and (ii)



⇒18 – 2a + 2a = 9a – a2


⇒a2 – 9a +18 = 0


⇒ (a – 3)(a – 6) = 0


⇒a = 3, 6


For a = 3, b = 9 – 3 = 6


For a = 6, b = 9 – 6 = 3


Thus the equation of line is


or


⇒2x + y = 6 or x + 2y = 6


Hence, the equation of line is 2x + y = 6 or x + 2y = 6



Question 17.

Find the equation of the straight line which passes through the point P(2, 6) and cuts the Coordinates axes at the point A and B respectively so that .


Answer:

Concept Used:


The equation of the line with intercepts a and b is


Assuming:


The line meets the coordinate axes at A and B, So the coordinates A (a, 0) and B (0, b )


Given:


AP : BP =2 : 3


Explanation:


Here p= (2, 6)



⇒3a = 10,2b = 30


, b = 15


Thus the equation of the line is




⇒ 9x + 2y = 30



Question 18.

Find the equations of the straight lines each of which passes through the point (3, 2) and cuts off intercepts a and b respectively on x and y-axes such that a – b = 2.


Answer:

Concept Used:

The equation of the line with intercepts a and b is


Given:


Here, a – b = 2


⇒ a = b + 2 ……(i)


Explanation:


The line is passing through (3,2).


……(ii)


From equation (i) and (ii)



⇒3b + 2b +4 = b2 + 2b


⇒b2 – 3b – 4 = 0


⇒ (b – 4)(b + 1) = 0⇒b = 4, -1


Now, from equation (i)


For b = 4, a = 4 + 2 = 6


For b = -1, b = -1 + 2 = 1


Thus the equation of line is


or


⇒x – y = 1 or 2x + 3y = 12



Question 19.

Find the equations of the straight lines which pass through the origin and trisect the portion of the straight line 2x + 3y = 6 which is intercepted between the axes.


Answer:

To find: Equations of the straight lines which pass through the origin and trisect the portion of the line which is intercepted between the axes.


Assuming:


The line 2x + 3y = 6 intercept the x-axis and the y-axis at A and B, respectively.


Explanation:


At x = 0 we have,


3y + 0 = 6


⇒ 3y = 6


⇒ y = 2


At y = 0 we have,


2x + 0 = 6


⇒x = 3


A = (3, 0) and B = (0, 2)


Let y = m1x and y = m2x pass through origin trisecting the line 2x + 3y = 6 at P and Q.


AP = PQ = QB


Let us find the coordinates of P and Q using the section formula


P


Q


Clearly, P and Q lie on y = m1x and y = m2x, respectively



and


Hence, the required lines are


y nd y


⇒ x – 3y = 0 and 4x – 3y = 0


Hence, the equation of line is x – 3y = 0 and 4x – 3y = 0



Question 20.

Find the equation of the straight line passing through the point (2, 1) and bisecting the portion of the straight line 3x – 5y = 15 lying between the axes.


Answer:

Concept Used:


The equation of a line in intercept form is


Given:


The line passes through (2, 1)


……(i)


Assuming:


The line 3x – 5y = 15 intercept the x-axis and the y-axis at A and B, respectively.


Explanation:


At x = 0 we have,


0– 5y = 15


⇒ 5y = -15


⇒ y = -3


At y = 0 we have,


3x – 0 =15


⇒ x = 5


A= (0, -3) and B = (5, 0)


The midpoint of AB is ()


Clearly, the point () lies on the line


……(ii)


Using eq(i) + eq(ii) we get,



⇒a


For a = we have,



⇒ b = 11


Therefore, the equation of the required line is:




⇒ 5x + y = 11


Hence, the equation of line is 5x + y = 11



Question 21.

Find the equation of the straight line passing through the origin and bisecting the portion of the line ax + by + c = 0 intercepted between the coordinate axes.


Answer:

Concept Used:


The equation of the line passing through the origin is y = mx


To find:


Equation of the straight line passing through the origin and bisecting the portion of a line intercepted between the coordinate axes.


Assuming:


The line ax + by + c = 0 meets the coordinate axes at A and B.


Explanation:


So, the coordinate of A and B are A() and B (0,)


Now,


The midpoint of AB is



⇒ m


Hence, the equation of the required line is


y


⇒ ax – by = 0




Exercise 23.7
Question 1.

Find the equation of a line for which

p = 5, α = 60°


Answer:

Given: p = 5, α = 60°

Concept Used:


Equation of line in normal form.


Explanation:


So, the equation of the line in normal form is


Formula Used: x cos α + y sin α = p


x cos 60° + y sin 60° = 5



⇒ x + √3y = 10


Hence, the equation of line in normal form is x + √3y = 10.



Question 2.

Find the equation of a line for which

p = 4, α = 150°


Answer:

Given: p = 4, α = 150°

Concept Used:


Equation of line in normal form.


Explanation:


So, the equation of the line in normal form is


Formula Used: x cos α + y sin α = p


x cos 150° + y sin 150° = 4


cos (180° – θ) = – cos θ , sin (180° – θ) = sin θ


⇒ x cos(180° – 30°) + y sin(180° – 30°) = 4


⇒ – x cos 30° + y sin 30° = 4



⇒ √3x – y + 8 = 0


Hence, the equation of line in normal form is √3 x – y + 8 = 0



Question 3.

Find the equation of a line for which

p = 8, α = 225°


Answer:

Given p = 8, α = 225°

Concept Used:


Equation of line in normal form.


Explanation:


So, the equation of the line in normal form is


Formula Used: x cos α + y sin α = p


x cos 225° + y sin 225° = 8


We know, cos (180° + θ) = – cos θ , sin (180° + θ) = – sin θ


⇒ – cos 45° – ysin 45° = 8



⇒ x + y + 8√2 = 0


Hence, the equation of line in normal form is x + y + 8√2 = 0



Question 4.

Find the equation of a line for which

p = 8, α = 300°


Answer:

Given: p = 8, α = 300°

Concept Used:


Equation of line in normal form.


Explanation:


So, the equation of the line in normal form is


Formula Used: x cos α + y sin α = p


x cos 300° + y sin 300° = 8


⇒ x cos (360° – 60°) + y sin (360° – 60°) = 8


We know, cos (360° – θ) = cos θ, sin (360° – θ) = – sin θ


⇒ x cos60° – y sin60° = 8



⇒ x – √3y = 16


Hence, the equation of line in normal form is x – √3y = 16



Question 5.

Find the equation of the line on which the length of the perpendicular segment from the origin to the line is 4 and the inclination of the perpendicular segment with the positive direction of x–axis is 30°.


Answer:

Given: p = 4, α = 30∘

Concept Used:


Equation of line in normal form.


Explanation:


So, the equation of the line in normal form is


Formula Used: x cos α + y sin α = p


⇒ x cos30° + y sin30° = 4


⇒ x + y = 4


cos 30°, sin 30°


⇒ √3 x + y = 8


Hence, the equation of line is √3 x + y = 8.



Question 6.

Find the equation of the line whose perpendicular distance from the origin is 4 units and the angle which the normal makes with the positive direction of x–axis is 15°.


Answer:

Given: p = 4, α = 15°

Concept Used:


Equation of line in normal form.


Explanation:


We know that, cos 15° = cos (45° – 30°) = cos45°cos30° + sin45°sin30°


cos(A – B) = cosAcosB + sinAsinB



And sin 15 = sin (45° – 30°) = sin 45° cos 30° – cos 45° sin 30°


sin (A – B) = sinAcosB – cosAsinB



So, the equation of the line in normal form is


Formula Used: x cos α + y sin α = p



⇒ (√3 + 1)x + (√3 – 1)y = 8√2


Hence, the equation of line in normal form is (√3 + 1)x + (√3–1)y = 8√2



Question 7.

Find the equation of the straight line at a distance of 3 units from the origin such that the perpendicular from the origin to the line makes an angle α given by with the positive direction of x–axis.


Answer:

Given: p = 3,


and


Concept Used:


The equation of a line in normal form.


Explanation:


So, the equation of the line in normal form is


Formula Used: x cos α + y sin α = p



⇒ 12x + 5y = 39


Hence, the equation of line in normal form is 12x + 5y = 39



Question 8.

Find the equation of the straight line on which the length of the perpendicular from the origin is 2 and the perpendicular makes an angle α with x–axis such that .


Answer:

Given: p = 2, sinα = 1/3

We know that, cos



Concept Used:


The equation of a line in normal form.


Explanation:


So, the equation of the line in normal form is


Formula Used: x cos α + y sin α = p



⇒ 2√2x + y = 6


Hence, the equation of line in normal form is 2√2x + y = 6



Question 9.

Find the equation of the straight line upon which the length of the perpendicular from the origin is 2, and the slope of this perpendicular is .


Answer:

Assuming:

The perpendicular drawn from the origin make acute angle α with the positive x–axis. Then, we have, tanα = 5/12


We know that, tan(180∘ + α) = tanα


So, there are two possible lines, AB and CD, on which the perpendicular drawn from the origin has a slope equal to 5/12 .


Given:


Now tan α = 5/12



Explanation:


So, the equations of the lines in normal form are


Formula Used: x cos α + y sin α = p


⇒ x cos α + y sin α = p and x cos(180° + α) + ysin(180° + α) = p


⇒ x cos α + y sin α = 2 and –x cos α – ysin α = 2


cos (180° + θ) = – cos θ , sin (180° + θ) = – sin θ


and 12x + 5y = – 26


Hence, the equation of line in normal form is and 12x + 5y = – 26



Question 10.

The length of the perpendicular from the origin to a line is 7, and the line makes an angle of 1500 with the positive direction of y–axis. Find the equation of the line.


Answer:

Assuming:


AB be the given line which makes an angle of 1500 with the positive direction of y–axis and OQ be the perpendicular drawn from the origin on the line.


Given:


p = 7 and α = 30°


Explanation:


So, the equation of the line AB is


Formula Used: x cos α + y sin α = p


⇒ x cos 30° + y sin 30° = 7



⇒ √3 x + y = 14


Hence, the equation of line in normal form is √3 x + y = 14



Question 11.

Find the value of θ and p if the equation x cos θ + y sin θ = p is the normal form of the line .


Answer:

Given: the normal form of a line is x cos θ + y sin θ = p …..… (1)

To find:


P and θ.


Explanation:


Let us try to write down the equation √3 + y + 2 = 0 in its normal form.


Now √3 + y + 2 = 0


⇒ √3 + y = – 2


Dividing both sides by 2,


⇒ – √3/2 – y/2 = 1


…… (2)


Comparing equations (1) and (2) we get,


and p = 1


⇒ θ = 210° = 7π/6 and p = 1


Hence, θ = 210° = 7π/6 and p = 1



Question 12.

Find the equation of the straight line which makes a triangle of the area with the axes and perpendicular from the origin to it makes an angle of 300 with y–axis.


Answer:

Assuming:


AB be the given line, and OL = p be the perpendicular drawn from the origin on the line.


Given:


α = 60°


Explanation:


So, the equation of the line AB is


Formula Used: x cos θ + y sin θ = p


⇒ x cos 60° + y sin 60° = p



⇒ x + √3y = 2p …… (1)


Now, in triangles OLA and OLB


Cos 60° cos30°


and


⇒ OA = 2p and OB =


It is given that the area of triangle OAB is 96√3




⇒ p2 = 122


⇒ p = 12


Substituting the value of p in (1)


x + √3 y = 24


Hence, the equation of the line AB is x + √3 y = 24



Question 13.

Find the equation of a straight line on which the perpendicular from the origin makes an angle of 30° with x–axis and which forms a triangle of the area with the axes.


Answer:

Assuming: AB be the given line, and OL = p be the perpendicular drawn from the origin on the line.


Given: α = 60°


Explanation:


So, the equation of the line AB is


x cos θ + y sin θ = p


⇒ x cos 30 + y sin 30 = p



⇒ √3x + y = 2p …… (1)


Now, in triangles OLA and OLB


Cos 30° = , cos6° =


,


⇒ OA = and OB = 2p


It is given that the area of triangle OAB is 50√3




⇒ p2 = 75


⇒ p = √75


Substituting the value of p in (1)


√3 x + y = √75


Hence, the equation of the line AB is √3 x + y = √75




Exercise 23.8
Question 1.

A line passes through a point A (1, 2) and makes an angle of 600 with the x–axis and intercepts the line x + y = 6 at the point P. Find AP.


Answer:

Given: (x1,y1) = A(1, 2), θ = 60°

To find:


Distance AP.


Explanation:


So, the equation of the line is


Formula Used:




Here, r represents the distance of any point on the line from point A (1, 2).


The coordinate of any point P on this line are


Clearly, P lies on the line x + y = 6


⇒ 1 + + 2 + = 6


= 3


⇒ r (√3 + 1) = 6


⇒ r =


Therefore, AP = 3(√3 – 1)



Question 2.

A straight line drawn through the point A (2, 1) making an angle with positive x–axis intersects another line x + 2y + 1 = 0 in the point B. Find length AB.


Answer:

Given: (x1,y1) = A(2, 1), θ = 45°

To find:


Length AB.


Explanation:


So, the equation of the line is


Formula Used:




⇒ x – y – 1 = 0


Let PQ = r


Then, the coordinate of Q is given by



⇒ x , y


The coordinate of point Q is


Clearly, Q lies on the line x + 2y + 1 = 0




⇒ r


Hence, the length of AB is



Question 3.

A line a drawn through A (4, – 1) parallel to the line 3x – 4y + 1 = 0. Find the coordinates of the two points on this line which are at a distance of 5 units from A.


Answer:

Given: (x1,y1) = A(4, – 1)

To find:


Coordinates of the two points on this line which are at a distance of 5 units from A.


Explanation:


Line 3x – 4y + 1 = 0


⇒ 4y = 3x + 1


⇒ y


Slope


⇒ sin θ and cos θ


So, the equation of the line passing through A (4, −1) and having slope is


Formula Used:



⇒ 3x – 4y = 16


Here, AP = r = ± 5
Thus, the coordinates of P are given by




⇒ x and y


⇒ x and y


⇒ x= ±4 + 4 and y = ±3–1


So x = 8, 0 and y = 2, – 4


Hence, the coordinates of the two points at a distance of 5 units from A are (8, 2) and (0, −4).



Question 4.

The straight line through P(x1, y1) inclined at an angle θ with the x–axis meets the line ax + by + c = 0 in Q. Find the length of PQ.


Answer:

Given: the equation of the line that passes through P(x1, y1) and makes an angle of θ with the x–axis.

To find:


Length of PQ.


Explanation:



Let PQ = r
Then, the coordinates of Q are given by


Formula Used:



Thus, the coordinates of Q are


Clearly, Q lies on the line ax + by + c = 0.



⇒ r


∴ PQ


Thus, length PQ



Question 5.

Find the distance of the point (2, 3) from the line 2x – 3y + 9 = 0 measured along a line making an angle of 45° with the x–axis.


Answer:

Given: (x1,y1) = A(2, 3), θ = 45°

To find:


Distance of point from line.


Explanation:


So, the equation of the line is


Formula Used:




⇒ x – y + 1 = 0


Let PQ = r


Then, the coordinate of Q are given by



⇒ x, y


The coordinate of point Q is


Clearly, Q lies on the line 2x – 3y + 9 = 0




⇒ r = 4√2


Hence, the distance of the point from the given line is 4√2.



Question 6.

Find the distance of the point (3, 5) from the line 2x + 3y = 14 measured parallel to a line having slope 1/2.


Answer:

Given: (x1,y1) = A(3, 5), tanθ

⇒ sin θ and cos θ


To find:


The distance of a point from the line parallel to another line.


Explanation:


Formula Used:



⇒ x – 2y + 7 = 0


Let x – 2y + 7 = 0 intersect the line 2x + 3y = 14 at point P.


Let AP = r


Then, the coordinate of P is given by



⇒ x and y


Thus, the coordinate of P is


Clearly, P lies on the line 2x + 3y = 14





⇒ r


Hence, the distance of the point (3, 5) from the line 2x + 3y = 14 is



Question 7.

Find the distance of the point (2, 5) from the line 3x + y + 4 = 0 measured parallel to a line having slope 3/4.


Answer:

Given: (x1,y1) = A(2, 5), tanθ

⇒ sin θ and cos θ


To find:


The distance of a point from the line parallel to another line.


Explanation:


So, the equation of the line passing through (2, 5) and having a slope is


Formula Used:



⇒ 3x – 4y + 14 = 0


Let 3x – 4y + 7 = 0 intersect the line 3x + y + 4 = 0 at point P.


Let AP = r


Then, the coordinate of P are given by



⇒ x and y


Thus, the coordinate of P is


Clearly, P lies on the line 3x + y + 4 = 0




⇒ 3r = – 15


⇒ r = – 5


Hence, the distance of the point (2, 5) from the line 3x + y + 4 = 0 is 5



Question 8.

Find the distance of the point (3, 5) from the line 2x + 3y = 14 measured parallel to the line x – 2y = 1.


Answer:

Given: (x1,y1) = A(3, 5)

To find:


The distance of a point from the line parallel to another line.


Explanation:


It is given that the required line is parallel to x – 2y = 1


⇒ 2y = x – 1


⇒ y



⇒ sin θ and cos θ


So, the equation of the line is


Formula Used:



⇒ x – 2y + 7 = 0


Let x – 2y + 7 = 0 intersect the line 2x + 3y = 14 at point P.


Let AP = r


Then, the coordinate of P is given by



⇒ x and y


Thus, the coordinate of P is


Clearly, P lies on the line 2x + 3y = 14





⇒ r


Hence, the distance of the point (3, 5) from the line 2x + 3y = 14 is



Question 9.

Find the distance of the line 2x + y = 3 from the point ( – 1, – 3) in the direction of the line whose slope is 1.


Answer:

Given: (x1,y1) = A( – 1, – 3)

And tan θ = 1 ⇒


To find:


The distance of a point from the line in the direction of the line.


Explanation:


So, the equation of the line passing through ( – 1, – 3) and having slope 1 is


Formula Used:



⇒ x – y = 2


Let x – y = 2 intersect the line 2x + y = 3 at point P.


Let AP = r


Then, the coordinate of P is given by



⇒ x and y


Thus, the coordinate of P is


Clearly, P lies on the line 2x + y = 3




⇒ 3r


⇒ r


Hence, the distance of the point ( – 1, – 3) from the line 2x + y = 3 is



Question 10.

A line is such that its segment between the straight line 5x – y – 4 = 0 and 3x + 4y – 4 = 0 is bisected at the point (1, 5). Obtain its equation.


Answer:

Assuming:


P1 P2 be the intercept between the lines 5x − y − 4 = 0 and 3x + 4y − 4 = 0.


P1 P2 makes an angle θ with the positive x–axis.


Given: (x1, y1) = A (1, 5)


Explanation:


So, the equation of the line passing through A (1, 5) is


Formula Used:




Let AP1 = AP2 = r


Then, the coordinates of P1 and P2 are given by


and


So, the coordinates of P1 and P2 are 1 + rcosθ, 5 + rsinθ and 1 – rcosθ, 5 – rsinθ, respectively.


Clearly, P1 and P2 lie on 5x − y − 4 = 0 and 3x + 4y − 4 = 0, respectively.


∴5(1 + rcos θ) – 5 – rsin θ – 4 = 0 and 3(1 – r cos θ) + 4(5 – rsin θ) + 4(5 – r sin θ) – 4 = 0


and



⇒ 95 cos θ – 19 sinθ = 12 cos + 16 sinθ


⇒ 83 cosθ = 35 sinθ


⇒ tan θ = 83/35


Thus, the equation of the required line is




⇒ 83x – 35y + 92 = 0


Hence, the equation of line is 83x – 35 y + 92 = 0



Question 11.

Find the equation of straight line passing through ( – 2, – 7) and having an intercept of length 3 between the straight lines 4x + 3y = 12 and 4x + 3y = 3.


Answer:

Given: (x1,y1) = A ( – 2, – 7)

To find:


Equation of required line.


Explanation:



So, the equation of the line is


Formula Used:



Let the required line intersect the lines 4x + 3y = 3 and 4x + 3y = 12 at P1 and P2.


Let AP1 = r1 and AP2 = r2


Then, the coordinates of P1 and P2 are given by and respectively. Thus, the coordinates of P1 and P2 are ( – 2 + r1cos θ, – 7 + r1sin θ) and ( – 2 + r2cos θ, – 7 + r2sin θ), respectively.


Clearly, the points P1 and P2 lie on the lines 4x + 3y = 3 and 4x + 3y = 12


4( – 2 + r1cos θ) + 3( – 7 + r1sin θ) = 3 and 4( – 2 + r2cos θ) + 3( – 7 + r2sin θ) = 12,


and


Here AP2 – AP1 = 3




⇒ 3 = 4 cos θ + 3 sin θ


⇒ 3(1 – sin θ) = 4cos θ


⇒ 9(1 + sin2 θ – 2sin θ) = 16 cos2 θ = 16(1 – sin2 θ)


⇒ 25 sin2 θ – 18 sin θ – 7 = 0


⇒ (sin θ – 1)(25 sin θ + 7 ) = 0


⇒ Sin θ = 1, sin θ = – 7/25


⇒ Cos θ = 0, cos θ = 24/25


Thus, the equation of the required line is


x + 2 = 0 or


⇒ x + 2 = 0 or 7x + 24 y + 182 = 0


Hence, the equation of line is x + 2 = 0 or 7x + 24 y + 182 = 0




Exercise 23.9
Question 1.

Reduce the equation to:

(i) slope - intercept form and find slope and y - intercept;

(ii) Intercept form and find intercept on the axes

(iii) The normal form and find p and α.


Answer:

(i) Given:


Explanation:



This is the slope intercept form of the given line.


Hence, slope = - √3 and y - intercept = - 2


(ii) Given:


Explanation:



Dividing both sides by - 2



Hence, the intercept form of the given line. Here, x - intercept = - 23 and y - intercept = - 2


(iii) Given:


Explanation:




Dividing both sides by



This is the normal form of the given line.


Hence, p = 1 cos and sinα = - 1/2


Hence, α = 210



Question 2.

Reduce the following equations to the normal form and find p and α in each case :



Answer:

Given:


Explanation:




Dividing both sides by



Hence, the normal form of the given line, where p = 2, cosα = 1/2 and sin α =


⇒ α = π/3



Question 3.

Reduce the following equations to the normal form and find p and α in each case :



Answer:

Given:


Explanation:




Dividing both sides by



Hence, the normal form of the given line, where p = 1, cosα = and sin α =


⇒ α = 225 [ The coefficient of xand y are negative So lies in third quadrant ]



Question 4.

Reduce the following equations to the normal form and find p and α in each case :



Answer:

Given:


Explanation:




Dividing both sides by



Hence, the normal form of the given line, where p = 2, cosα = and sin α =


⇒ α = 135


The coefficient of x and y are negative and positive respectively. So, α lies in the second quadrant



Question 5.

Reduce the following equations to the normal form and find p and α in each case :

x – 3 = 0


Answer:

Given: x – 3 = 0


Explanation:


⇒ x = 3


⇒ x + 0 × y = 3



Dividing both sides by


⇒ x + 0 × y = 3


Hence, the normal form of the given line, where p = 3, cosα = 1 and sin α = 0


⇒ α = 0



Question 6.

Reduce the following equations to the normal form and find p and α in each case :

y – 2 = 0


Answer:

Given: y – 2 = 0


Explanation:


⇒ y = 2


⇒ 0*x + y = 2



Dividing both sides by


⇒ 0 * x + y = 2


Hence, the normal form of the given line, where p = 2, cosα = 0 and sin α = 1


⇒ α = 90



Question 7.

Put the equation the slope intercept form and find its slope and y - intercept.


Answer:

Given: the equation is


Concept Used:


General equation of line y = mx + c.


Explanation:


bx + ay = ab


⇒ ay = - bx + ab


⇒ y = x + b


Hence, the slope intercept form of the given line.


∴ Slope = - b/a and y - intercept = b



Question 8.

Reduce the lines 3x – 4y + 4 = 0 and 2x + 4y – 5 = 0 to the normal form and hence find which line is nearer to the origin.


Answer:

Given:


The normal forms of the lines 3x − 4y + 4 = 0 and 2x + 4y − 5 = 0.


To find:


In given normal form of a line, Which is nearer to the origin.


Explanation:




Dividing both sides by


…… (1)


Now 2x + 4y = - 5


⇒ - 2x – 4y = 5



Dividing both sides by


…… (2)


From equations (1) and (2):


45<525


Hence, the line 3x − 4y + 4 = 0 is nearer to the origin.



Question 9.

Show that the origin is equidistant from the lines 4x + 3y + 10 = 0; 5x – 12y + 26 = 0 and 7x + 24y = 50.


Answer:

Given: The lines 4x + 3y + 10 = 0; 5x – 12y + 26 = 0 and 7x + 24y = 50.


To prove:


The origin is equidistant from the lines 4x + 3y + 10 = 0; 5x – 12y + 26 = 0 and 7x + 24y = 50.


Explanation:


Let us write down the normal forms of the given lines.


First line: 4x + 3y + 10 = 0


⇒ - 4x - 3y = 10



Dividing both sides by



∴ p = 2


Second line: 5x − 12y + 26 = 0


⇒ - 5x + 12y = 26



Dividing both sides by



∴ p = 2


Third line: 7x + 24y = 50



Dividing both sides by



∴ p = 2


Hence, the origin is equidistant from the given lines.



Question 10.

Find the values of θ and p, if the equation x cos θ + y sin θ = p is the normal form of the line .


Answer:

Given: The normal form of the line .

To find:


Value of θ and p.


Explanation:



Divide Both side by 2




Comparing the equations xcos θ + ysin θ = p we get,


cos θ, sin θ and p = 1


∴ θ = 210∘ and p = 1


Hence, θ = 210∘ and p = 1



Question 11.

Reduce the equation 3x – 2y + 6 = 0 to the intercept form and find the x and y - intercepts.


Answer:

Given: equation is 3x − 2y + 6 = 0


Concept Used:


Line in intercepts form is ( a and b are x and y intercepts resp.)


Explanation:


3x − 2y = - 6


[ Dividing both sides by - 6 ]



Thus, the intercept form of the given line


∴ x - intercept = −2 and y - intercept = 3



Question 12.

The perpendicular distance of a line from the origin is 5 units, and its slope is - 1. Find the equation of the line.


Answer:

Given: slope = - 1 and p = 5


Assuming: c be the intercept on the y - axis.


Explanation:


Then, the equation of the line is


y = - x + c [∴ m = - 1]


⇒ x + y = c



Dividing both sides by



This is the normal form of the given line.


Therefore, denotes the length of the perpendicular from the origin.


But, the length of the perpendicular is 5 units.



⇒ c =


Thus, substituting c = in y = - x + c , we get the equation of line to or